HARR Clinical Chemistry

Pataasin ang iyong marka sa homework at exams ngayon gamit ang Quizwiz!

A green-colored solution would show highest transmittance at: A. 475 nm B. 525 nm C. 585 nm D. 620 nm

B. 525 nm B Green light consists of wavelengths from 500-550 nm. A green-colored solution with a transmittance maximum of 525 nm and a 50-nm bandpass transmits light of 525 nm and absorbs light below 475 nm and above 575 nm. A solution that is green would be quantitated using a wavelength that it absorbs strongly, such as 450 nm.

All of the following are required when measuring magnesium by atomic absorption spectrophotometry except: A. A hollow cathode lamp with a magnesium cathode B. A chopper to prevent optical interference from magnesium emission C. A monochromator to isolate the magnesium emission line at 285 nm D. A 285-nm reference beam to correct for background absorption

D. A 285-nm reference beam to correct for background absorption D Atomic absorption requires a lamp with a cathode made from the metal to be assayed. The lamp emits the line spectrum of the metal, providing the wavelength that the atoms can absorb. The chopper pulses the source light, allowing it to be discriminated from light emitted by excited atoms. A monochromator eliminates light emitted by the ideal gas in the lamp. Deuterium (wide bandpass light) or Zeeman correction (splitting the incident light into side bands by a magnetic field) may be used to correct for background absorption.

Which statement about enzymes is true? A. An enzyme alters the Gibb's free energy of the reaction B. Enzymes cause a reaction with a positive free energy to occur spontaneously C. An enzyme's natural substrate has the highest Km D. A competitive inhibitor will alter the apparent Km of the reaction

D. A competitive inhibitor will alter the apparent Km of the reaction D Enzymes alter the energy of activation by forming a metastable intermediate, the enzyme substrate complex. Enzymes do not alter the free energy or direction of a reaction. Competitive inhibitors bind to the active site where the enzyme binds substrate and are overcome by increasing the substrate concentration.

Which statement regarding the diagnosis of iron deficiency is correct? A. Serum iron levels are always higher at night than during the day B. Serum iron levels begin to fall before the body stores become depleted C. A normal level of serum ferritin rules out iron deficiency D. A low serum ferritin is diagnostic of iron deficiency

D. A low serum ferritin is diagnostic of iron deficiency D Serum iron levels are falsely elevated by hemolysis and subject to diurnal variation. Levels are highest in the morning and lowest at night, but this pattern is reversed in persons who work at night. A low ferritin is specific for iron deficiency. However, only about 1% of ferritin is in the vascular system. Any disease that increases ferritin release may mask iron deficiency

Select the correct order of Hgb migration on citrate agar at pH 6.2. A. - F→S→C→A + B. - F→A→S→C + C. - A→S→F→C + D. - A→C→S→F +

B. - F→A→S→C + B In an acid buffer, the hemoglobins are expected to migrate to the cathode, with hemoglobin A being the slowest because it has the weakest net positive charge. However, Hgb C and Hgb S bind to sulfated pectins in the agar gel, forming a complex that is negatively charged causing them to migrate toward the anode. Hgb C migrates furthest toward the anode, followed by Hgb S. Hgb F migrates furthest toward the cathode. Hgbs A, A2, DPunjab, E, G, and HgbLepore migrate slightly toward the cathode

What is the pH of a solution of HNO3, if the hydrogen ion concentration is 2.5 × 10-2 M? A. 1.0 B. 1.6 C. 2.5 D. 2.8

B. 1.6 B For a strong acid, the pH is equal to the negativelogarithm of the hydrogen ion concentration. pH = -Log H+ pH = -Log 0.025 pH = 1.6

The normal difference between alveolar and arterial PO2 (PAO2-PaO2 difference) is: A. 3 mm Hg B. 10 mm Hg C. 40 mm Hg D. 50 mm Hg

B. 10 mm Hg B The PAO2-PaO2 difference results from the low ratio of ventilation to perfusion in the base of the lungs. The hemoglobin in the blood coming from the base of the lung has a lower O2 saturation. This blood will take up O2 from the plasma of blood leaving well-ventilated areas of the lung, thus lowering the mixed arterial PO2.

According to American Diabetes Association criteria, which result is consistent with a diagnosis of impaired fasting glucose? A. 99 mg/dL B. 117 mg/dL C. 126 mg/dL D. 135 mg/dL

B. 117 mg/dL B Impaired fasting glucose is defined as a plasma glucose ≥100 but <126 mg/dL. A fasting glucose of 126 or higher on two consecutive occasions indicates diabetes. A fasting glucose of 99 mg/dL is considered normal.

A glycerol kinase method for triglyceride calls for a serum blank in which normal saline is substituted for lipase in order to measure endogenous glycerol. Given the following results, and assuming the same volume of sample and reagent are used for each test, calculate the triglyceride concentration in the patient's sample. Std Concentration : 125mg/dL Absorbance of RB: 0.000 Abs of Std: 0.62 Abs of Pt Serum: 0.750 Abs of Serum Blank: 0.100 A. 119 mg/dL B. 131 mg/dL C. 156 mg/dL D. 180 mg/dL

B. 131 mg/dL B The serum blank absorbance is subtracted from the result for the patient's serum before applying the ratiometric formula to calculate concentration. Cu = [(Au - ASB)/As] × Cs where ASB = absorbance of serum blank = (0.750-0.100)/0.620 × 125 mg/dL = 131 mg/dL

Which whole-blood level is suggestive of excessive exposure to lead in children but not adults? A. 4 µg/dL B. 14 µg/dL C. 28 µg/dL D. 32 µg/dL

B. 14 µg/dL B Because lead exposure in children leads to learning impairment, the cutoff for exposure recommended by the Centers for Disease Control is 5 μg/dL in venous whole blood. Values of 5 μg/dL or more should be monitored closely with follow-up testing, and if they increase, steps should be taken to remove lead contamination from the home and environment. For adults the recommended cutoff is 25 μg/dL. Because lead readily enters the red blood cells, and passes from plasma to urine quickly, whole blood is a more sensitive measure of exposure than plasma. Because lead from the fingers may contaminate the specimen, a venous sample is preferred over a capillary sample collected by finger stick.

Which of the following values is the threshold critical value (alert or action level) for high plasma sodium? A. 150 mmol/L B. 160 mmol/L C. 170 mmol/L D. 180 mmol/L

B. 160 mmol/L B The adult reference range for plasma sodium is approximately 135-145 mmol/L. Levels in excess of 160 mmol/L are associated with severe dehydration, hypovolemia, and circulatory and heart failure. The threshold for the low critical value for sodium is 120 mmol/L. This is associated with edema, hypervolemia, and circulatory overload. Alert levels must also be established for potassium, bicarbonate, calcium, pH, PO2, glucose, bilirubin, hemoglobin, platelet count, and prothrombin time. When a sample result is below or above the low or high alert level, respectively, the physician must be notified immediately.

Which set of the following laboratory results is most likely from a patient who has suffered an AMI? Reference intervals are in parenthesis. Total CK | CK-MB | CK index (10-110 U/L) (1-4 µg/L) (1%-2.5%) A. 760 U/L 16 µg/L 2.1% B. 170 U/L 14 µg/L 8.2% C. 160 U/L 4 µg/L 2.5% D. 80 U/L 2 µg/L 2.5%

B. 170 U/L 14 µg/L 8.2% B Results shown in C and D can be excluded because the CK-MB is not increased. Results shown in A and B have CK-MB levels above the URL. However, patient A has a CK index under 2.5% and a 5- to 10-fold elevation of total CK. These results indicate release of a small of amount of CK-MB from skeletal muscle rather than from cardiac muscle. To maximize the sensitivity of CK-MB, laboratories use an URL of 4 or 5 μg/L. This cutoff can detect about two-thirds of AMI cases within 3 hours of the infarct, but requires the use of a conservative CK index and other cardiac markers to avoid a high number of false positives.

A procedure for cholesterol is calibrated with a serum-based cholesterol standard that was determined by the Abell-Kendall method to be 200.0 mg/dL. Assuming the same volume of sample and reagent are used, calculate the cholesterol concentration in the patient's sample from the following results. Standard Concentration /Absorbance of Reagent Blank / Absorbance of Standard/ Absorbance of Patient Serum 200 mg/dL/ 0.00 /0.860/ 0.740 A. 123 mg/dL B. 172 mg/dL C. 232 mg/dL D. 314 mg/dL

B. 172 mg/dL B Cu = Au/As × Cs where Cu = concentration of unknown, Au = absorbance of unknown, As = absorbance of standard, and Cs = concentration of standard. Cu = 0.740/0.860 × 200 mg/dL = 172 mg/dL

In most circumstances, when two controls within a run are both greater than ±2s from the mean, what action should be taken first? A. Recalibrate, then repeat controls followed by selected patient samples if quality control is acceptable B. Repeat the controls before taking any corrective action C. Change the reagent lot, then recalibrate D. Prepare fresh standards and recalibrate

A When a 22s rule is broken an SE is present and corrective action is required (repeating just the QC will not correct the problem). If recalibration yields acceptable QC results, both sets of QC results and the corrective action taken are documented in the QC log. If the magnitude of the error is large enough to be medically significant, then all patient samples since the last previously acceptable QC should be repeated. If in question, the magnitude of the error can be evaluated by repeating abnormal patient samples. If the average difference between results before and after recalibration is > 2s, then all samples should be repeated since the last acceptable QC.

Which of the following quality control (QC) rules would be broken 1 out of 20 times by chance alone? A. 12s B. 22s C. 13s D. 14s

A. 12s A The notation 12S means that one control is outside ±2 standard deviation units. QC results follow the bellshaped curve called the Gaussian (normal) distribution. If a control is assayed 100 times, 68 out of 100 results would fall within +1 s and -1 s of the mean. Ninetyfive (95.4) out of 100 results would fall within +2 s and -2 s. This leaves only 5 out of 100 results (1:20) that fall outside the ±2 s limit. Also, 99.7 out of 100 results fall within ±3 s of the mean.

Which PCO2 value would be seen in maximally compensated metabolic acidosis? A. 15 mm Hg B. 30 mm Hg C. 40 mm Hg D. 60 mm Hg

A. 15 mm Hg A In metabolic acidosis, hyperventilation increases the ratio of bicarbonate to dissolved CO2. The extent of compensation is limited by the rate of both gas diffusion and diaphragm contraction. The lower limit is between 10 and 15 mm Hg PCO2, which is the maximum compensatory effect.

Given the following data, calculate the coefficient of variation for glucose. Analyte / Mean / Std Deviation Glucose / 76mg/dL / 2.3 A. 3.0% B. 4.6% C. 7.6% D. 33.0%

A. 3.0% A The coefficient of variation is calculated by dividing the standard deviation by the mean and multiplying by 100. % CV = s/X × 100 = 2.3/76 × 100 = 3.0% The CV is the most appropriate statistic to use when comparing the precision of samples that have different means. For example, when comparing the precision of the level 1 control to the level 2 control, the coefficient of variation normalizes the variance to be independent of the mean. The control with the lower CV is the one for which the analysis is more precise.

Which wavelength would be absorbed strongly by a red-colored solution? A. 450 nm B. 585 nm C. 600 nm D. 650 nm

A. 450 nm A A solution transmits light corresponding in wavelength to its color, and usually absorbs light of wavelengths complementary to its color. A red solution transmits light of 600-650 nm and strongly absorbs 400-500 nm light.

Which of the following mechanisms is responsible for metabolic acidosis? A. Bicarbonate deficiency B. Excessive retention of dissolved CO2 C. Accumulation of volatile acids D. Hyperaldosteronism

A. Bicarbonate deficiency A Metabolic acidosis is caused by bicarbonate deficiency and metabolic alkalosis by bicarbonate excess. Respiratory acidosis is caused by PCO2 retention (defective ventilation), and respiratory alkalosis is caused by PCO2 loss (hyperventilation). Important causes of metabolic acidosis include renal failure, diabetic ketoacidosis, lactate acidosis, and diarrhea

Which metabolite is most often increased in carcinoid tumors of the intestine? A. 5-Hydroxyindolacetic acid (5-HIAA) B. 3-Methoxy-4-hydroxyphenylglycol (MHPG) C. 3-Methoxydopamine D. HVA

A. 5-Hydroxyindolacetic acid (5-HIAA A 5-HIAA is a product of serotonin catabolism. Excess levels are found in urine of patients with carcinoid tumors composed of argentaffin cells. Carcinoid tumors are usually found in the intestine or lung, and produce serotonin and 5 hydroxytryptophan, a serotonin precursor. Serotonin is deaminated by monamine oxidase, forming 5-HIAA, and the 5-HIAA is excreted in the urine. Some carcinoid tumors produce mainly 5-hydroxytryptophan because they lack an enzyme needed to convert it to serotonin. The 5-hydroxytryptophan is converted by the kidneys to serotonin resulting in high urinary serotonin. Both 5-HIAA and serotonin are usually measured by HPLC with EDC

Which of the following best represents the reference (normal) range for arterial pH? A. 7.35-7.45 B. 7.42-7.52 C. 7.38-7.68 D. 6.85-7.56

A. 7.35-7.45 A The reference range for arterial blood pH is 7.35-7.45 and is only 0.03 pH units lower for venous blood owing to the buffering effects of hemoglobin (Hgb) known as the chloride isohydric shift. Most laboratories consider less than 7.20 and greater than 7.60 the critical values for pH.

A method calls for extracting an acidic drug from urine with an anion exchange column. The pKa of the drug is 6.5. Extraction is enhanced by adjusting the sample pH to: A. 8.5 B. 6.5 C. 5.5 D. 4.5

A. 8.5 A Extraction of a negatively charged drug onto an anion exchange (positively charged) column is optimal when more than 99% of the drug is in the form of anion. The extraction pH should be 2 pH units above the pKa of an acidic drug. When pH = pKa the drug will be 50% ionized, and when pH is greater than pKa the majority of drug is anionic.

What is the recommended cutoff for the early detection of chronic kidney disease in diabetics using the test for microalbuminuria? A. >30 mg/g creatinine B. >80 mg/g creatinine C. >200 mg/g creatinine D. >80 mg/L

A. >30 mg/g creatinine A Microalbuminuria is the excretion of small quantities of albumin in the urine. In diabetics, excretion of albumin that is within allowable limits for healthy persons may signal the onset of chronic kidney disease. The term microalbuminuria is defined as albumin excretion ≥ 30 mg/g creatinine but ≤ 300 mg/g creatinine. The use of the albumin to creatinine ratio is preferred to measures of albumin excretory rate (μg/min) because the latter is subject to error associated with timed specimen collection. ADA recommends the test be done annually for all type 2 diabetics and type 1 diabetics who have had the disease for > 5 years

A flameless atomic absorption spectrophotometer dehydrates and atomizes a sample using: A. A graphite capillary furnace B. An electron gun C. A thermoelectric semiconductor D. A thermospray platform

A. A graphite capillary furnace A Flameless atomic absorption uses a hollow tube of graphite with quartz ends. The tube is heated in stages by an electric current to successively dry, ash, and atomize the sample. During the ash and atomization steps, argon is injected into the tube to distribute the atoms. The furnace is more sensitive than a flame atomizer and more efficient in atomizing thermostable salts. However, it is prone to greater matrix interference and is slower than the flame atomizer because it must cool down before introduction of the next sample.

Capillary electrophoresis differs from agarose gel electrophoresis in which respect? A. A stationary support is not used B. An acidic buffer is used C. A low voltage is used D. Electroendosmosis does not occur

A. A stationary support is not used A Capillary electrophoresis is a rapid automated procedure for separating serum or body fluid proteins. Instead of a stationary support, the proteins migrate based upon their charge/mass ratio inside a small-bore silica capillary tube (20-200 μm). The cations in the buffer are attracted to the negatively charged silicates and migrate to the cathode rapidly when voltage is applied. The electroendosmotic force created moves the proteins toward the cathode and they are detected by an in-line UV photometer that measures their absorbance. High voltage (e.g., 9,000 volts) is used to effect separation of serum proteins in an 8-10 minute run, giving resolution equal to or greater than HR agarose gel electrophoresis.

In which liver disease is the DeRitis ratio (ALT:AST) usually greater than 1.0? A. Acute hepatitis B. Chronic hepatitis C. Hepatic cirrhosis D. Hepatic carcinoma

A. Acute hepatitis A ALT prevails over AST in hepatitis; however, AST is greater than ALT in carcinoma, alcoholic liver disease, and cirrhosis of the liver.

Which condition gives rise to the highest serum level of transaminases? A. Acute hepatitis B. Alcoholic cirrhosis C. Obstructive biliary disease D. Diffuse intrahepatic cholestasis

A. Acute hepatitis A The transaminases usually reach 20-50 times the URL in acute viral and toxic hepatitis. Both transaminases are moderately increased (5-10 × URL) in infectious mononucleosis, toxic hepatitis, diffuse intrahepatic obstruction, lymphoma, and cancer of the liver, and slightly increased (2-5 × URL) in cirrhosis and extrahepatic obstruction

Bilirubin is transported from reticuloendothelial cells to the liver by: A. Albumin B. Bilirubin-binding globulin C. Haptoglobin D. Transferrin

A. Albumin A Albumin transports bilirubin, haptoglobin transports free Hgb, and transferrin transports ferric iron. When albumin binding is exceeded, unbound bilirubin, called free bilirubin, increases. This may cross the blood-brain barrier, resulting in kernicterus.

Which of the following statements regarding the TIBC assay is correct? A. All TIBC methods require addition of excess iron to saturate transferrin B. All methods require the removal of unbound iron C. Measurement of TIBC is specific for transferrin-bound iron D. The chromogen used must be different from the one used for measuring serum iron

A. All TIBC methods require addition of excess iron to saturate transferrin A All TIBC methods require addition of excess iron to saturate transferrin. Excess iron is removed by ion exchange or alumina gel columns or precipitation with MgCO3 and the bound iron is measured by the same procedure as is used for serum iron. Alternatively, excess iron in the reduced state can be added at an alkaline pH. Under these conditions, transferrin will bind Fe2+ and the unbound Fe2+ can be measured directly.

In a nonmyocardial as opposed to a myocardial cause of an increased serum or plasma CK-MB, which would be expected? A. An increase in CK-MB that is persistent B. An increase in the percent CK-MB as well as concentration C. The presence of increased TnI D. A more modest increase in total CK than CK-MB

A. An increase in CK-MB that is persistent A Plasma CK-MB becomes abnormal 4 hours postinfarction, peaks in 16-20 hours, and usually returns to normal within 48 hours. In some noncardiac causes of elevated plasma CK-MB such as muscular dystrophy, there is a persistent elevation of both total CK and CK-MB. TnI and TnT are cardiac-specific markers. They become elevated slightly before CK-MB when a CK-MB URL of 4 μg/L is used, remain elevated for 7-10 days following an AMI, and are not increased in muscular dystrophy, malignant hyperthermia, or crush injuries that are associated with an increase in the concentration of CK-MB. Absolute CK-MB increases are evaluated cautiously, when CK-MB is less than 2.5% of total enzyme because noncardiac sources may be responsible.

The following conditions are all causes of alkalosis. Which condition is associated with respiratory (rather than metabolic) alkalosis? A. Anxiety B. Hypovolemia C. Hyperaldosteronism D. Hypoparathyroidism

A. Anxiety A Respiratory alkalosis is caused by hyperventilation, which leads to decreased PCO2. Anxiety and drugs such as epinephrine that stimulate the respiratory center are common causes of respiratory alkalosis. Excess aldosterone increases net acid excretion by the kidneys. Low parathyroid hormone causes increased bicarbonate reabsorption, resulting in alkalosis. Hypovolemia increases the relative concentration of bicarbonate. This is common and is called dehydrational alkalosis, chloride responsive alkalosis, or alkalosis of sodium deficit.

Which of the following statements is true? A. Apoenzyme + prosthetic group = holoenzyme B. A coenzyme is an inorganic molecule required for activity C. Cofactors are as tightly bound to the enzyme as prosthetic groups D. All enzymes have optimal activity at pH 7.00

A. Apoenzyme + prosthetic group = holoenzyme A A coenzyme is an organic molecule required for full enzyme activity. A prosthetic group is acoenzyme that is tightly bound to the apoenzyme and is required for activity. Cofactors are inorganic atoms or molecules needed for full catalytic activity. Pyridoxyl5´ phosphate is a prosthetic group for ALT and AST. Consequently, patients with low levels of pyridoxal5´ phosphate (P-5´-P) (vitamin B6 deficiency) may have reduced transaminase activity in vitro. Enzymes can have diverse pH (and temperature) optimas

Which apoprotein is inversely related to risk of coronary heart disease? A. Apoprotein A-I B. Apoprotein B100 C. Apoprotein C-II D. Apoprotein E4

A. Apoprotein A-I A Apoprotein A-I and apo A-II are the principal apoproteins of HDL, and low apo A-I has a high correlation with atherosclerosis. Conversely, apo-B100 is the principal apoprotein of LDL, and an elevated level is a major risk factor in developing coronary heart disease. Apoprotein assays are not recommended as screening tests because they are not as well standardized as LDL cholesterol assays. However, apo-B100 assay is more sensitive than LDL cholesterol in predicting coronary artery disease risk. Apo-B100 may be abnormal in persons with increased small dense LDL. Small dense LDL is more atherogenic than large LDL molecules. In addition, persons with hyperapobetalipoproteinemia overproduce apo-B100 without having significantly elevated LDL cholesterol

A patient presents to the emergency department with symptoms of intoxication including impaired speech and movement. Te plasma osmolality was measured and found to be 330 mOs/kg. Te osmolal gap was 40 mOsm/Kg. A blood alcohol was measured by the alcohol dehydrogenase method and found to be 0.15% w/v (150 mg/dL). Electrolyte results showed an increased anion gap. Ethylene glycol intoxication was suspected because the osmolal gap was greater than could be explained by ethanol alone, but gas chromatography was not available. Which of the following would be abnormal if this suspicion proved correct? A. Arterial blood gases B. Lactic acid C. Urinary ketones D. Glucose

A. Arterial blood gases A Ethylene glycol is sometimes used as a substitute forethanol by alcoholics. It is metabolized to formic acid and glycolic acid by the liver, resulting in metabolic acidosis and an increased anion gap. Lactic acid, glucose, and urinary ketones would be useful in ruling out other causes of metabolic acidosis, but would not be abnormal as a result of ethylene glycol intoxication.

Which of the following tumor markers is classified as a tumor suppressor gene? A. BRCA-1 B. Carcinoembryonic antigen (CEA) C. Human chorionic gonadotropin (hCG) D. Nuclear matrix protein

A. BRCA-1 A Tumor markers may be enzymes, hormones, receptors, oncofetal (glycoprotein) antigens, or oncogenes. BRCA-1 is located on the long arm of chromosome 17 and carries an 85% lifetime risk of breast or ovarian cancer when present. Its product functions in DNA repair and slows cell proliferation

In real-time PCR, what value is needed in order to determine the threshold? A. Background signal B. Melt temperature C. Maximum fluorescence D. Treshold cycle

A. Background signal A In real-time PCR, the fluorescence of the reporter probe is proportional to the concentration of PCR products. For quantitation of PCR products, a well factor and background fluorescence must be determined. Well-factor values are analogous to cuvette blanks. They are used to correct the measurements from each well so that the same concentration of fluorescent dye gives the same signal intensity regardless of the well. The threshold is the lowest signal that indicates the presence of product. It can be calculated manually from a real-time amplification curve by finding the average standard deviation of the fluorescent signal (RFU) from cycles 2-10. This is multiplied by 10 to give the threshold value in RFUs.

Which statement regarding bilirubin metabolism is true? A. Bilirubin undergoes rapid photo-oxidation when exposed to daylight B. Bilirubin excretion is inhibited by barbiturates C. Bilirubin excretion is increased by chlorpromazine D. Bilirubin is excreted only as the diglucuronide

A. Bilirubin undergoes rapid photo-oxidation when exposed to daylight A Samples for bilirubin analysis must be protected from direct sunlight. Drugs may have a significant in vivo effect on bilirubin levels. Barbiturates lower serum bilirubin by increasing excretion. Other drugs that cause cholestasis, such as chlorpromazine, increase the serum bilirubin. Although most conjugated bilirubin is in the form of diglucuronide, some monoglucuronide and other glycosides are excreted. In glucuronyl transferase deficiency, some bilirubin is excreted as sulfatides.

What are the likely laboratory findings in a person suspected of having Wilson's disease? A. Blood copper and ceruloplasmin low, urinary copper excretion high B. Blood and urine copper concentration high, ceruloplasmin low C. Blood and urine copper concentration high, ceruloplasmin high D. Blood and urine copper concentration low, ceruloplasmin low

A. Blood copper and ceruloplasmin low, urinary copper excretion high A Wilson's disease is an autosomal recessive disease in which copper transport is abnormal. The gene causing the disease codes for an ATPase (called Wilson's protein or ATP7B) that is needed to excrete copper into bile and incorporate copper into ceruloplasmin. There are over 200 reported mutations of this gene. The absence of Wilson's protein results in failure to load ceruloplasmin with copper, dramatically reducing its half-life in blood. Therefore, blood levels of ceruloplasmin are low, and blood levels of copper are usually low because there is little ceruloplasmin to bind it. Copper deposits in tissues, particularly the liver and brain, causing necrosis, and excess is excreted in the urine.

In gas chromatography, the elution order of volatiles is usually based upon the: A. Boiling point B. Molecular size C. Carbon content D. Polarity

A. Boiling point A The order of elution is dependent upon the velocity of the analyte. Usually, the lower the boiling point of the compound, the greater its velocity or solubility in carrier gas.

Which of the following compounds can interfere with the coulometric chloride assay? A. Bromide B. Ascorbate C. Acetoacetate D. Nitrate

A. Bromide A Chloride assays based upon either coulometric or chemical titration are subject to positive interference from other anions and electronegative radicals that may be titrated instead of chloride ions. These include other halogens such as bromide, cyanide, and cysteine.

Which of the following assays has the poorest precision? Analyte / Mean (mmol/L) / Std Deviation A. Ca / 2.5 / 0.3 B. K / 4.0 / 0/4 C. Na / 140 / 4.0 D. Cl / 100 / 2.5

A. Ca / 2.5 / 0.3 A Although calcium has the lowest s, it represents the assay with poorest precision. Relative precision between different analytes or different levels of the same analyte must be evaluated by the coefficient of variation (CV) because standard deviation is dependent upon the mean. CV = s × 100/Mean. This normalizes standard deviation to a mean of 100. The CV for calcium in the example is 12.0%.

In which circumstance will the reporting of calculated oxygen saturation of hemoglobin based on PO2, PCO2, pH, temperature, and hemoglobin be in error? A. Carbon monoxide poisoning B. Diabetic ketoacidosis C. Patient receiving oxygen therapy D. Assisted ventilation for respiratory failure

A. Carbon monoxide poisoning A CO has about 200 times the affinity as O2 for hemoglobin and will displace O2 from hemoglobin at concentrations that have no significant effect on the PAO2. Consequently, calculated oxygen saturation will be erroneously high. Other cases in which the calculated O2Sat should not be used include any hemoglobinopathy that affects oxygen affinity and methemoglobinemia. The other situations above affect the O2 saturation of hemoglobin in a manner that can be predicted by the effect of pH, PO2, and PCO2 on the oxyhemoglobin dissociation curve

Which of the electrodes below is a currentproducing (amperometric) rather than a voltage-producing (potentiometric) electrode? A. Clark electrode B. Severinghaus electrode C. pH electrode D. Ionized calcium electrode

A. Clark electrode A The Clark electrode is composed of two half cells that generate current, not voltage. It is used to measure partial pressure of oxygen (PO2), and is based upon an amperometric method called polarography. When -0.8 V is applied to the cathode, O2 is reduced, causing current to flow. Current is proportional to the PO2 of the sample.

For drugs with first-order elimination, which statement about drug clearance is true? A. Clearance = elimination rate ÷ serum level B. It is most often performed by the liver C. It is directly related to half-life D. Clearance rate is independent of dose

A. Clearance = elimination rate ÷ serum level A First-order elimination represents a linear relationship between the amount of drug eliminated per hour and the blood level of drug. For drugs following linear kinetics, clearance equals the elimination rate divided by the drug concentration in blood. When clearance (in milligrams per hour) and f are known, the dose per hour needed to give a desired average drug level at steady state can be calculated. Clearance is inversely related to the drug's half-life and is accomplished mainly by the kidneys.

The reference potential of a silver-silver chloride electrode is determined by the: A. Concentration of the potassium chloride filling solution B. Surface area of the electrode C. Activity of total anion in the paste covering the electrode D. The concentration of silver in the paste covering the electrode

A. Concentration of the potassium chloride filling solution A The activity of any solid or ion in a saturated solution is unity. For a silver electrode covered with silver chloride paste, the Nernst equation is E = E° - RT/nF x 2.3 log10 [Ag° × Cl-]/[AgCl]. Because silver and silver chloride have an activity of 1.0, and all components except chloride are constants, the potential of the reference electrode is determined by the chloride concentration of the filling solution. E = Eo - RT/nF × 2.3 log10[Cl-] = E° - 59.2 mV × log[Cl-] (at room temperature)

The purpose of the nebulizer in an atomic absorption spectrophotometer that uses a flame is to: A. Convert ions to atoms B. Cause ejection of an outer shell electron C. Reduce evaporation of the sample D. Burn off organic impurities

A. Convert ions to atoms A The atomizer of the atomic absorption spectrophotometer consists of either a nebulizer and flame or a graphite furnace. The nebulizer disperses the sample into a fine aerosol, distributing it evenly into the flame. Heat from the flame is used to evaporate water and break the ionic bonds of salts, forming ground state atoms. The flame also excites a small percentage of the atoms, which release a characteristic emission line.

Which is normally the most abundant corticosteroid hormone secreted by the adrenal cortex? A. Cortisol B. Dehydroepiandrosterone C. Aldosterone D. Corticosterone

A. Cortisol A Cortisol is the most abundant adrenal hormone, and abnormal levels have pronounced effects on carbohydrate and lipid metabolism. Cortisol is a 21-carbon steroid with a dihydroxyacetone group at C17 and hydroxyl group at C11 that account for its glucocorticoid potency. Plasma and urinary cortisol measurements are used to diagnose most types of adrenocortical dysfunction. Dehydroepiandrosterone (DHEA), an adrenal androgen, is the next most abundant adrenal hormone. Aldosterone is the principal mineral corticoid made by the adrenals, and corticosterone is the immediate precursor to aldosterone. Both regulate salt balance

Which of the following enzymes is increased in persons with prostate and small-cell lung cancer? A. Creatine kinase-1 (CK-1) B. Gamma glutamyl transferase (GGT) C. Amylase D. Lactate dehydrogenase

A. Creatine kinase-1 (CK-1) A CK-1 (CK-BB) is not normally found in plasma or serum except in neonates. It may be present in persons with central nervous system damage and some other disorders but its presence is often associated with various malignancies, especially prostate cancer and small cell carcinoma of the lung. Several other commonly measured enzymes are elevated by malignancy. ALP and LD are associated with various tumors. GGT levels are very high in hepatoma, and amylase is elevated in pancreatic cancer.

Which of the following formulas is the correct expression for creatinine clearance? A. Creatinine clearance = U/P X V X 1.73/A B. Creatinine clearance = P/V X U X A/1.73 C. Creatinine clearance = P/V X U X 1.73/A D. Creatinine clearance = U/V X P X 1.73/A

A. Creatinine clearance = U/P X V X 1.73/A A Clearance is the volume of plasma that contains the same quantity of substance that is excreted in the urine in 1 minute. Creatinine clearance is calculated as the ratio of urine creatinine to plasma creatinine in milligrams per deciliter. This is multiplied by the volume of urine produced per minute and corrected for lean body mass by multiplying by 1.73/A, where A is the patient's body surface area in square meters. Separate reference ranges are needed for males, females, and children because each has a different percentage of lean muscle mass.

Which of the following statements is true? A. Cystatin C is measured immunochemically B. The calibrator used for cystatin C is traceable to the National Bureau of Standards calibrator C. Cystatin C assays have a lower coefficient of variation than plasma creatinine D. Enzymatic and rate Jaffe reactions for creatinine give comparable results

A. Cystatin C is measured immunochemically A Cystatin C can be measured by enzyme immunoassay, immunonephelometry, and immunoturbidimetry. However, there is no standardized calibrator as for creatinine, and therefore, results vary considerably from lab to lab. The coefficient of variation for these methods tends to be slightly higher than for creatinine. Since the enzymatic methods are specific, they give lower plasma creatinine results than the Jaffe method in persons with normal renal function. However, they tend to give higher clearance results than for inulin or iohexol clearance because some creatinine is secreted by the renal tubules.

If the steady-state drug level is too high, the best course of action is to: A. Decrease the dose B. Decrease the dose interval C. Decrease the dose and decrease the dose interval D. Change the route of administration

A. Decrease the dose A Decreasing both dose and dosing interval will have offsetting effects on peak and trough blood levels. The appropriate dose can be calculated if the clearance or Vd and f are known. For example, the initial dose is calculated by multiplying the desired peak blood drug concentration by the Vd.

Hyperalbuminemia is caused by: A. Dehydration syndromes B. Liver disease C. Burns D. Gastroenteropathy

A. Dehydration syndromes A A high serum albumin level is caused only by dehydration or administration of albumin. Liver disease, burns, gastroenteropathy, nephrosis, starvation, and malignancy cause hypoalbuminemia.

Alcoholism, liver failure, and hypoxia induce acidosis by causing: A. Depletion of cellular NAD+ B. Increased excretion of bicarbonate C. Increased retention of PCO2 D. Loss of carbonic anhydrase

A. Depletion of cellular NAD+ A Oxygen debt and liver failure block oxidative phosphorylation, preventing NADH from being oxidized back to NAD+. Oxidation of ethanol to acetate results in accumulation of NADH. When NAD+ is depleted, glycolysis cannot proceed. It is regenerated by reduction of pyruvate to lactate, causing lactate acidosis.

A trend in QC results is most likely caused by: A. Deterioration of the reagent B. Miscalibration of the instrument C. Improper dilution of standards D. Electronic noise

A. Deterioration of the reagentA A trend occurs when six or more consecutive quality control results either increase or decrease in the same direction; however, this is not cause for rejection until a multirule is broken. Trends are systematic errors (affecting accuracy) linked to an unstable reagent, calibrator, or instrument condition. For example, loss of volatile acid from a reagent causes a steady pH increase, preventing separation of analyte from protein. This results in lower QC results each day.

Which of the following conditions is associated with hypernatremia? A. Diabetes insipidus B. Hypoaldosteronism C. Burns D. Diarrhea

A. Diabetes insipidus A Diabetes insipidus results from failure to produce ADH. Because the collecting tubules are impermeable to water in the absence of ADH, severe hypovolemia and dehydration result. Hypovolemia stimulates aldosterone release, causing sodium reabsorption, which worsens the hypernatremia. Burns, hypoaldosteronism, diarrhea, and diuretic therapy are common causes of hyponatremia.

Which of the following conditions is classified as normochloremic acidosis? A. Diabetic ketoacidosis B. Chronic pulmonary obstruction C. Uremic acidosis D. Diarrhea

A. Diabetic ketoacidosis A Bicarbonate deficit will lead to hyperchloremia unless the bicarbonate is replaced by an unmeasured anion. In diabetic ketoacidosis, acetoacetate and other ketoacids replace bicarbonate. The chloride remains normal or low and there is an increased anion gap

Which of the following conditions is associated with hyponatremia? A. Diuretic therapy B. Cushing's syndrome C. Diabetes insipidus D. Nephrotic syndrome

A. Diuretic therapy A Diuretics lower blood pressure by promoting water loss. This is accomplished by causing sodium loss from the proximal tubule and/or loop. Addison's disease, syndrome of inappropriate ADH release, burns, diabetic ketoacidosis, hypopituitarism, vomiting, diarrhea, and cystic fibrosis also cause hyponatremia. Cushing's syndrome causes hypernatremia by promoting sodium reabsorption in the collecting tubule in exchange for potassium. Diabetes insipidus and nephrotic syndrome promote hypernatremia by causing water loss

Which formula can be used to estimate dosage needed to give a desired steady-state blood level? A. Dose per hour = clearance (milligrams per hour) × average concentration at steady state ÷ f B. Dose per day = fraction absorbed - fraction excreted C. Dose = fraction absorbed × (1/protein-bound fraction) D. Dose per day = half-life × log Vd (volume distribution)

A. Dose per hour = clearance (milligrams per hour) × average concentration at steady state ÷ f A After a patient receives a loading dose to rapidly bring the drug level up to the desired therapeutic range, a maintenance dose must be given at consistent intervals to maintain the blood drug level at the desired concentration. The dose per hour is determined by multiplying the clearance per hour by the desired average steady-state concentration, then dividing by f (bioavailable fraction).

Which fact must be considered when evaluating a patient who displays signs of drug toxicity? A. Drug metabolites (e.g., N-acetylprocainamide) may need to be measured as well as parent drug B. If the concentration of total drug is within therapeutic limits, the concentration of free drug cannot be toxic C. If the drug has a wide therapeutic index, then it will not be toxic D. A drug level cannot be toxic if the trough is within the published therapeutic range

A. Drug metabolites (e.g., N-acetylprocainamide) may need to be measured as well as parent drug A Altered drug pharmacokinetics may result in toxicity even when the dose of drug is within the accepted therapeutic range. Two common causes of this are the presence of unmeasured metabolites that arephysiologically active, and the presence of a higher than expected concentration of free drug. Because only free drug is physiologically active, decreased binding protein or factors that shift the equilibrium favoring more unbound drug can result in toxicity when the total drug concentration is within the therapeutic range. Some drugs with a wide therapeutic index are potentially toxic because they may be ingested in great excess with little or no initial toxicity. For example, acetaminophen overdose does not usually become apparent until 3-5 days after the overdose. This creates the potential for hepatic damage to occur from continued use, especially in patients who have decreased hepatic or renal function because the drug half-life is extended

Which set of results for ER and PR is associated with the highest likelihood of a favorable response to treatment with estrogen-suppression therapy (tamoxifen)? A. ER positive, PR positive B. ER positive, PR negative C. ER negative, PR positive D. ER negative, PR negative

A. ER positive, PR positive A Both ER and PR receptor assays are performed on breast tissue biopsies to determine the probability of response to tamoxifen. The PR receptor is produced from the ER receptor and expression of both predicts a positive response to the drug. Less than 15% of persons who are ER negative and PR negative have a favorable response, whereas over 75% of those who are positive for both receptors have a favorable response to tamoxifen.

At pH 8.6, the cathodal movement of γ globulins is caused by: A. Electroendosmosis B. Wick flow C. A net positive charge D. Cathodal sample application

A. Electroendosmosis A Agarose and cellulose acetate contain fixed anions (e.g., acetate) that attract counterions when hydrated with buffer. When voltage is applied the cations migrate to the cathode, creating an osmotic force that draws H2O with them. This force, called electroendosmosis, opposes protein migration toward the anode and may cause some γ-globulins to be displaced toward the cathode.

What component is used in a GC-MS but not used in an LC-MS? A. Electron source B. Mass filter C. Detector D. Vacuum

A. Electron source A The mass spectrometer requires a sample that is suspended in a gas phase, and therefore, the sample from a GC can be directly injected into the mass spectrometer. While chemical ionization of the sample is possible, most GC-MS instruments utilize electron ionization. Electrons are produced by applying 70 electron volts to a filament of tungsten or rhenium under vacuum. The electrons collide with the neutral molecules coming from the GC, splitting them into fragments. The array of fragments is a unique identifier of each molecule

What process is most often used in LC-MS to introduce the sample into the mass filter? A. Electrospray ionization B. Chemical ionization C. Electron impact ionization D. Fast atom bombardment

A. Electrospray ionization A HPLC instruments use solvent rather than gas to separate molecules. The sample is converted into a gaseous state by electrospray ionization before it enters the mass filter. Electrospray ionization uses a small-bore tube that forms a 1-4 μ nozzle at the mass filter inlet and which is charged by several kilovolts. The sample enters the tube along with inert drying gas. The tube is heated to help evaporate solvent, but unlike electron impact used in GC-MS, the ionizer is not under vacuum. When a droplet of the sample reaches the nozzle, it becomes highly charged. The size of the droplet is decreased owing to evaporation. This causes the charge density to become excessive, and the droplets break apart. The tiny charged droplets repel each other and break apart again, forming a plume. These particles are drawn into the mass filter by "ion optics" (a system of repeller plates, counter electrode, and magnets). ESI does not result in extensive fragmentation, producing mostly the parent or "molecular" ion, a process called soft ionization.

Which of the following hormones is often decreased by approximately 25% in the serum of pregnant women who have a fetus with Down syndrome? A. Estriol (E3) B. Human chorionic gonadotropin (hCG) C. Progesterone D. Estradiol (E2)

A. Estriol (E3) A E3 is produced in the placenta and fetal liver from dehydroepiandrosterone derived from the mother and fetal liver. E3 is the major estrogen produced during pregnancy, and levels rise throughout gestation. Serum free E3 is often lower than expected for the gestational age in a pregnancy associated with Down syndrome. The combination of low serum free estriol, low-α fetoprotein, high hCG, and highinhibin A is used as a screening test to detect Down syndrome. When one of the four markers is abnormal, amniocentesis should be performed for the diagnosis of Down syndrome by karyotyping or FISH. The four markers have a combined sensitivity (detection rate) of approximately 75%.

Which of the following is the mechanism causing Cushing's disease? A. Excess secretion of pituitary ACTH B. Adrenal adenoma C. Treatment with corticosteroids D. Ectopic ACTH production by tumors

A. Excess secretion of pituitary ACTH A Cushing's disease refers to adrenal hyperplasia resulting from misregulation of the hypothalamic-pituitary axis. It is usually caused by small pituitary adenomas. Cushing's syndrome may be caused by Cushing's disease, adrenal adenoma or carcinoma, ectopic ACTH-producing tumors, or excessive corticosteroid administration. The cause of Cushing's syndrome can be differentiated using the ACTH and dexamethasone suppression tests

What measurement in addition to true negatives and prevalence is required to calculate the predictive value of a negative test result (PV-)? A. False negatives B. Variance C. True positives D. False positives

A. False negatives A The PV- is defined as the probability that a person with a negative test result is free of disease. A high PV- is a characteristic of a good screening test. The predictive value of a negative test is calculated by multiplying the true negatives by 100, then dividing by the sum of the true negatives and false negatives. %PV- = TN × 100 / (TN + FN)

Which of the following conditions is classified as a renal-type aminoaciduria? A. Fanconi syndrome B. Wilson's disease C. Hepatitis D. Homocystinuria

A. Fanconi syndrome A Fanconi syndrome is an inherited disorder characterized by anemia, mental retardation, rickets, and aminoaciduria. Because the aminoaciduria results from a defect in the renal tubule, it is classified as a (secondary-inherited) renal-type aminoaciduria. Wilson's disease (inherited ceruloplasmin deficiency) causes hepatic failure. It is classified as a secondary-inherited overflow-type aminoaciduria because the aminoaciduria results from urea cycle failure. Hepatitis is classified as a secondary-acquired overflow-type aminoaciduria. Homocystinuria is a primary-inherited overflow-type aminoaciduria, and is caused by a deficiency of cystathionine synthase.

Which of the following statements is true regarding reverse T3 (rT3)? A. Formed in the blood by degradation of T4 B. Physiologically active, but less than T3 C. Decreased in euthyroid sick syndrome D. Interferes with the measurement of serum T3

A. Formed in the blood by degradation of T4 A Reverse T3 is formed from the deiodination of T4 in the blood. It is an inactive isomer of T3, (3,3´,5´-triiodothyronine). Reverse T3 is increased in acute and chronic illness and is used to identify patients with euthyroid sick syndrome.

Which of the following statements about carbohydrate intolerance is true? A. Galactosemia results from deficiency of galactose-1-phosphate (galactose-1-PO4) uridine diphosphate transferase B. Galactosemia results in a positive glucose oxidase test for glucose in urine C. Urinary galactose is seen in both galactosemia and lactase deficiency D. A galactose tolerance test is used to confirm a diagnosis of galactosemia

A. Galactosemia results from deficiency of galactose-1-phosphate (galactose-1-PO4) uridine diphosphate transferase A Galactose is metabolized to galactose-1-PO4 by the action of galactokinase. Galactose-1-PO4 uridine diphosphate (UDP) transferase converts galactose-1-PO4 to glucose. Deficiency of either enzyme causes elevated blood and urine galactose. Lactase deficiency results in the presence of urinary lactose because it is not broken down to glucose and galactose. Tests for reducing sugars employing copper sulfate are used to screen for galactose, lactose, and fructose in urine. Nonglucose-reducing sugars are not detected by the glucose oxidase reaction. A positive test is followed by TLC to identify the sugar, and demonstration of the enzyme deficiency in RBCs. The galactose tolerance test is used (rarely) to evaluate the extent of liver failure since the liver is the site of galactose metabolism.

Zollinger-Ellison (Z-E) syndrome is characterized by great (e.g., 20-fold) elevation of: A. Gastrin B. Cholecystokinin C. Pepsin D. Glucagon

A. Gastrin A Z-E syndrome is caused by a pancreatic or intestinal tumor secreting gastrin (gastrinoma), and results in greatly increased gastric acid production. A serum gastrin level 10-fold greater than the URL in a person with hyperacidity and stomach or duodenal ulcers is diagnostic. Confirmation of gastric hyperacidity is demonstrated using the basal acid output (BAO) test

Identify the enzyme deficiency responsible for type 1 glycogen storage disease (von Gierke's disease). A. Glucose-6-phosphatase B. Glycogen phosphorylase C. Glycogen synthetase D. β-Glucosidase

A. Glucose-6-phosphatase A Type 1 glycogen storage disease (von Gierke's disease) is an autosomal recessive deficiency of glucose-6 phosphatase. Glycogen accumulates in tissues, causing hypoglycemia, ketosis, and fatty liver. There are seven types of glycogen storage disease, designated type 1 through type 7, involving deficiency of an enzyme that acts on glycogen. Types 1, 4, and 6 cause deficient glycogen breakdown in the liver. Types 2, 5, and 7 involve skeletal muscle and are less severe. Type 3 usually involves both liver and muscle, although an uncommon subtype (3B) involves only the liver

Select the coupling enzyme used in the hexokinase method for glucose. A. Glucose-6-phosphate dehydrogenase B. Peroxidase C. Glucose dehydrogenase D. Glucose-6-phosphatase

A. Glucose-6-phosphate dehydrogenase A The hexokinase reference method uses a protein-free filtrate prepared with barium hydroxide (BaOH) and zinc sulfate (ZnSO4). Hexokinase catalyzes the phosphorylation of glucose in the filtrate using ATP as the phosphate donor. Glucose-6-phosphate (glucose-6-PO4) is oxidized to 6-phosphogluconate and NAD+ is reduced to NADH using G-6-PD. The increase in absorbance at 340 nm is proportional to glucose concentration. Although hexokinase will phosphorylate some other hexoses including mannose, fructose, and glucosamine, the coupling reaction is entirely specific for glucose-6-PO4 eliminating interference from other sugars.

Which substrate concentration is needed to achieve zero-order conditions? A. Greater than 99 × Km B. [S] = Km C. Less than 10 × Km D. [S] = 0

A. Greater than 99 × Km A A zero-order reaction rate is independent of substrate concentration because there is sufficient substrate to saturate the enzyme. V = Vmax × [S]/Km + [S] where V = velocity, Vmax = maximum velocity, [S] = substrate concentration, and Km = substrate concentration required to give 1/2 Vmax. If [S] >>> Km, then the Km can be ignored. V = Vmax × S]/[S] = Vmax × [S]° or velocity approaches maximum and is independent of substrate concentration

In addition to velocity, what variable is also needed to calculate the relative centrifugal force (g force) of a centrifuge? A. Head radius B. Angular velocity coefficient C. Diameter of the centrifuge tube D. Ambient temperature in degrees Centigrade

A. Head radius A The relative centrifugal force (number times the force of gravity) is proportional to the square of the rotor speed in revolutions per minute and the radius in centimeters of the head (distance from the shaft to the end of the tube). RCF = s2 x r x 1.118 x 10-5 where s is the speed in RPM, r is the radius in cM, and 1.118 x 10-5 is a conversion constant.

In addition to sodium bicarbonate, what other substance contributes most to the amount of base in the blood? A. Hemoglobin concentration B. Dissolved O2 concentration C. Inorganic phosphorus D. Organic phosphate

A. Hemoglobin concentration A The primary blood buffer bases preventing acidosis in order of concentration are bicarbonate, deoxyhemoglobin, albumin, and monohydrogen phosphate. At physiological pH, there is significantly more H2PO4 -1 than HPO4 -2, and phosphate is a more efficient buffer system at preventing alkalosis than acidosis. Since all of the blood buffer systems are in equilibrium, the pH can be calculated accurately from the concentration of bicarbonate and dissolved CO2 using the Henderson-Hasselbalch equation.

Which of the statements below regarding the methods of Henry for AST and ALT is correct? A. Hemolysis will cause positive interference in both AST and ALT assays B. Loss of activity occurs if samples are frozen at -20°C C. The absorbance at the start of the reaction should not exceed 1.0 A D. Reaction rates are unaffected by addition of P-5´-P to the substrate

A. Hemolysis will cause positive interference in both AST and ALT assays A RBCs are rich in AST and to a lesser extent in ALT. Hemolysis causes positive interference in both assays, although the effect on AST is greater. Samples are stable for up to 24 hours at room temperature and up to 3 days at 4°C, and should be frozen if kept longer. The starting absorbance should be at least 1.5 A for both assays. Substrates with lower concentrations of NADH are subject to NADH depletion during the lag phase due to side reactions or high transaminase activity. When P-5'-P is added, a significant increase in activity sometimes occurs because some of the enzyme in the serum is in the inactive apoenzyme form

Which of the following would cause an increase in only the unconjugated bilirubin? A. Hemolytic anemia B. Obstructive jaundice C. Hepatitis D. Hepatic cirrhosis

A. Hemolytic anemia A Conjugated bilirubin increases as a result of obstructive processes within the liver or biliary system or from failure of the enterohepatic circulation. Hemolytic anemia (prehepatic jaundice) presents a greater bilirubin load to a normal liver, resulting in increased bilirubin excretion. When the rate of bilirubin formation exceeds the rate of excretion, the unconjugated bilirubin rises.

A patient with hemolytic-uremic syndrome associated with septicemia has a haptoglobin level that is normal, although the plasma free hemoglobin is elevated and hemoglobinuria is present. Which test would be more appropriate than haptoglobin to measure this patient's hemolytic episode? A. Hemopexin B. Alpha-1 antitrypsin C. C-reactive protein D. Transferrin

A. Hemopexin A Hemopexin is a small β globulin that binds to free heme. Haptoglobin is an α-2 globulin that binds to free hemoglobin and disappears from the serum when intravascular hemolysis produces more than 3 grams of free plasma hemoglobin. However, haptoglobin is an acute phase protein, and hepatic production and release are increased in response to acute infections. The normal serum haptoglobin is most likely the result of increased synthesis and would not accurately estimate the hemolytic episode in this patient

Which type of cancer is associated with the highest level of AFP? A. Hepatoma B. Ovarian cancer C. Testicular cancer D. Breast cancer

A. Hepatoma A AFP is increased in all persons with yolk sac tumors and over 80% of those with hepatoma. Levels above 1000 ng/mL are diagnostic of hepatoma. Ectopic AFP-secreting tumors are produced by ovarian, testicular, breast, GI, and bladder cancers, and these sources should be considered when 10-fold or higher elevations are seen in the absence of abnormal liver function. AFP is used along with hCG to increase the diagnostic sensitivity of nonseminoma testicular tumors and to stage the disease. Approximately 42% of persons with nonseminoma testicular cancer are positive for hCG but over 70% are positive for hCG or AFP

In the Oliver-Rosalki method, the reverse reaction is used to measure CK activity. The enzyme(s) used in the coupling reactions is (are): A. Hexokinase and G-6-PD B. Pyruvate kinase and LD C. Luciferase D. Adenylate kinase

A. Hexokinase and G-6-PD A The Oliver-Rosalki method for CK is based upon the formation of ATP from creatine phosphate. Hexokinase catalyzes the phosphorylation of glucose by ATP. This produces glucose-6-PO4 and adenosine diphosphate (ADP). The glucose-6-PO4 is oxidized to 6-phosphogluconate as NADP+ is reduced to NADPH. ATP + glucose -Hexokinase-> ADP + glucose-6-PO4 glucose-6-PO4 + NADP+ -G-6-PD-> 6-phosphogluconate + NADPH + H+

Which Hgb separates from Hgb S on citrate (acid) agar, but not agarose or cellulose acetate? A. Hgb DPunjab B. Hgb E C. Hgb CHarlem (Georgetown) D. Hgb OArab

A. Hgb DPunjab A Hgbs OArab, E, and CHarlem migrate to the same position as Hgbs A2 and C on agarose × or cellulose acetate) at pH 8.6. Hgb DPunjab migrates to the same position as Hgb S on agarose, but moves with Hgb A on citrate agar. Agarose is a purified form of agar; it lacks the sulfated pectins required to separate Hgbs DPunjab and G from Hgb S, and Hgbs E, CHarlem, and OArab from Hgb C. Hgb CHarlem is a sickling Hgb and it migrates to the same position as Hgb S on citrate (acid) agar.

Which of the following statements describes a nonkinetic enzyme assay? A. Initial absorbance is measured followed by a second reading after 5 minutes B. Absorbance is measured at 10-second intervals for 100 seconds C. Absorbance is monitored continuously for 1 minute using a chart recorder D. Reflectance is measured from a xenon source lamp pulsing at 60 Hz

A. Initial absorbance is measured followed by a second reading after 5 minutes A A kinetic assay uses several evenly spaced absorbance measurements to calculate the change in absorbance per unit time. A constant change in absorbance per unit of time occurs only when the rate of the reaction is zero order (independent of substrate concentration). Enzyme activity is proportional to rate only under zero-order conditions.

A neonatal bilirubin assay performed at the nursery by bichromatic direct spectrophotometry is 4.0 mg/dL. Four hours later, a second sample assayed for total bilirubin by the Jendrassik-Grof method gives a result of 3.0 mg/dL. Both samples are reported to be hemolyzed. What is the most likely explanation of these results? A. Hgb interference in the second assay B. δ-Bilirubin contributing to the result of the first assay C. Falsely high results from the first assay caused by direct bilirubin D. Physiological variation owing to premature hepatic microsomal enzymes

A. Hgb interference in the second assay A The Jendrassik-Grof method is based upon a diazo reaction that may be suppressed by Hgb. Because serum blanking and measurement at 600 nm correct for positive interference from Hgb, the results may be falsely low when significant hemolysis is present. Direct spectrophometric bilirubin methods employing bichromatic optics correct for the presence of Hgb. These are often called "neonatal bilirubin" tests. A commonly used approach is to measure absorbance at 454 nm and 540 nm. The absorbance contributed by Hgb at 540 nm is equal to the absorbance contributed by Hgb at 454 nm. Therefore, the absorbance difference will correct for free Hgb. Neonatal samples contain little or no direct δ-bilirubin. They also lack carotene pigments that could interfere with the direct spectrophotometric measurement of bilirubin.

Quantitative determination of Hgb A2 and Hgb F are best performed by: A. High-performance liquid chromatography B. Alkali denaturation C. Electrophoresis D. Direct bichromatic spectrophotometry

A. High-performance liquid chromatography A. Hgb A2 and Hgb F are often quantitated to diagnose persons with thalassemia. The method of choice is HPLC using cation exchange chromatography. Hemoglobins are eluted from the column in order of increasing positive charge using a sodium phosphate buffer to produce a gradient of increasing ionic strength. Hemoglobin F elutes from the column earlier than Hgb A2 because it is less positively charged.

Which of the following is one advantage of high-resolution (HR) agarose electrophoresis over lower-current electrophoresis? A. High-resolution procedures detect monoclonal and oligoclonal bands at a lower concentration B. A smaller sample volume is used C. Results are obtained more rapidly D. Densitometric scanning of HR gels is more accurate

A. High-resolution procedures detect monoclonal and oligoclonal bands at a lower concentration A HR agarose procedures use higher current and a cooling device to resolve 12 or more bands. Advantages include phenotyping of α1-antitrypsin (detection of Z and S variants), detection of β2 microglobulin in urine indicating tubular proteinuria (often associated with drug-induced nephrosis), and greater sensitivity detecting monoclonal gammopathies, immune complexes, and oligoclonal bands in CSF associated with multiple sclerosis. Its disadvantage is that densitometric scans of HR gels usually underestimate albumin

Which definition best describes the catalytic activity of amylase? A. Hydrolyzes second α 1-4 glycosidic linkages of starch, glycogen, and other polyglucans B. Hydrolyzes all polyglucans completely to produce glucose C. Oxidatively degrades polysaccharides containing glucose D. Splits polysaccharides and disaccharides by addition of water

A. Hydrolyzes second α 1-4 glycosidic linkages of starch, glycogen, and other polyglucans A Amylase in humans is a hydrolase that splits the second α 1-4 glycosidic bonds of polyglucans forming maltose. There are two major types of amylase: P-type derived from the pancreas and S-type derived from the salivary glands. These can be differentiated by both electrophoresis and immunoassay. In healthy persons, the principal form in plasma is the salivary isoenzyme. There are several genetic variants of the salivary isoenzyme, which in part accounts for the broad reference range.

Which of the following is the primary mechanism causing respiratory alkalosis? A. Hyperventilation B. Deficient alveolar diffusion C. Deficient pulmonary perfusion D. Parasympathetic inhibition

A. Hyperventilation A Hyperventilation via stimulation of the respiratory center (or induced by a respirator) is the mechanism of respiratory alkalosis. Causes include low PO2, anxiety, fever, and drugs that stimulate the respiratory center. Acute respiratory alkalosis is often uncompensated because renal compensation is not rapid. Uncompensated respiratory alkalosis is characterized by an elevated pH and a low PCO2 with normal bicarbonate.

Which of the following is the primary mechanism of compensation for metabolic acidosis? A. Hyperventilation B. Release of epinephrine C. Aldosterone release D. Bicarbonate excretion

A. Hyperventilation A In metabolic acidosis, the respiratory center is stimulated by chemoreceptors in the carotid sinus, causing hyperventilation. This results in increased release of CO2. Respiratory compensation begins almost immediately unless blocked by pulmonary disease or respiratory therapy. Hyperventilation can bring the PCO2 down to approximately 10-15 mm Hg.

Which of the following conditions is most consistently associated with secondary hypercholesterolemia? A. Hypothyroidism B. Pancreatitis C. Oral contraceptive therapy D. Diabetes mellitus

A. Hypothyroidism A The conditions listed are very commonly encountered causes of secondary hyperlipoproteinemia. Oral contraceptives, pregnancy, and estrogens may cause secondary hypertriglyceridemia owing to increased VLDL and endogenous triglycerides. Hypothyroidism and obstructive hepatobiliary diseases are usually associated with secondary hypercholesterolemia owing to high LDL. Diabetesmellitus and chronic pancreatitis may produce hypertriglyceridemia, chylomicronemia, or mixed hyperlipidemia

Which set of results is most likely in an adult male with primary testicular failure? A. Increased LH, FSH, and decreased testosterone B. Decreased LH, FSH, and testosterone C. Decreased testosterone, androstenedione, and FSH D. Increased androstenedione, decreased testosterone, and normal FSH

A. Increased LH, FSH, and decreased testosterone A Primary testicular failure produces a picture that is hypergonadotropic. The LH and FSH are increased because the pituitary gland is normal and responds to decreased free testosterone. Androstenedione is an adrenal androgen and is unaffected. In testicular failure secondary to pituitary deficiency (hypogonadotropic testicular failure), the LH, FSH, and testosterone are low.

In the Oliver-Rosalki method for CK, adenosine monophosphate (AMP) is added to the substrate in order to: A. Inhibit adenylate kinase B. Block the oxidation of glutathione C. Increase the amount of ADP that is available D. Block the action of diadenosine pentaphosphate

A. Inhibit adenylate kinase A Positive interference in the Oliver-Rosalki method can occur when adenylate kinase is present in the serum from hemolysis or damaged tissue. Adenylate kinase hydrolyzes ADP, forming AMP and ATP (2 ADP AK AMP + ATP). This reaction is inhibited by adding AMP and diadenosine pentaphosphate (Ap5A) to the substrate.

Which of the following is associated with low serum iron and high total iron-binding capacity (TIBC)? A. Iron deficiency anemia B. Hepatitis C. Nephrosis D. Noniron deficiency anemias

A. Iron deficiency anemia A Iron-deficiency anemia is the principal cause of low serum iron and high TIBC because it promotes increased transferrin. Pregnancy without iron supplementation depletes maternal iron stores and also results in low serum iron and high TIBC. Iron-supplemented pregnancy and use of contraceptives increase both iron and TIBC. Nephrosis causes low iron and TIBC due to loss of both iron and transferrin by the kidneys. Hepatitis causes increased release of storage iron, resulting in high levels of iron and transferrin. Noniron deficiency anemias may cause high iron and usually show low TIBC and normal or high ferritin.

Lipoprotein (a), or Lp(a), is significant when elevated in serum because it: A. Is an independent risk factor for atherosclerosis B. Blocks the clearance of VLDLs C. Displaces apo-AI from HDLs D. Is linked closely to a gene for obesity

A. Is an independent risk factor for atherosclerosis A Lp(a) is a complex of apo-B100 and protein (a) formed by a disulfide bridge. The complex is structurally similar to plasminogen and is thought to promote coronary heart disease by interfering with the normal fibrinolytic process. Lp(a) is measured by immunoassay; however, the measurement will vary depending on the type of antibodies used and their epitope specificity.

Which statement about multiple endocrine neoplasia (MEN) is true? A. It is associated with hyperplasia or neoplasia of at least two endocrine organs B. Insulinoma is always present when the pituitary is involved C. It is inherited as an autosomal recessive disorder D. Plasma hormone levels from affected organs are elevated at least 10-fold

A. It is associated with hyperplasia or neoplasia of at least two endocrine organs A Multiple-endocrine neoplasia syndrome is inherited as an autosomal dominant disease involving excess production of hormones from several endocrine glands. MEN I results from adenomas (usually benign) of at least two glands, including the pituitary, adrenal cortex, parathyroid, and pancreas. The parathyroid gland is the organ most commonly involved, and in those patients an elevated Cai is an early sign. The pancreas is the next most frequently involved organ, but the hormone most commonly oversecreted is gastrin (not insulin). MEN II is characterized by pheochromocytoma and thyroid carcinoma. MEN II-B is a variant of MEN II showing the addition of neurofibroma.

Which of the following statements regarding urobilinogen is true? A. It is formed in the intestines by bacterial reduction of bilirubin B. It consists of a single water-soluble bile pigment C. It is measured by its reaction with p-aminosalicylate D. In hemolytic anemia, it is decreased in urine and feces

A. It is formed in the intestines by bacterial reduction of bilirubin A Urobilinogen is a collective term given to the reduction products of bilirubin formed by the action of enteric bacteria. Urobilinogen excretion is increased in extravascular hemolytic anemias and decreased in obstructive jaundice (cholestatic disease). Urobilinogen is measured using Ehrlich's reagent, an acid solution of p-dimethylaminobenzaldehyde.

Which of the following is a characteristic of conjugated bilirubin? A. It is water soluble B. It reacts more slowly than unconjugated bilirubin C. It is more stable than unconjugated bilirubin D. It has the same absorbance properties as unconjugated bilirubin

A. It is water soluble A Conjugated bilirubin refers to bilirubin mono- and diglucuronides. Conjugated bilirubin reacts almost immediately with the aqueous diazo reagent without need for a nonpolar solvent. Historically, conjugated bilirubin has been used synonymously with direct-reacting bilirubin, although the latter includes the δ-bilirubin fraction when measured by the Jendrassik-Grof method. Conjugated bilirubin is excreted in both bile and urine. It is easily photo-oxidized and has very limited stability. For this reason, bilirubin standards are usually prepared from unconjugated bilirubin stabilized by the addition of alkali and albumin.

Orders for uric acid are legitimate stat requests because: A. Levels above 10 mg/dL cause urinary tract calculi B. Uric acid is hepatotoxic C. High levels induce aplastic anemia D. High levels cause joint pain

A. Levels above 10 mg/dL cause urinary tract calculi A Uric acid calculi form quickly when the serum uric acid level reaches 10 mg/dL. They are translucent compact stones that often lodge in the ureters, causing postrenal failure.

Which statement about glucose in cerebrospinal fluid (CSF) is correct? A. Levels below 40 mg/dL occur in septic meningitis, cancer, and multiple sclerosis B. CSF glucose is normally the same as the plasma glucose level C. Hyperglycorrhachia is caused by dehydration D. In some clinical conditions, the CSF glucose can be greater than the plasma glucose

A. Levels below 40 mg/dL occur in septic meningitis, cancer, and multiple sclerosis A High glucose in CSF is a reflection of hyperglycemia and not central nervous system disease. The CSF glucose is usually 50%-65% of the plasma glucose. Low levels are significant and are most often associated with bacterial or fungal meningitis, malignancy in the central nervous system, and some cases of subarachnoid hemorrhage, rheumatoid arthritis, and multiple sclerosis.

Which statement regarding measurement of Hgb A1c is true? A. Levels do not need to be done fasting B. Both the labile and stable Hgb A1c fractions are measured C. Samples should be measured within 2 hours of collection D. The assay must be done by chromatography

A. Levels do not need to be done fasting A Since Hgb A1C represents the average blood glucose 2-3 months prior to blood collection, the dietary status of the patient on the day of the test has no effect upon the results. Refrigerated whole-blood samples are stable for up to 1 week. Hgb A1C is assayed by cation exchange high-performance liquid chromatography or immunoassay (immunoturbidimetric inhibition) because both methods are specific for stable Hgb A1C, and do not demonstrate errors caused by abnormal hemoglobins, temperature of reagents, or fractions other than A1c.

Which of the following plots is best for detecting all types of QC errors? A. Levy-Jennings B. Tonks-Youden C. Cusum D. Linear regression

A. Levy-Jennings A The Levy-Jennings plot is a graph of all QC results with concentration plotted on the y axis and run number on the x axis. The mean is at the center of the y axis, and concentrations corresponding to -2 and +2s are highlighted. Results are evaluated for multirule violations across both levels and runs. Corrective action for shifts and trends can be taken before QC rules are broken.

Which statement about the clinical utility of plasma or serum lipase is true? A. Lipase is not increased in mumps, malignancy, or ectopic pregnancy B. Lipase is not increased as dramatically as amylase in acute pancreatitis C. Increased plasma or serum lipase is specific for pancreatitis D. Lipase levels are elevated in both acute and chronic pancreatitis

A. Lipase is not increased in mumps, malignancy, or ectopic pregnancy A Lipase elevation is of greater magnitude (2-50 × N) and duration than amylase in acute pancreatitis. When the lipase method is optimized by inclusion of colipase and bile salts, the test is more sensitive and specific than serum amylase for detection of acute pancreatitis. However, lipase is also increased in peptic ulcers, renal insufficiency, and intestinal obstruction. Lipase levels are often low in chronic pancreatitis, and are low in cystic fibrosis.

All of the following compounds contribute to the osmolality of plasma except: A. Lipids B. Creatinine C. Drug metabolites D. Glucose

A. Lipids A Osmolality is the concentration (in moles) of dissolved solute per kilogram solvent. Proteins and lipids are not in solution, and do not contribute to osmolality. The nonionized solutes such as glucose and urea contribute 1 osmole per mole per kilogram water, whereas dissociated salts contribute 1 osmole per mole of each dissociated ion or radical.

Which of the following laboratory results is consistent with primary hypoparathyroidism? A. Low calcium; high inorganic phosphorus Pi B. Low calcium; low Pi C. High calcium; high Pi D. High calcium; low Pi

A. Low calcium; high inorganic phosphorus Pi A Parathyroid hormone deficiency causes reduced resorption of calcium from bone, increased renal excretion of calcium, and decreased renal excretion of phosphorus. It is distinguished from other causes of hypocalcemia by Cai , which is reduced only by primary hypoparathyroidism and alkalosis.

The term reverse phase is used in HPLC to indicate that the mobile phase is: A. More polar than the stationary phase B. Liquid and the stationary phase is solid C. Organic and the stationary phase is aqueous D. A stronger solvent than the stationary phase

A. More polar than the stationary phase A In reverse-phase HPLC, the separation takes place using a nonpolar sorbent (stationary phase) such as octadecylsilane (C18). Solutes that are nonpolar are retained longer than polar solutes. Most clinical separations of drugs, hormones, and metabolites use reverse phase because aqueous mobile phases are far less toxic and flammable.

High serum total protein but low albumin is usually seen in: A. Multiple myeloma B. Hepatic cirrhosis C. Glomerulonephritis D. Nephrotic syndrome

A. Multiple myeloma A In multiple myeloma, synthesis of large quantities of monoclonal immunoglobulin by plasma cells often results in decreased synthesis of albumin. In glomerulonephritis and nephrotic syndrome, both total protein and albumin are low owing to loss of proteins through the glomeruli. In hepatic cirrhosis, decreased hepatic production of protein results in low total protein and albumin.

Which of the following conditions is associated with a high level of S-type amylase? A. Mumps B. Intestinal obstruction C. Alcoholic liver disease D. Peptic ulcers

A. Mumps A Both salivary and pancreatic amylases designated S-type and P-type, respectively, are present in normal serum. High amylase occurs in mumps, ectopic pregnancy, biliary obstruction, peptic ulcers, alcoholism, malignancies, and other nonpancreatic diseases. Isoenzymes can be separated by electrophoresis (S-type is faster than P-type), but more commonly immunoinhibition of S-type amylase is used to rule out mumps, malignancy, and ectopic pregnancy, which give rise to high S-type amylase

Which of the following conditions is usually associated with an acute inflammatory pattern? A. Myocardial infarction (MI) B. Malignancy C. Rheumatoid arthritis D. Hepatitis

A. Myocardial infarction (MI) A MI produces a pattern of acute inflammation usually associated with tissue injury. This pattern results from production of acute phase proteins including α1-antitrypsin, α1 antichymotrypsin, and haptoglobin. It is also seen in early infection, pregnancy, and early nephritis. Malignancy, rheumatoid arthritis, and hepatitis are associated with a chronic inflammatory pattern. This differs from the acute pattern by the addition of a polyclonal gammopathy

Which of the following statements about enzymatic reactions is true? A. NADH has absorbance maximas at 340 and 366 nm B. Enzyme concentration must be in excess to achieve zero-order kinetics C. Rate is proportional to substrate concentration in a zero-order reaction D. Accumulation of the product increases the reaction rate

A. NADH has absorbance maximas at 340 and 366 nm A Most enzymes are measured by monitoring the rate of absorbance change at 340 nm as NADH is produced or consumed. This rate will be proportional to enzyme activity when substrate is in excess. When the enzyme is present in excess, the initial reaction rate will be proportional to substrate concentration. This condition, called a first-order reaction, is needed when the enzyme is used as a reagent to measure a specific analyte.

When measuring trace metals in blood other than lead, what type of tube should be used? A. Navy blue top B. Green top C. Purple top D. Red top

A. Navy blue top A In order to avoid trace contamination by metals present in the stopper lubricants, a tube with a navy blue top is used for measuring trace metals. These tubes are validated for most but not all trace metals. Such tubes are available with or without EDTA for serum or whole-blood analysis, respectively. Tubes with tan stoppers containing EDTA are used for lead assay because they are certified to contain no more than 0.25 μg/dL lead. In addition, type 1 purity water (10 Mohm, 10 or less CFU/mL) and analytical reagent grade chemicals are always used to prepare reagents such as matrix modifiers. Although most trace metals are measured in whole blood or serum, arsenic is usually measured in urine because it is metabolized and excreted within hours of ingestion.

At pH 8.6, proteins are _________ charged and migrate toward the _________. A. Negatively, anode B. Positively, cathode C. Positively, anode D. Negatively, cathode

A. Negatively, anode A Proteins are amphoteric owing to ionization of acidic and basic side chains of amino acids. When the pH of the solution equals the isoelectric point (pI), the protein will have no net charge and is insoluble. When the pH of the solution is above the pI, the protein will have a net negative charge. Anions migrate toward the anode (positive electrode).

For a drug that follows first-order pharmacokinetics, adjustment of dosage to achieve the desired blood level can be made using which formula? A. New dose = current dose /concentration at steady state × (desired concentration) B. New dose = current dose/desired concentration × (concentration at steady state) C. New dose = concentration at steady state/ desired concentration × (half-life) D. New dose = concentration at steady state/ current dose × (desired concentration)

A. New dose = current dose /concentration at steady state × (desired concentration) A Most drugs follow first-order pharmacokinetics, meaning the clearance of drug is linearly related to the drug dose. The dose of such drugs can be adjusted by multiplying the ratio of the current dose to blood concentration by the desired drug concentration, provided the blood concentration is measured at steady state.

Which of the following formulas for O2 content is correct? A. O2 content = %O2 saturation/100 × Hgb g/dL × 1.39 mL/g + (0.0031 × PO2) B. O2 content = PO2 × 0.0306 mmol/L/mm C. O2 content = O2 saturation × Hgb g/dL × 0.003 mL/g D. O2 content = O2 capacity × 0.003 mL/g

A. O2 content = %O2 saturation/100 × Hgb g/dL × 1.39 mL/g + (0.0031 × PO2) A Oxygen content is the sum of O2 bound to Hgb and O2 dissolved in the plasma. It is dependent upon the Hgb concentration and the percentage of Hgb bound to O2 (O2 saturation). Each gram of Hgb binds 1.39 mL of O2. The dissolved O2 is determined from the solubility coefficient of O2 (0.0031 mL per dL/mm Hg) and the PO2. O2 content = % Sat/100 × Hgb in g/dL × 1.39 mL/g + (0.0031 × PO2).

Which of the following stains is used for lipoprotein electrophoresis? A. Oil Red O B. Coomassie Brilliant Blue C. Amido Black D. Ponceau S

A. Oil Red O A Oil Red O and Sudan Black B stain neutral fats and are used to stain lipoproteins as well as fat in urine or stool. The other stains are used for proteins. Coomassie Brilliant Blue is more sensitive than Ponceau S or Amido Black, and all three stains have slightly greater affinity for albumin than globulins. In addition, silver nitrate may be used to stain CSF proteins because it has greater sensitivity than the other stains.

Which of the following statements regarding paraproteins is true? A. Oligoclonal banding is seen in the CSF of greater than 90% of multiple sclerosis cases B. The Bence-Jones protein heat test is confirmatory for monoclonal light chains C. Light chains found in urine are always derived from monoclonal protein D. The IgA band is usually cathodal to the IgG precipitin band

A. Oligoclonal banding is seen in the CSF of greater than 90% of multiple sclerosis cases A The α heavy chain is more acidic than γ or μ chains, giving IgA a greater net negative charge at alkaline pH. The IgA precipitin band is anodal to the IgG or IgM band. In hepatic cirrhosis, the β-γ bridging observed on serum protein electrophoresis results from increased IgA. Light chains in the form of Fab fragments are often found in increased amounts in the urine of patients with polyclonal gammopathies, especially from patients with an autoimmune disease. These can cause positive Bence-Jones test and will produce a polyclonal (spread-out) appearance on IFE gels.

Select the elimination model that best describes most oral drugs. A. One compartment, linear first-order elimination B. Michaelis-Menton or concentration-dependent elimination C. Two compartment with a biphasic elimination curve D. Logarithmic elimination

A. One compartment, linear first-order elimination A Most drugs given orally distribute uniformly through the tissues reaching rapid equilibrium, so both blood and tissues can be viewed as a single compartment. Elimination according to Michaelis-Menton kinetics is nonlinear because at high concentrations, the hepatic enzyme system becomes saturated, reducing the elimination efficiency.

Which of the following is a marker for bone formation? A. Osteocalcin B. Tartrate resistant acid phosphatase (TRAP) C. Urinary pyridinoline and deoxypyridinoline D. Urinary C-telopeptide and N-telopeptide crosslinks (CTx and NTx

A. Osteocalcin A Biochemical markers for osteoporosis are classified as either markers for bone formation or resorption. Osteocalcin is a proteinhormone that stimulates osteoblasts and increases bone mineralization. Pyridinoline is formed when hydroxylysine groups on adjacent fibrils are joined together, and deoxypyridinoline when hydroxylysine and lysine groups are joined. These form crosslinks between the C and N terminal ends of one fibril (which are nonhelical) and the helical portion of an adjacent fibril. The resulting products are called C- and N-telopeptide crosslinks of type 1 collagen. Osteoclasts cause cleavage of these bonds, resulting in loss of both telopeptides—deoxypyridinoline and pyridinoline—in the urine. TRAP is an enzyme (produced by osteoclasts) that hydrolyzes phosphate in the hydroxyapatite matrix of the bone.

The reference method for lipase uses olive oil as the substrate because: A. Other esterases can hydrolyze triglyceride and synthetic diglycerides B. The reaction product can be coupled to NADH generating reactions C. Synthetic substrates are less soluble than olive oil in aqueous reagents D. Triglyceride substrates cause product inhibition

A. Other esterases can hydrolyze triglyceride and synthetic diglycerides A Triglycerides may be hydrolyzed by nonspecific esterases in serum as well as lipase. Lipase acts only at an interface of oil and H2O and requires bile salts and colipase for activity. Colipase is a protein secreted by the pancreas.

Which is the most widely used screening test for Cushing's syndrome? A. Overnight low-dose dexamethasone suppression test B. Corticotropin-releasing hormone stimulation test C. Petrosal sinus sampling D. Metyrapone stimulation test

A. Overnight low-dose dexamethasone suppression test A Dexamethasone is a synthetic corticosteroid that exhibits 30-fold greater negative feedback on the hypothalamus than cortisol. When an oral dose of 1 mg of the drug is given to a patient at 11 p.m., the 8 a.m. serum total cortisol level should be below 5.0 μg/dL. Patients with Cushing's syndrome almost always exceed this cutoff. Therefore, a normal response to dexamethasone excludes Cushing's syndrome with a sensitivity of about 98%. CRH stimulation and petrosal sinus sampling are confirmatory tests for Cushing's disease, and are used when the high-dose dexamethasone suppression test is inconclusive. The metyrapone stimulation test measures the patient's ACTH reserve. Metyrapone blocks cortisol formation by inhibiting 11-β hydroxylase. This causes an increase in ACTH output in normals. A subnormal ACTH response is seen in persons with Addison's disease caused by pituitary failure

Creatinine is formed from the: A. Oxidation of creatine B. Oxidation of protein C. Deamination of dibasic amino acids D. Metabolism of purines

A. Oxidation of creatine A Creatinine is produced at a rate of approximately 2% daily from the oxidation of creatine mainly in skeletal muscle. Creatine can be converted to creatinine by addition of strong acid or alkali or by the enzyme creatine hydroxylase

Which of the following effects results from exposure of a normal arterial blood sample to room air? A. PO2 increased PCO2 decreased pH increased B. PO2 decreased PCO2 increased pH decreased C. PO2 increased PCO2 decreased pH decreased D. PO2 decreased PCO2 decreased pH decreased

A. PO2 increased PCO2 decreased pH increased A The PO2 of air at sea level (21% O2) is about 150 mm Hg. The PCO2 of air is only about 0.3 mm Hg. Consequently, blood releases CO2 gas and gains O2 when exposed to air. Loss of CO2 shifts the equilibrium of the bicarbonate buffer system to the right, decreasing hydrogen ion concentration and blood becomes more alkaline.

The D-xylose absorption test is used for the differential diagnosis of which two diseases? A. Pancreatic insufficiency from malabsorption B. Primary from secondary disorders of glycogen synthesis C. Type 1 and type 2 diabetes mellitus D. Generalized from specific carbohydrate intolerance

A. Pancreatic insufficiency from malabsorption A Xylose is a pentose that is absorbed without the help of pancreatic enzymes and is not metabolized. In normal adults, more than 25% of the dose is excreted into the urine after 5 hours. Low blood or urine levels are seen in malabsorption syndrome, sprue, Crohn's disease, and other intestinal disorders, but not pancreatitis.

Which condition produces the highest elevation of serum lactate dehydrogenase? A. Pernicious anemia B. Myocardial infarction C. Acute hepatitis D. Muscular dystrophy

A. Pernicious anemia A Serum LD levels are highest in pernicious anemia, reaching 10-50 times the upper reference limit (URL) as a result of intramedullary hemolysis. Moderate elevations (5-10 × URL) usually are seen in acute myocardial infarction, necrotic liver disease, and muscular dystrophy. Slight increases (2-3 × URL) are sometimes seen in obstructive liver disease

Which enzyme deficiency is responsible for phenylketonuria (PKU)? A. Phenylalanine hydroxylase B. Tyrosine transaminase C. p-Hydroxyphenylpyruvic acid oxidase D. Homogentisic acid oxidase

A. Phenylalanine hydroxylase A PKU is an overflow aminoaciduria resulting from the accumulation of phenylalanine. It is caused by a deficiency of phenylalanine hydroxylase, which converts phenylalanine to tyrosine. Excess phenylalanine accumulates in blood. This is transaminated, forming phenylpyruvic acid, which is excreted in the urine.

Which of the following statements is correct in assessing GH deficiency? A. Pituitary failure may involve one, several, or all adenohypophyseal hormones; but GH deficiency is usually found B. A normal random serum level of GH in a child under 6 years old rules out GH deficiency C. Administration of arginine, insulin, or glucagon will suppress GH release D. GH levels in the blood show little variation within a 24-hour period

A. Pituitary failure may involve one, several, or all adenohypophyseal hormones; but GH deficiency is usually found A Because GH is the most abundant pituitary hormone, it may be used as a screening test for pituitary failure in adults. Pituitary hormone deficiencies are rare and are evaluated by measuring those hormones associated with the specific type of target organ dysfunction. GH secretion peaks during sleep, and pulsed increases are seen following exercise and meals. In adults, a deficiency of GH can be ruled out by demonstrating normal or high levels on two successive tests. In children, there is extensive overlap between normal and low GH levels, and a stimulation (provocative) test is usually needed to establish a diagnosis of deficiency. Exercise is often used to stimulate GH release. If GH levels are greater than 6 μg/L after vigorous exercise, then deficiency is ruled out. In addition to exercise, drugs such as arginine, insulin, propranolol, and glucagon can be used to stimulate GH release. Deficiency is documented by registering a subnormal response to two stimulating agents.

The determination of the oxygen saturation of hemoglobin is best accomplished by: A. Polychromatic absorbance measurements of a whole-blood hemolysate B. Near infrared transcutaneous absorbance measurement C. Treatment of whole blood with alkaline dithionite prior to measuring absorbance D. Calculation using PO2 and total hemoglobin by direct spectrophotometry

A. Polychromatic absorbance measurements of a whole-blood hemolysate A Measurement of oxyhemoglobin, deoxyhemoglobin (reduced hemoglobin), carboxyhemoglobin, methemoglobin, and sulfhemoglobin can be accomplished by direct spectrophotometry at multiple wavelengths and analysis of the absorptivity coefficients of each pigment at various wavelengths. The O2 saturation is determined by dividing the fraction of oxyhemoglobin by the sum of all pigments. This eliminates much of the error that occurs in the other methods when the quantity of an abnormal hemoglobin pigment is increased.

One of two controls within a run is above +2s and the other control is below -2s from the mean. What do these results indicate? A. Poor precision has led to random error (RE) B. A systematic error (SE) is present C. Proportional error is present D. QC material is contaminated

A. Poor precision has led to random error (RE) A When control results deviate from the mean in opposite directions, the run is affected by RE, which results from imprecision. An analytical run is rejected when two controls within the same run have an algebraic difference in excess of 4s (R4s). The R4S rule is applied only to controls within a run (Level 1 - Level 2), never across runs or days.

A patient has an elevated serum T3 and free T4 and undetectable TSH. What is the most likely cause of these results? A. Primary hyperthyroidism B. Secondary hyperthyroidism C. Euthyroid with increased thyroxine-binding proteins D. Euthyroid sick syndrome

A. Primary hyperthyroidism A An undetectable TSH with increased T3 is caused by primary hyperthyroidism (suppression via high free thyroid hormone). In secondary hyperthyroidism, the TSH will be elevated in addition to at least the T3. Patients with an increased thyroxine-binding protein level will have an increase in total T3 but not free T4 or TSH. Patients with euthyroid sick syndrome usually have a low total T3 due to deficient conversion of T4 to T3, normal free T4, and a normal or slightly elevated TSH.

Enzymatic measurement of ammonia requires which of the following substrates and coenzymes? Substrate / Coenzyme A. α-Ketoglutarate / NADH B. Glutamate / NADH C. Glutamine / ATP D. Glutamine / NAD+

A. α-Ketoglutarate / NADH A Enzymatic assays of ammonia utilize glutamate dehydrogenase (GLD). This enzyme forms glutamate from α-ketoglutarate (2 oxoglutarate) and ammonia, resulting in oxidation of NADH. The rate of absorbance decrease at 340 nm is proportional to ammonia concentration when the reaction rate is maintained under first order conditions.

The serum TSH level is almost absent in: A. Primary hyperthyroidism B. Primary hypothyroidism C. Secondary hyperthyroidism D. Euthyroid sick syndrome

A. Primary hyperthyroidism A Low TSH and a high T3 (and usually T4) occur in primary hyperthyroidism, but may also occur in systemic nonthyroid illnesses where T4 has been converted to T3. A 2-fold increase in free hormone can produce a 100-fold decrease in TSH. In primary hyperthyroidism, the TSH will be within a range of 0-0.02 mU/mL, while in nonthyroid illnesses it will be 0.03 mU/mL or higher. A high TSH and low T4 occur in primary hypothyroidism but can also occur in an acutely ill patient without thyroid disease, the euthyroid sick syndrome. Secondary hyperthyroidism is caused by pituitary hyperfunction, resulting in increased serum TSH.

Which component is needed for a thermal cycler to amplify DNA? A. Programmable heating and cooling unit B. Vacuum chamber with zero head space C. Sealed airtight constant-temperature chamber D. Temperature-controlled ionization chamber

A. Programmable heating and cooling unit A The polymerase chain reaction for DNA amplification consists of three phases. Denaturation requires a temperature of 90°C 94°C and separates the double-stranded DNA. Annealing requires a temperature between 40°C-65°C and allows the primers to bind to the target base sequence. Extension requires a temperature of 72°C and allows the heat-stable polymerase to add complementary bases to the primer in the 5' to 3' direction. A cycle consists of each temperature stage for a specific number of minutes and most procedures require 30 or more cycles to generate a detectable quantity of target DNA. Rapid heating and cooling is usually achieved using a thermoelectric block that is cooled by forced air flow.

When pituitary adenoma is the cause of decreased estrogen production, an increase of which hormone is most frequently responsible? A. Prolactin B. FSH C. LH D. Thyroid-stimulating hormone (TSH)

A. Prolactin A Prolactinoma can result in anovulation becausehigh levels of prolactin suppress release of LHRH (gonadotropin releasing hormone), causing suppression of growth hormone (GH), FSH, and estrogen. Prolactinoma is the most commonly occurring pituitary tumor accounting for 40%-60%. Adenomas producing FSH have a frequency of about 20%, while those pituitary tumors secreting LH and TSH are rare.

Which statement best describes immunofixation electrophoresis (IEF)? A. Proteins are separated by electrophoresis followed by overlay of monospecific anti-immunoglobulins B. Proteins react with monospecific antisera followed by electrophoresis C. Antisera are electrophoresed, then diffused against patient's serum D. Serum is electrophoresed; the separated immunoglobulins diffuse against specific antisera placed into troughs

A. Proteins are separated by electrophoresis followed by overlay of monospecific anti-immunoglobulins A Immunofixation electrophoresis (IFE) is used to identify monoclonal bands in serum or urine. Electrophoresis is performed on the serum or urine sample in the same manner as for protein electrophoresis, except that six lanes are used for the same sample. After the proteins are separated, a different monospecific antiserum is applied across the surface of each lane. After incubating, the gel is washed and blotted to remove uncomplexed proteins and salts. The immune complexes that remain are stained. Monoclonal bands will be seen only in those lanes where the monoclonal immunoglobulins were recognized by the corresponding antiserum.

Which of the following is the most accurate measurement of Pi in serum? A. Rate of unreduced phosphomolybdate formation at 340 nm B. Measurement of phosphomolybdenum blue at 680 nm C. Use of aminonaptholsulfonic acid to reduce phosphomolybdate D. Formation of a complex with malachite green dye

A. Rate of unreduced phosphomolybdate formation at 340 nm A The colorimetric method (Fiske and SubbaRow) used previously for Pi reacted ammonium molybdate with Pi, forming ammonium phosphomolybdate (NH4)3[PO4(MoO3)12]. A reducing agent, aminonaptholsulfonic acid (ANS), was added, forming phosphomolybdenum blue. The product was unstable and required sulfuric acid, making precipitation of protein a potential source of error. These problems are avoided by measuring the rate of formation of unreduced phosphomolybdate at 340 nm

Light scattering when the wavelength is greater than 10 times the particle diameter is described by: A. Rayleigh's law B. Te Beer-Lambert law C. Mie's law D. Te Rayleigh-Debye law

A. Rayleigh's law A Rayleigh's law states that when the incident wavelength is much longer than the particle diameter, there is maximum backscatter and minimum right-angle scatter. The Rayleigh-Debye law predicts maximum right-angle scatter when wavelength and particle diameter approach equality. In nephelometry, the relationship between wavelength and diameter determines the angle at which the detector is located

The term pharmacokinetics refers to the: A. Relationship between drug dose and the drug blood level B. Concentration of drug at its sites of action C. Relationship between blood concentration and therapeutic response D. The relationship between blood and tissue drug levels

A. Relationship between drug dose and the drug blood level A Pharmacokinetics is the mathematical expression of the relationship between drug dose and drug blood level. When the appropriate formula is applied to quantitative measures of drug dose, absorption, distribution, and elimination, the blood concentration can be accurately determined

Which of the following conditions is associated with hyperuricemia? A. Renal failure B. Chronic liver disease C. Xanthine oxidase deficiency D. Paget's disease of the bone

A. Renal failure A Excessive retention of uric acid results from renal failure and diuretics (or other drugs) that block uric acid excretion. Hyperuricemia may result from overproduction of uric acid in primary essential gout or excessive cell turnover associated with malignancy and chemotherapy. Overproduction may also result from an enzyme deficiency in the pathway forming guanosine triphosphate (GTP) or adenosine monophosphate (AMP) (purine salvage). Hyperuricemia is also associated with ketoacidosis and lactate acidosis, hypertension, and hyperlipidemia. Xanthine oxidase converts xanthine to uric acid; therefore, a deficiency of this enzyme results in low serum levels of uric acid. Paget's disease of bone causes cyclic episodes of bone degeneration and regeneration and is associated with very high serum ALP and urinary calcium levels.

Which of the following conditions is associated with total body sodium excess? A. Renal failure B. Hyperthyroidism C. Hypoparathyroidism D. Diabetic ketoacidosis

A. Renal failure A Total body sodium excess often occurs in persons with renal failure, congestive heart failure, and cirrhosis of the liver. When water is retained along with sodium, total body sodium excess results rather than hypernatremia. Heart failure causes sodium and water retention by reducing blood flow to the kidneys. Cirrhosis causes obstruction of hepatic lymphatics and portal veins, leading to local hypertension and accumulation of ascites fluid. Renal failure results in poor glomerular filtration and isosmotic equilibration of salt and water.

An AFP measured on a 30-year-old pregnant woman at approximately 12 weeks gestation is 2.5 multiples of the median (MOM). What course of action is most appropriate? A. Repeat the serum AFP in 2 weeks B. Recommend AFP assay on amniotic fluid C. Repeat the AFP using the same sample by another method D. Repeat the AFP using the sample by the same method

A. Repeat the serum AFP in 2 weeks A The analytical sensitivity of immunochemical AFP tests is approximately 5 ng/mL. The maternal serum AFP at 12 weeks' gestation is barely above the analytical detection limit. Therefore, to achieve the needed sensitivity, the test should be repeated at 14 weeks. If the result is still equal to or greater than 2.5 MOM, then ultrasound should be performed to verify last menstrual period dating. AFP normally first becomes detectable in maternal serum at week 12 and increases by 15% per week through the 26th week. Levels of 2.5 MOM or greater are associated with spina bifida but also occur in ventral wall and abdominal wall defects, fetal death, Turner's syndrome, trisomy 13, congenital hypothyroidism, tyrosinemia, and several other fetal conditions. A positive serum test should always be repeated, and if positive again, followed by ultrasound. If ultrasound does not explain the elevation, amniotic fluid testing including AFP and acetylcholinesterase is usually recommended.

SITUATION: A technologist is performing an enzyme assay at 340 nm using a visible-range spectrophotometer. After setting the wavelength and adjusting the readout to zero %T with the light path blocked, a cuvette with deionized water is inserted. With the light path fully open and the 100%T control at maximum, the instrument readout will not rise above 90%T. What is the most appropriate first course of action? A. Replace the source lamp B. Insert a wider cuvette into the light path C. Measure the voltage across the lamp terminals D. Replace the instrument fuse

A. Replace the source lamp A Visible spectrophotometers are usually supplied with a tungsten or quartz halogen source lamp. Tungsten lamps produce a continuous range of wavelengths from about 320-2,000 nm. Output increases as wavelength becomes longer peaking at around 1,000 nm, and is poor below 400 nm. As the lamp envelope darkens with age, the amount of light reaching the photodetector at 340 nm becomes insufficient to set the blank reading to 100%T. Quartz halogen lamps produce light from 300 nm through the infrared region. Deuterium or hydrogen lamps produce ultraviolet-rich spectra optimal for ultraviolet (UV) work. Mercury vapor lamps produce a discontinuous spectrum that includes a high output at around 365 nm that is useful for fluorescent applications. Xenon lamps generate a continuous spectrum of fairly uniform intensity from 300-2,000 nm, making them useful for both visible and UV applications.

SITUATION: A patient's biochemistry results are: ALT = 55 IU/L AST = 165 IU/L glucose = 87 mg/dL LD = 340 IU/L Na = 142 mmol/L K = 6.8 mmol/L Ca = 8.4 mg/dL Pi = 7.2 mg/dL Select the best course of action. A. Report results along with an estimate of the degree of hemolysis B. Repeat LD but report all other results C. Request a new sample D. Dilute the serum 1:2 and repeat AST and LD

A. Report results along with an estimate of the degree of hemolysis A Results indicate a moderately hemolyzed sample. Because sodium, calcium, and glucose are not significantly affected, results should be reported along with an estimate of visible hemolysis. The physician may reorder affected tests of interest.

Which of the following conditions is associated with hypophosphatemia? A. Rickets B. Multiple myeloma C. Renal failure D. Hypervitaminosis D

A. Rickets A Rickets can result from dietary phosphate deficiency, vitamin D deficiency, or an inherited disorder of either vitamin D or phosphorus metabolism. Vitamin D-dependent rickets (VDDR) can be reversed by megadoses of vitamin D. Type 1 is caused by a deficiency in renal cells of 1-α-hydroxylase, an enzyme that converts 25 hydroxyvitamin D to the active form, 1,25 hydroxyvitamin D. Type 2 is caused by a deficiency in the vitamin D receptor of bone tissue. Vitamin D-resistant rickets (VDRR) is caused by a deficiency in the renal reabsorption of phosphate. Consequently, affected persons (usually men because it is most commonly X-linked) have a normal serum calcium and a low Pi.

SITUATION: Laboratory results on a patient from the emergency department are: glucose = 1,100 mg/dL Na = 155 mmol/L K = 1.2 mmol/L Cl = 115 mmol/L TCO2 = 3.0 mmol/L What is the most likely explanation of these results? A. Sample drawn above an IV B. Metabolic acidosis with increased anion gap C. Diabetic ketoacidosis D. Laboratory error measuring electrolytes caused by hyperglycemia

A. Sample drawn above an IV A These results are consistent with dilution of venous blood by intravenous fluid containing 5% dextrose and normal saline. The intravenous fluid is free of potassium and bicarbonate, accounting for the low level of these electrolytes (incompatible with life).

Which measurement principle is employed in a vapor pressure osmometer? A. Seebeck B. Peltier C. Hayden D. Darlington

A. Seebeck A The Seebeck effect refers to the increase in voltage across the two junctions of a thermocouple caused by a difference in the temperature at the junctions. Increasing osmolality lowers the dew point of a sample. When sample is cooled to its dew point, the voltage change across the thermocouple is directly proportional to osmolality.

Referring to the Levy-Jennings chart, what analytical error is present during the second half of the month? A. Shift B. Trend C. Random error D. Kurtosis

A. Shift A A shift is characterized by six consecutive points lying on the same side of the mean. This occurs from day 15 to day 20. Shifts are caused by a change in the assay conditions that affect the accuracy of all results, such as a change in the concentration of the calibrator; change in reagent; a new lot of reagent that differs in composition; or improper temperature setting, wavelength, or sample volume. The term kurtosis refers to the degree of flatness or sharpness in the peak of a set of values having a Gaussian distribution

Which of the following conditions will interfere with the measurement of LD? A. Slight hemolysis during sample collection B. Storage at 4°C for 3 days C. Storage at room temperature for 16 hours D. Use of plasma collected in heparin

A. Slight hemolysis during sample collection A RBCs are rich in LD-1 and LD-2, and even slight hemolysis will falsely elevate results. Hemolytic, megaloblastic, and pernicious anemias are associated with LD levels of 10-50 times the URL. LD is stable for 2 days at room temperature or 1 week at 4°C; however, freezing causes deterioration of LD-5. The activity of LD is inhibited by EDTA, which binds divalent cations; serum or heparinized plasma should be used.

In which condition would an elevated serum alkaline phosphatase be likely to occur? A. Small cell lung carcinoma B. Hemolytic anemia C. Prostate cancer D. Acute myocardial infarction

A. Small cell lung carcinoma A The primary diagnostic utility of ALP is to help differentiate necrotic jaundice (↑ALT) from obstructive jaundice (↑ ALP). ALP is also increased in several bone diseases. Large increases are seen in Paget's disease, moderate increases in bone cancer, and slight increases in rickets. Total ALP may be slightly increased in osteoporosis but often it is not. In addition to obstructive jaundice and bone diseases, alkaline phosphatase is a tumor marker. In most cases, the alkaline phosphatase is the product of fetal gene activation, and resembles placental ALP (e.g., hepatoma, small cell carcinoma of the lung, ovarian cancer). Leukemia and Hodgkin's disease may cause an elevated leukocyte or bone-derived ALP.

Which electrolyte level best correlates with plasma osmolality? A. Sodium B. Chloride C. Bicarbonate D. Calcium

A. Sodium A Sodium and chloride are the major extracellular ions. Chloride passively follows sodium, making sodium the principal determinant of plasma osmolality

Which of the following components is not needed in a chemiluminescent immunoassay analyzer? A. Source lamp B. Monochromator C. Photodetector D. Wash station

A. Source lamp A Chemiluminescence is the production of light following a chemical reaction. Immunoassays based upon chemiluminescence generate light when the chemiluminescent molecule becomes excited; therefore, a light source is not used. In immunoassay platforms, chemiluminescent molecules such as acridinium can be used to label antigens or antibodies. Alternatively, chemiluminescent substrates such as luminol or dioxetane phosphate may be used. Light will be emitted when the enzyme-labeled molecule reacts with the substrate. In such assays, free and bound antigen separation is required and is usually accomplished using paramagnetic particles bound to either antibody or reagent antigen

Which of the following statistical tests is used to compare the means of two methods? A. Student's t test B. F distribution C. Correlation coefficient (r) D. Linear regression anal

A. Student's t test A Student's t test is the ratio of mean difference to the standard error of the mean difference (bias/random error) and tests for a significant difference in means. The F test is the ratio of variances and determines if one method is significantly less precise. The correlation coefficient is a measure of the association between two variables and should be high in any method comparison. An r value less than 0.90 in method comparisons usually occurs when the range of results is too narrow.

Correction of pH for a patient with a body temperature of 38°C would require: A. Subtraction of 0.015 B. Subtraction of 0.01% C. Addition of 0.020 D. Subtraction of 0.020

A. Subtraction of 0.015 A The pH decreases by 0.015 for each degree Celsius above the 37°C. Because the blood gas analyzer measures pH at 37°C, the in vivo pH would be 0.015 pH units below the measured pH.

Which of the following reagents is used in the direct HDL cholesterol method? A. Sulfated cyclodextrin B. Magnesium sulfate and dextran sulfate C. Anti-apoA-I D. Manganese heparin

A. Sulfated cyclodextrin A The direct HDL cholesterol method most commonly employed uses cholesterol esterase and oxidase enzymes conjugated to polyethylene glycol. In the presence of sulfated cyclodextrin, the enzymes do not react with non-HDL cholesterol molecules. Anti-apoA-I binds to HDL and is not used in HDL assays

Which of the following laboratory results is usually associated with cystic fibrosis? A. Sweat chloride greater than 60 mmol/L B. Elevated serum sodium and chloride C. Elevated fecal trypsin activity D. Low glucose

A. Sweat chloride greater than 60 mmol/L A Cystic fibrosis causes obstruction of the exocrine glands including the sweat glands, mucus glands, and pancreas. Newborns with pancreatic involvement demonstrate fecal trypsin deficiency, which may be detected by a low fecal chymotrypsin or immunoreactive trypsin result. However, these tests require confirmation. Serum sodium and chloride levels are low. More than 98% of affected infants have elevated sweat sodium and chloride and low serum levels. Sweat chloride in excess of 60 mmol/L confirms the clinical diagnosis. Some persons with the disease have insulin deficiency and elevated blood glucose. Genetic tests are available to detect several mutations that occur at the cystic fibrosis transmembrane conductance regulator (CFTR) locus on chromosome 7

Which of the following statements about blood alcohol measurement is correct? A. Symptoms of intoxication usually begin when the level exceeds 0.05% w/v B. The skin puncture site should be disinfected with isopropanol C. The reference method is based upon enzymatic oxidation of ethanol by alcohol dehydrogenase D. Gas chromatography methods require extraction of ethanol from serum

A. Symptoms of intoxication usually begin when the level exceeds 0.05% w/v A Alcohol dehydrogenase is not specific for ethanol, and in vitro interference can occur with some ADH methods when skin is disinfected with other alcohols. For this reason, and to avoid interference with the interpretation of chromatograms for volatiles, blood samples are collected after disinfecting the skin site with benzalkonium chloride or other nonalcohol antiseptic. GLC is the legally accepted method of ethanol analysis. The low boiling point of ethanol permits direct analysis on blood or plasma diluted with water containing 1-propanol or other suitable internal standard.

Why are three levels used for quality control of pH and blood gases? A. Systematic errors can be detected earlier than with two controls B. Analytical accuracy needs to be greater than for other analytes C. High, normal, and low ranges must always be evaluated D. A different level is needed for pH, PCO2, and PO2

A. Systematic errors can be detected earlier than with two controls A Error detection occurs sooner when more controls are used. Some errors, such as those resulting from temperature error and protein coating of electrodes, are not as pronounced near the calibration point, as in the acidosis and alkalosis range. The minimum requirement for blood gas QC is one sample every 8 hours and three levels (acidosis, normal, alkalosis) every 24 hours. Three levels of control are also used commonly for therapeutic drug monitoring and hormone assays because precision differs significantly in the high and low ranges.

Of the methods used to measure amino acids, which is capable of measuring fatty acids simultaneously? A. Tandem-mass spectroscopy B. High-performance liquid chromatography C. Capillary electrophoresis D. Two-dimensional thin-layer chromatography

A. Tandem-mass spectroscopy A All four methods are able to separate each amino acid (up to 40 species); however, tandem-mass spectroscopy with electrospray ionization can measure amino acids, organic acids such as methylmalonic acid, and fatty acids. The acids are eluted from the dried blood spot with methanol after addition of internal standards and then derivatized with butanol-hydrochloric acid. Soft ionization of the butyl esters of the amino acids and butyl acylcarnitines of organic and fatty acids yields parent ions, and these are fragmented by collision with argon in the second mass filter to produce daughter ions. A process called multiple reaction monitoring identifies both parent ions and neutral fragments that identify the acids. Carnitines are quarternary ammonium compounds that carry the acids across the mitochondrial membrane.

In general, in which of the following situations is the analysis of a tumor marker most useful? A. Testing for recurrence B. Prognosis C. Screening D. Diagnosis

A. Testing for recurrence A Most tumor markers are expressed at very low levels so that the concentration in early malignancy overlaps that seen in normal individuals. This makes them ineffective for screening. Three exceptions are hCG in males for testicular cancer, calcitonin for thyroid medullary cancer, and prostate-specific antigen (PSA) for prostate cancer. Most tumor markers are increased in nonmalignant disease, and this nonspecificity reduces their usefulness for diagnosis of malignancy. In addition to the three markers mentioned, the hormones insulin (insulinoma), gastrin (gastrinoma), and prolactin (prolactinoma), and the catecholamines (pheochromocytoma) have some diagnostic utility. Some tumor markers are useful predictors of disease progression and response to treatment. These include BRCA-1, estrogen and progesterone receptors, cathepsin-D, and the Philadelphia chromosome. The major use of tumor markers is to monitor recurrence and therapy. Successful treatment reduces the concentration of the marker significantly or results in an undetectable level. A rise in level following treatment signals recurrence.

When referring to quality control (QC) results, what parameter usually determines the acceptable range? A. The 95% confidence interval for the mean B. The range that includes 50% of the results C. The central 68% of results D. The range encompassed by ±2.5 standard deviations

A. The 95% confidence interval for the mean A The acceptable range for quality control results is usually set at the 95% confidence interval. This is defined as the range between -1.96s and +1.96s. This means that we can expect a QC result to fall within this range 95 out of 100 times. For practical purposes, this is the same as ±2 s (95.4 out of 100 results should fall within ±2 s of the mean on the basis of chance).

Which statement regarding bone-specific ALP is true? A. The bone isoenzyme can be measured immunochemically B. Bone ALP is increased in bone resorption C. Bone ALP is used for the diagnosis of osteoporosis D. There are two distinct bone isoenzymes

A. The bone isoenzyme can be measured immunochemically A Bone ALP assays (Ostase and Alkphase-B) use monoclonal antibodies to measure the bone isoenzyme in mass units. The assays may be used to monitor bone remodeling by osteoblasts in osteoporosis, and thus, are useful for following treatment. Bone specific ALP is not sufficiently sensitive to diagnose osteoporosis, and antibodies may cross-react with other ALP isoenzymes, depending on their source.

Urea is produced from: A. The catabolism of proteins and amino acids B. Oxidation of purines C. Oxidation of pyrimidines D. The breakdown of complex carbohydrates

A. The catabolism of proteins and amino acids A Urea is generated by deamination of amino acids. Most is derived from the hepatic catabolism of proteins. Uric acid is produced by the catabolis of purines. Oxidation of pyrimidines produces orotic acid.

The reagent blank corrects for absorbance caused by: A. The color of reagents B. Sample turbidity C. Bilirubin and hemolysis D. All of these options

A. The color of reagents A When a spectrophotometer is set to 100%T with the reagent blank instead of water, the absorbance of reagents is automatically subtracted from each unknown reading. The reagent blank does not correct for absorbance caused by interfering chromogens in the sample such as bilirubin, hemolysis, or turbidity.

In the Henderson-Hasselbalch expression pH = 6.1 + log HCO3 - /dCO2, the 6.1 represents: A. The combined hydration and dissociation constants for CO2 in blood at 37°C B. The solubility constant for CO2 gas C. The dissociation constant of H2O D. The ionization constant of sodium bicarbonate (NaHCO3)

A. The combined hydration and dissociation constants for CO2 in blood at 37°C A The equilibrium constant, Kh, for the hydration of CO2 (dCO2 + H2O → H2CO3) is only about 2.3 × 10-3M, making dCO2 far more prevalent than carbonic acid. The dissociation constant, Kd, for the reaction H2CO3 →H+ + HCO3 - is about 2 × 10-4 M. The product of these constants is the combined equilibrium constant, K´. The negative logarithm of K´ is the pK´, which is 6.103 in blood at 37°C.

Bichromatic measurement of absorbance can correct for interfering substances if: A. The contribution of the interferent to absorbance is the same at both wavelengths B. Both wavelengths pass through the sample simultaneously C. The side band is a harmonic of the primary wavelength D. The chromogen has the same absorbance at both wavelengths

A. The contribution of the interferent to absorbance is the same at both wavelengths A In bichromatic photometry, the absorbance of sample is measured at two different wavelengths. The primary wavelength is at or near the absorbance maximum. An interfering substance having the same absorbance at both primary and secondary (side band) wavelengths does not affect the absorbance difference (Ad).

Which of two instruments can be assumed to have the narrower bandpass? Assume that wavelength is accurately calibrated. A. The instrument giving the highest absorbance for a solution of 0.1 mmol/L NADH at 340 nm B. The instrument giving the lowest %T for a solution of nickel sulfate at 700 nm C. The instrument giving the highest %T reading for 1.0% v/v HCl at 350 nm D. The instrument giving the most linear plot of absorbance versus concentration

A. The instrument giving the highest absorbance for a solution of 0.1 mmol/L NADH at 340 nm A Bandpass is defined by the range of wavelengths passed through the sample at the specified wavelength setting. It can be measured using any solution having a narrow absorbance peak (e.g., NADH at 340 nm). The instrument producing the purest monochromatic light will have the highest absorbance reading.

Which of the following factors is most likely to cause a falsely low result when using the BCG dye-binding assay for albumin? A. The presence of penicillin B. An incubation time of 120 seconds C. The presence of bilirubin D. Lipemia

A. The presence of penicillin A BCG and BCP are not significantly affected by bilirubin or hemolysis, although negative interference caused by free Hgb has been reported with some BCG methods. Lipemic samples may cause positive interference, which can be eliminated by serum blanking. Incubation times as long as 2 minutes result in positive interference from globulins, which react with the dye. Penicillin and some other anionic drugs bind to albumin at the same site as the dye, causing falsely low results

The half-band width of a monochromator is defined by: A. The range of wavelengths passed at 50% maximum transmittance B. One-half the lowest wavelength of optical purity C. The wavelength of peak transmittance D. One-half the wavelength of peak absorbance

A. The range of wavelengths passed at 50% maximum transmittance A Half-band width is a measure of bandpass made using a solution or filter having a narrow natural bandpass (transmittance peak). The wavelength giving maximum transmittance is set to 100%T (or 0 A). Then, the wavelength dial is adjusted downward, until a readout of 50%T (0.301 A) is obtained. Next, the wavelength is adjusted upward until 50%T is obtained. The wavelength difference is the half-band width. The narrower the half-band width, the better the photometric resolution of the instrument.

Which of the following is a characteristic of obstructive jaundice? A. The ratio of direct to total bilirubin is greater than 1:2 B. Conjugated bilirubin is elevated, but unconjugated bilirubin is normal C. Urinary urobilinogen is increased D. Urinary bilirubin is normal

A. The ratio of direct to total bilirubin is greater than 1:2 A Obstruction prevents conjugated bilirubin from reaching the intestine, resulting in decreased production, excretion, and absorption of urobilinogen. Conjugated bilirubin regurgitates into sinusoidal blood and enters the general circulation via the hepatic vein. The level of serum direct (conjugated) bilirubin becomes greater than unconjugated bilirubin. The unconjugated form is also increased because of accompanying necrosis, deconjugation, and inhibition of UDP-glucuronyl transferase

The following results are reported on an adult male patient being evaluated for chest pain: Myoglobin Troponin I CK-MB (Cutoff = (Cutoff = (Cutoff = 100 µg/L) 0.03 µg/L) 4 µg/L) Admission 12 µg/L 1.1 µg/L 18 µg/L 3 hours 360 µg/L 1.8 µg/L 26 µg/L postadmission 6 hours 300 µg/L 2.4 µg/L 40 µg/L postadmission What is the most likely cause of these results? A. The wrong sample was assayed for the first myoglobin B. The patient did not suffer an MI until after admission C. Hemolysis caused interference with the 3-hour and 6-hour myoglobin result D. The patient is experiencing unstable angina

A. The wrong sample was assayed for the first myoglobin A Myoglobin is the first cardiac marker to rise outside the URL following an MI (2-3 hours) followed by TnI (4-6 hours) and CK-MB (4-8 hours). The admission TnI and CK-MB are both elevated, and they continue to rise in all three samples. Because TnI and CK-MB peak before 24 hours post-AMI, the infarction likely occurred within the last 12-24 hours. The myoglobin can remain elevated for up to 36 hours post-AMI and should have been elevated in the admission sample.

Serum samples collected from hospitalized patients over a 2 week period are split into two aliquots and analyzed for prostate specific antigen (PSA) by two methods. Each sample was assayed by both methods within 30 minutes of collection by a technologist familiar with both methods. The reference method is method × (upper reference limit = 4.0 µg/L). Linear regression analysis was performed by the least-squares method, and results are as follows: Linear Regression/ Correlation Coefficient (r)/ Std Error of Estimates (sy/x) ŷ = 2.10 + 1.01x / 0.984 /0.23 Which statement best characterizes the relationship between the methods? A. There is a significant bias caused by constant error B. There is a significant proportional error C. There is no disagreement between the methods because the correlation coefficient approaches 1.0 D. There is no systematic error, but the random error of the new method is unacceptable

A. There is a significant bias caused by constant error A The linear regression analysis is the most useful statistic to compare paired patient results because it estimates the magnitude of specific errors. The y intercept of the regression line is a measure of constant error, and the slope is a measure of proportional error. Together, these represent the bias or SE of the new method. The correlation coefficient is influenced by the range of the sample and the RE. Two methods that measure the same analyte will have a high correlation coefficient, provided the concentrations are measured over a wide range, and this statistic should not be used to judge the acceptability of the new method. The standard error of estimate is a measure of the closeness of data points to the regression line and is an expression of RE.

Two freezing point osmometers are compared by running 40 paired patient samples one time on each instrument, and the following results are obtained: Instrument / Mean / SD Osmometer A /280 mOsm/kg /3.1 Osmometer B /294 mOsm/kg /2.8 If the critical value for F = 2.8, then what conclusion can be drawn regarding the precision of the two instruments? A. There is no statistically significant difference in precision B. Osmometer A demonstrates better precision that is statistically significant C. Osmometer B demonstrates better precision that is statistically significant D. Precision cannot be evaluated statistically when single measurements are made on samples

A. There is no statistically significant difference in precision A The F test determines whether there is a statistically significant difference in the variance of the two sampling distributions. Assuming the samples are collected and stored in the same way and the analysis is done by a technologist who is familiar with the instrument, then differences in variance can be attributed to a difference in instrument precision. The F test is calculated by dividing the variance (s1)2 of the instrument having the higher standard deviation by the variance (s2)2 of the instrument having the smaller standard deviation. F = (s1)2 ÷ (s2)2 = (3.1)2 ÷ (2.8)2 = 9.61 ÷7.84 = 1.22 If the value of F is smaller than the critical value at the 0.10 level of significance, then the hypothesis (there is no significant difference in the variance of the two instruments) is accepted.

What component of a freezing point osmometer measures the sample temperature? A. Thermistor B. Thermocouple C. Capacitor D. Electrode

A. Thermistor A A thermistor is a temperature sensitive resistor. The resistance to current flow increases as temperature falls. The temperature at which a solution freezes can be determined by measuring the resistance of the thermistor. Resistance is directly proportional to the osmolality of the sample.

A patient has a plasma CK-MB of 14 µg/L at admission and a total CK of 170 IU/L. Serum myoglobin is 130 µg/L and TnI is 1.6 µg/L. Three hours later, the TnI is 3.0 µg/L. Which statement best describes this situation? A. This patient has had an AMI and further testing is unnecessary B. A second CK-MB and myoglobin test should have been performed at 3 hours postadmission to confirm AMI C. These results are consistent with skeletal muscle damage associated with a crush injury that elevated the CK-MB D. Further testing 6-12 hours postadmission is required to establish a diagnosis of AMI

A. This patient has had an AMI and further testing is unnecessary A Results on admission indicate strongly that the patient has suffered an MI. The 3-hour TnI confirms this and rules out the possibility of a sample collection or transcription error for the admission sample. Repeat testing of other cardiac markers at 3 hours was not necessary because admission results were significantly increased for all three markers. Skeletal muscle damage or crush injury does not cause an increase in cardiac TnI.

What is the purpose of an internal standard in HPLC and GC methods? A. To compensate for variation in extraction and injection B. To correct for background absorbance C. To compensate for changes in flow rate D. To correct for coelution of solutes

A. To compensate for variation in extraction and injection A Internal standards should have the same affinity as the analyte for the extraction reagents. Dividing peak height (or area) of all samples (standards and unknowns) by the peak height (or area) of the internal standard reduces error caused by variation in extraction recovery and injection volume.

Which statement regarding total and direct bilirubin levels is true? A. Total bilirubin level is a less sensitive and specific marker of liver disease than the direct level B. Direct bilirubin exceeds 3.5 mg/dL in most cases of hemolytic anemia C. Direct bilirubin is normal in cholestatic liver disease D. The ratio of direct to total bilirubin exceeds 0.40 in hemolytic anemia

A. Total bilirubin level is a less sensitive and specific marker of liver disease than the direct leve A Direct bilirubin measurement is a sensitive and specific marker for hepatic and posthepatic jaundice because it is not elevated by hemolytic anemia. In hemolytic anemia, the total bilirubin does not exceed 3.5 mg/dL, and the ratio of direct to total is less than 0.20. Unconjugated bilirubin is the major fraction in necrotic liver disease because microsomal enzymes are lost. Unconjugated bilirubin is elevated along with direct bilirubin in cholestasis because some necrosis takes place and some conjugated bilirubin is hydrolyzed back to unconjugated bilirubin.

A person has a fasting triglyceride level of 240 mg/dL. The physician wishes to know the patient's non-HDL cholesterol level. What cholesterol fractions should be measured? A. Total cholesterol and HDL cholesterol B. Total cholesterol and LDL cholesterol C. HDL cholesterol and LDL cholesterol D. Total cholesterol and chylomicrons

A. Total cholesterol and HDL cholesterol A When the HDL cholesterol is subtracted from the total cholesterol, the result is called the non-HDL cholesterol. This result, the sum of LDL cholesterol and VLDL cholesterol, represents the fraction with atherogenic remnant lipoproteins as well as LDL cholesterol. People who have a fasting triglyceride ≥ 200 mg/dL are at increased risk for coronary artery disease owning to atherogenic VLDL remnants, and the treatment goal is to have a non-HDL cholesterol no more than 30 mg/dL greater than the LDL cholesterol.

Which of the following statements regarding proteins is true? A. Total protein and albumin are about 10% higher in ambulatory patients B. Plasma total protein is about 20% higher than serum levels C. Albumin normally accounts for about one-third of the cerebrospinal fluid total protein D. Transudative serous fluid protein is about two-thirds of the serum total protein

A. Total protein and albumin are about 10% higher in ambulatory patients A Water pools in the vascular bed in nonambulatory patients, lowering the total protein, albumin, hematocrit, and calcium. Plasma levels of total protein are 0.2-0.4 g/dL higher than serum (about 5%) owing to fibrinogen. Cerebrospinal fluid albumin levels are normally 10-30 mg/dL, which is approximately two-thirds of the CSF total protein. Transudates have a total protein below 3.0 g/dL and less than 50% of the serum total protein

In which type of liver disease would you expect the greatest elevation of LD? A. Toxic hepatitis B. Alcoholic hepatitis C. Cirrhosis D. Acute viral hepatitis

A. Toxic hepatitis A Liver disease produces an elevated LD-4 and LD-5. Levels may reach up to 10 times the URL in toxic hepatitis and in hepatoma. However, LD levels are lower in viral hepatitis (2-5 × URL), only slightly elevated in cirrhosis (2-3 × URL) and not significantly elevated in alcoholic liver disease

Which of the following statements about cortisol in Cushing's syndrome is true? A. Twenty-four-hour urinary free cortisol is a more sensitive test than plasma total cortisol B. Patients with Cushing's disease show pronounced diurnal variation in serum cortisol C. Free cortisol is increased by a high-serum cortisol-binding protein concentration D. An elevated serum total cortisol level is diagnostic of Cushing's syndrome

A. Twenty-four-hour urinary free cortisol is a more sensitive test than plasma total cortisol A Serum cortisol can be increased by factors such as stress, medications, and cortisol-binding protein, and the cortisol level of normal patients will overlap those seen in Cushing's syndrome because of pulse variation. When cortisol levels become elevated, cortisol-binding protein becomes saturated, and free (unbound) cortisol is filtered by the glomeruli. Most is reabsorbed, but a significant amount reaches the urine as free cortisol. Twenty-four-hour urinary free cortisol avoids the diurnal variation that may affect plasma free cortisol levels and is a more sensitive test than serum total or free cortisol.

Which of the following conditions is cause for rejecting an analytical run? A. Two consecutive controls greater than 2 s above or below the mean B. Tree consecutive controls greater than 1 s above the mean C. Four controls steadily increasing in value but less than ±1 s from the mean D. One control above +1 s and the other below -1 s from the mean

A. Two consecutive controls greater than 2 s above or below the mean A Rejecting a run when three consecutive controls fall between 1 and 2 s or when a trend of four increasing or decreasing control results occurs would lead to frequent rejection of valid analytical runs. Appropriate control limits are four consecutive controls above or below 1 s (41s ) to detect a significant shift, and a cusum result exceeding the ±2.7 s limit to detect a significant shift or trend. When controls deviate in opposite directions, the difference should exceed 4s before the run is rejected.

A 55-year-old male with early stage prostate cancer diagnosed by biopsy had his prostate gland removed (simple prostatectomy). His PSA prior to surgery was 10.0 ng/mL. If the surgery was successful in completely removing the tumor cells, what would the PSA result be 1 month after surgery? A. Undetectable B. 1-3 ng/mL C. Less than 4 ng/mL D. Less than 10 ng/mL

A. Undetectable A If the tumor were confined to the prostate, the PSA would be undetectable 1 month following successful surgery, since there is no other tissue source of PSA. The half-life of PSA is 2.2-3.2 days, and the minimum detection limit of most assays is 0.2 ng/mL or lower. Therefore, it would require at least 2 weeks before the PSA level would be undetectable. The low minimum detection limit of the PSA assay, combined with the high tissue specificity of PSA, makes the test very sensitive in detecting recurrence.

Which uric acid method is associated with negative bias caused by reducing agents? A. Uricase coupled to the Trinder reaction B. Ultraviolet uricase reaction coupled to catalase and alcohol dehydrogenase reactions C. Measurement of the rate of absorbance decrease at 290 nm after addition of uricase D. Phosphotungstic acid using a protein-free filtrate

A. Uricase coupled to the Trinder reaction A The peroxidase-coupled uricase reaction is the most common method for measuring uric acid in serum or plasma. Uricase methods form allantoin, carbon dioxide, and hydrogen peroxide from the oxidation of uric acid. When peroxide is used to oxidize a Trinder dye (e.g., a phenol derivative and 4-aminoantipyrine), some negative bias may occur when high levels of ascorbate or other reducing agents are present. Rate UV methods are free from this interference. Reduction of phosphotungstic acid by uric acid forms tungsten blue. This colorimetric reaction is nonspecific, resulting in falsely elevated uric acid caused by proteins and many other reducing substances.

Which analyte should be reported as a ratio using creatinine concentration as a reference? A. Urinary microalbumin B. Urinary estriol C. Urinary sodium D. Urinary urea

A. Urinary microalbumin A Measurement of urinary microalbumin concentration should be reported as a ratio of albumin to creatinine (e.g., mg albumin per g creatinine). This eliminates the need for 24-hour collection in order to avoid variation caused by differences in fluid intake. A dry reagent strip test for creatinine is available that measures the ability of a creatinine-copper complex to break down H2O2 , forming a colored complex. The strip uses buffered copper II sulfate, tetramethylbenzidine, and anhydrous peroxide. Binding of creatinine in urine to copper forms a peroxidase-like complex that results in oxidation of the benzidine compound. Also, 24-hour urinary metanephrines, vanillylmandelic acid, and homovanillic acid are reported per gram creatinine when measured in infants and children in order to compensate for differences in body size.

In the enzymatic assay of bilirubin, how is measurement of both total and direct bilirubin accomplished? A. Using different pH for total and direct assays B. Using UDP glucuronyl transferase and bilirubin reductase C. Using different polarity modifiers D. Measuring the rate of absorbance decrease at different time intervals

A. Using different pH for total and direct assays A Enzymatic methods use bilirubin oxidase to convert bilirubin back to biliverdin, and measure the decrease in absorbance that results. At pH 8, both conjugated, unconjugated, and delta bilirubin react with the enzyme, but at pH 4 only the conjugated form reacts.

Select the lipoprotein fraction that carries most of the endogenous triglycerides. A. VLDL B. LDL C. HDL D. Chylomicrons

A. VLDL A The VLDL is formed in the liver largely from chylomicron remnants and hepatic-derived triglycerides. Therefore, the VLDL transports the majority of endogenous triglycerides, while the triglycerides of chylomicrons are derived entirely from dietary absorption.

Blood sample collection time for peak drug levels: A. Varies with the drug, depending on its rate of absorption B. Is independent of drug formulation C. Is independent of the route of administration D. Is 30 minutes after a bolus intravenous injection is completed

A. Varies with the drug, depending on its rate of absorption A The peak concentration of a drug is the highest concentration obtained in the dosing interval. For oral drugs, the time of peak concentration is dependent upon their rates of absorption and elimination and is determined by serial blood measurements. Peak levels for oral drugs are usually drawn 1-2 hours after administration of the dose. For drugs given intravenously, peak levels are measured immediately after the infusion is completed.

Following ultracentrifugation of plasma, which fraction correlates with pre-β lipoprotein? A. Very low-density lipoprotein (VLDL) B. Low-density lipoprotein (LDL) C. High-density lipoprotein (HDL) D. Chylomicrons

A. Very low-density lipoprotein (VLDL) A The VLDL (very low-density lipoprotein) migrates in the pre-β zone. The VLDL is about 50% triglyceride, whereas LDL is only 10% triglyceride by weight. LDL is formed from VLDL in the circulation. The process is initiated by apoC-II on VLDL activating peripheral lipoprotein lipase. Hydrolysis of triglycerides and transfer of apoproteins from VLDL to HDL result in formation of IDL. Larger IDLs are returned to the liver as remnant lipoproteins. Further hydrolysis of triglycerides, transfer cholesterol esters from HDL, and transfer of apoproteins to HDL convert IDL to LDL.

Which specimen is the sample of choice for lead screening? A. Whole blood B. Hair C. Serum D. Urine

A. Whole blood A Lead accumulates in RBCs, bones, and neural tissues, and whole blood, hair, and urine are suitable for demonstrating lead toxicity. Greatest sensitivity is obtained by using whole blood, which can detect exposure over time. Because lead is rapidly eliminated from plasma, serum or plasma should not be used to test for lead exposure. Lead binds to sulfhydryl groups of proteins such as delta-aminolevulinic acid (∆-ALA) dehydratase and ferrochelatase and interferes with heme synthesis. This results in increased free erythrocyte protoporphyrin, erythrocyte zinc protoporphyrin, urinary coproporphyrin III, and δ aminolevulinic acid, which are also useful markers for lead poisoning. When screening for lead poisoning in children, the method of choice is graphite furnace atomic absorption spectrophotometry or inductively coupled plasma mass spectroscopy because they offer the best analytical sensitivity. The CDC cutoff for normal lead in children is less than 5.0 μg/dL

Which substrate is used in the Bowers-McComb method for ALP? A. p-Nitrophenyl phosphate B. β-Glycerophosphate C. Phenylphosphate D. α-Naphthylphosphate

A. p-Nitrophenyl phosphate A The method of Bowers-McComb (Szasz modification) is the IFCC-recommended method for ALP. This method uses 2-amino-2-methyl-1-propanol, pH 10.15, and measures the increase in absorbance at 405 nm as p-nitrophenyl phosphate is hydrolyzed to p-nitrophenol.

The electrophoretic pattern shown in the following densitometric tracing most likely indicates: A. α1-Antitrypsin deficiency B. Infection C. Nephrosis D. Systemic sclerosis

A. α1-Antitrypsin deficiency A. This pattern shows a marked decrease in the α1-globulin (slightly less than one-fifth of the expected peak area). Staining of the α1-globulin fraction is predominately determined by the α1-antitrypsin level. A value of less than 20% of normal (0.2-0.4 g/dL) is usually caused by homozygous α1-antitrypsin deficiency. There is a slight decrease in albumin and increase in the α2-fraction. Patients with α1-antitrypsin deficiency often display elevations in the α2-globulin and γ-globulin fraction because the condition is associated with chronic emphysema and hepatic cirrhosis.

How many significant figures should be reported when the pH of a 0.060 M solution of nitric acid is calculated? A. 1 B. 2 C. 3 D. 4

B. 2 B When zeros appear by themselves to the left of the decimal point, they are not significant. When they are to the left of the decimal point and are preceded by a number, they are significant. Zeros after the decimal point preceding a number are not significant. However, they are significant if they follow another number or are between two numbers. Therefore, 0.060 M has only two significant figures (the underlined digits). In laboratory practice, most analytes are reported with two significant figures. Routine analytes that are exceptions are pH, which includes three significant figures, and analytes with whole numbers above 100 such as sodium, cholesterol, triglycerides, and glucose

How soon following acute abdominal pain due to pancreatitis is the serum amylase expected to rise? A. 1-2 hours B. 2-12 hours C. 3-4 days D. 5-6 days

B. 2-12 hours B Serum amylase usually peaks 2-12 hours following acute abdominal pain resulting from pancreatitis. Levels reach 2-6 times the URL and return to normal within 3-4 days. Urinary amylase peaks concurrently with serum but rises higher and remains elevated for up to 1 week

A solution that has a transmittance of 1.0 %T would have an absorbance of: A. 1.0 B. 2.0 C. 1% D. 99%

B. 2.0 B A = 2.0 - log %T A = 2.0 - log 1.0 The log of 1.0 = 0 A = 2.0

Urea concentration is calculated from the BUN by multiplying by a factor of: A. 0.5 B. 2.14 C. 6.45 D. 14

B. 2.14 B BUN is multiplied by 2.14 to give the ureaconcentration in mg/dL. BUN (mg/dL) = urea × (% N in urea ÷ 100) Urea = BUN × 1/(% N in urea ÷100) Urea = BUN × (1/0.467) = 2.14

What is the pH of a buffer containing 40.0 mmol/L NaHC2O4 and 4.0 mmol/L H2C2O4? (pKa = 1.25) A. 1.35 B. 2.25 C. 5.75 D. 6.12

B. 2.25 B The Henderson-Hasselbalch equation can be used to determine the pH of a buffer containing a weak acid and a salt of the acid. pH = pKa + log salt / Acid = 1.25 + log 40.0 mmol/L / 4.0 mmol/L = 1.25 + log 10 = 2.25

A solvent needed for HPLC requires a 20.0 mmol/L phosphoric acid buffer, pH 3.50, made by mixing KH2PO4 and H3PO4. How many grams of KH2PO4 are required to make 1.0 L of this buffer? Formula weights: KH2PO4 = 136.1; H3PO4 = 98.0; pKa H3PO4 = 2.12 A. 1.96 g B. 2.61 g C. 2.72 g D. 19.2 g

B. 2.61 g B The Henderson-Hasselbalch equation is used to calculate the ratio of salt to acid needed to give a pH of 3.50. pH = pKa + log(salt/acid) 3.50 = 2.12 + log(KH2PO4/H3PO4) 1.38 = log(KH2PO4/H3PO4) antilog 1.38 = KH2PO4/H3PO4 KH2PO4/H3PO4 = 23.99 Rearranging gives KH2PO4 = 23.99 × H3PO4. Because the phosphate in the buffer is 20.0 mmol/L, then H3PO4 + KH2PO4 must equal 20. Because KH2PO4 = 23.99 × H3PO4 then: H3PO4 + (23.99 × H3PO4) = 20.0 mmol/L 24.99 × H3PO4 = 20.0 mmol/L H3PO4 = 20.0/24.99 = 0.800 mmol/L KH2PO4 = 20.0-0.800 = 19.2 mmol/L (0.0192 M)

Given the following serum electrolyte data, determine the anion gap. Na = 132 mmol/L Cl = 90 mmol/L HCO3- = 22 mmol/L A. 12 mmol/L B. 20 mmol/L C. 64 mmol/L D. Cannot be determined from the information provided

B. 20 mmol/L B The anion gap is defined as unmeasured anions minus unmeasured cations. It is calculated by subtracting the measured anions (bicarbonate and chloride) from the serum sodium (or sodium plus potassium). A normal anion gap is approximately 8-16 mmol/L (12-20 mmol/L when potassium is used). Anion gap = Na - (HCO3 + Cl) Anion gap = 132 - (90 + 22) = 20 mmol/L

What is the pH of a 0.05 M solution of acetic acid? Ka = 1.75 × 10-5, pKa = 4.76 A. 1.7 B. 3.0 C. 4.3 D. 4.6

B. 3.0 B Weak acids are not completely ionized, and pH must be calculated from the dissociation constant of the acid (in this case 1.75 × 10-5). Ka = [H+] × [Ac-]/[HAc] 1.75 x 10^-5 = [H+] × [Ac-]/5.0 x 10^-2 Since [H+] = [Ac-] X2 = (1.75 × 10^-5 ) × (5.0 × 10^-2 ) = 8.75 × 10^-7 x = √8.75 × 10^-7 = [H+] = 9.35 × 10^-4 M pH = -Log 9.35 × 10^-4 M = 3.0 Alternatively, pH = 1/2 (pKa - Log HA) pH = 1/2 (4.76 - Log 5.0 × 10^-2) = 1/2 (4.76 + 1.30) = 3.0

A patient's CK-MB is reported as 18 µg/L and the total CK as 560 IU/L. What is the CK relative index (CKI)? A. 0.10% B. 3.2% C. 10.0% D. 30.0%

B. 3.2% B The CKI is an expression of the percentage of the total CK that is attributed to CK-MB. CKI = (CK-MB in μg/L / Total CK in IU/L) × 100 The reference range is 0%-2.5%. Values above 2.5% point to an increase in CK-MB from cardiac muscle.

The following plot represents a study of a screening test for malignant prostate cancer using plasma PSA (ng/mL). Te outcome measured was positive cytology results obtained by biopsy. What concentration gives the highest sensitivity with the least number of unnecessary biopsies? A. 2.6 B. 3.6 C. 3.8 D. 5.2

B. 3.6 B A receiver operating characteristic (ROC) curve is used to identify the test result, giving the highest sensitivity with the least number of false-positive results. Sensitivity (true positives) is plotted against false positives. The number in the uppermost left corner represents the highest detection with the lowest number of false positives. In this case, a result of 3.6 ng/mL detects 72% of malignancies with 1 in 10 (10%) false positives

What is the percentage of serum calcium that is ionized (Cai)? A. 30% B. 45% C. 60% D. 80%

B. 45% B Calcium exists in serum in three forms: protein bound, ionized, and complexed (as undissociated salts). Only Cai is physiologically active. Protein bound and Cai each account for approximately 45% of total calcium, and the remaining 10% is complexed.

Which electrolyte measurement is least affected by hemolysis? A. Potassium B. Calcium C. Pi D. Magnesium

B. Calcium B Potassium, phosphorus, and magnesium are the major intracellular ions, and even slight hemolysis will cause falsely elevated results. Serum samples with visible hemolysis (20 mg/dL free Hgb) should be redrawn.

What is the American Diabetes Association recommended cutoff value for adequate control of blood glucose in diabetics as measured by glycated hemoglobin? A. 5% B. 6.5% C. 9.5% D. 11%

B. 6.5% B The ADA recommends that 6.5% be used as the cutoff for determining the adequacy of treatment for diabetes. A glycated hemoglobin test should be performed at the time of diagnosis and every 6 months thereafter if the result is < 6.5%. If the result is 6.5% or more, the treatment plan should be adjusted to achieve a lower level, and the test performed every 3 months until control is improved.

How many milliliters of a 2,000.0 mg/dL glucose stock solution are needed to prepare 100.0 mL of a 150.0 mg/dL glucose working standard? A. 1.5 mL B. 7.5 mL C. 15.0 mL D. 25.0 mL

B. 7.5 mL B To calculate the volume of stock solution needed, divide the concentration of working standard by the concentration of stock standard, then multiply by the volume of working standard that is needed. C1 × V1 = C2 × V2, where C1 = concentration of stock standard V1 = volume of stock standard C2 = concentration of working standard V2 = volume of working standard 2000.0 mg/dL × V1 = 150.0 mg/dL × 100.0 mL V1 = (150.0 ÷ 2000.0) × 100.0 mL V1 = 7.5 mL

What is the HDL cholesterol cutpoint recommend by NCEP? A. <30 mg/dL B. <40 mg/dL C. <30 mg/dL for males and < 40 mg/dL for females D. <45 mg/dL for males and < 50 mg/dL for females

B. <40 mg/dL B The HDL cholesterol cutpoint recommended by NCEP is < 40 mg/dL regardless of sex. A result below 40 mg/dL counts as a risk factor for coronary artery disease. Conversely, if the HDL cholesterol is ≥ 60 mg/dL, then one risk factor is subtracted from the total number. The therapeutic goal for someone with low HDL cholesterol is still reduction of LDL cholesterol (if elevated), weight loss, and increased exercise.

Which of the following statements about TLC for drug screening is true? A. Acidic drugs are extracted in an alkaline nonpolar solvent B. A drug is identified by comparing its Rf value and staining to standards C. Testing must be performed using a urine sample D. Opiates and other alkaloids are extracted at an acid pH

B. A drug is identified by comparing its Rf value and staining to standards B TLC can be performed on urine, serum, or gastric fluid and qualitatively identifies most drugs. Each has a characteristic Rf, which is the ratio of the distance migrated by the drug to the solvent. The Rf of the sample must match the Rf of the drug standard. Extraction of drugs for TLC is highly pH dependent. The pH must be adjusted to reduce the solubility (ionization) of the drug in the aqueous phase. Usually, alkaline drugs (e.g., opiates) are extracted at pH 9.0 and acidic drugs (e.g., barbiturates) at pH 4.5

Which statement accurately describes serum transaminase levels in AMI? A. ALT is increased 5- to 10-fold after an AMI B. AST peaks 24-48 hours after an AMI and returns to normal within 4-6 days C. AST levels are usually 20-50 times the upper limit of normal after an AMI D. Isoenzymes of AST are of greater diagnostic utility than the total enzyme level

B. AST peaks 24-48 hours after an AMI and returns to normal within 4-6 days B ALT may be slightly elevated after an AMI. AST levels can be up to 5-10 times the URL after AMI, but elevations of this range are also seen in patients with muscular dystrophy, crush injury, pulmonary embolism, infectious mononucleosis, and cancer of the liver.

Which of the following statements regarding amylase methods is true? A. Requires sulfhydryl compounds for full activity B. Activity will vary depending on the method used C. Amyloclastic methods measure the production of glucose D. Overrange samples are diluted in deionized water

B. Activity will vary depending on the method used B Chloride and Ca2+ ions are required for amylase activity. Samples with high activity should be diluted with NaCl to prevent inactivation. Lipase and CK require sulfhydryl activators. Saccharogenic methods measure the production of glucose, while amyloclastic methods measure the degradation of starch. Starch is a polymer of α-D glucose subunits linked together by both α 1-4 and α 1-6 glycosidic bonds. Different lots may have more or less branching owing to the number of α 1-6 bonds. Since amylase hydrolyzes at the α 1-4 sites only, the amount of product measured is influenced by the extent of branching.

When measuring calcium with the complexometric dye o-cresolphthalein complexone, magnesium is kept from interfering by: A. Using an alkaline pH B. Adding 8-hydroxyquinoline C. Measuring at 450 nm D. Complexing to EDTA

B. Adding 8-hydroxyquinoline B o-Cresolphthalein complexone can be used to measure either magnesium or calcium. Interference in calcium assays is prevented by addition of 8-hydroxyquinoline, which chelates magnesium. When magnesium is measured, ethyleneglycol bistetraacetic acid (EGTA) or EDTA is used to chelate calcium. Two other dyes that can be used for both magnesium and calcium assays are calmagite and methylthymol blue. Arsenazo III dye is commonly used to measure calcium. It is more specific for Ca+2 than the others, and does not require addition of a Mg+2 chelator.

Which reagent is used in thin-layer chromatography (TLC) to extract cocaine metabolites from urine? A. Acid and sodium chloride B. Alkali and organic solvent C. Chloroform and sodium acetate D. Neutral solution of ethyl acetate

B. Alkali and organic solvent B Alkaline drugs such as cocaine, amphetamine, and morphine are extracted at alkaline pH. Ideally, the pH for extracting alkaline drugs into an organic solvent should be 2 pH units greater than the negative log of dissociation constant (pKa) of the drug. More than 90% of the drug will be nonionized and will extract in ethyl acetate or another organic solvent.

The term pharmacodynamics is an expression of the relationship between: A. Dose and physiological effect B. Drug concentration at target sites and physiological effect C. Time and serum drug concentration D. Blood and tissue drug levels

B. Drug concentration at target sites and physiological effect B Pharmacodynamics is the relationship between the drug concentration at the receptor site (tissue concentration) and the response of the tissue to that drug. For example, the relationship between lidocaine concentration in the heart muscle and the duration of the action potential of Purkinje fibers.

What do "direct" or homogenous methods for LDL cholesterol assay have in common? A. They are inaccurate when plasma triglyceride is above 250 mg/dL B. All use a detergent to facilitate selective reactivity with reagent enzymes C. All use monoclonal antibodies to apo A1 and C D. All are free of interference from abnormal lipoproteins

B. All use a detergent to facilitate selective reactivity with reagent enzymes B The direct LDL cholesterol assays are all detergent based methods. One commonly used method employs a polyanionic detergent to release cholesterol from HDL, chylomicrons, and VLDL. The detergent binds to LDL and blocks its reaction with the esterase and oxidase enzymes in the reagent. Cholesterol oxidase oxidizes the non-LDL cholesterol, forming H2O2, and peroxidase catalyzes the oxidation of an electron donor by the H2O2,which does not result in color formation. A second nonionic detergent and chromogen is added. The second detergent removes the first from the LDL, allowing it to react with the enzymes. The resulting H2O2 reacts with the chromogen, forming a colored product

A biochemical profile routinely performed bimonthly on a renal dialysis patient showed a decreased serum calcium and decreased PTH level. Such a lab result may be explained by which of the following circumstances? A. Malignancy B. Aluminum toxicity C. Hypervitaminosis D D. Acidosis

B. Aluminum toxicity B Aluminum present in medications and dialysis bath fluid can cause aluminum toxicity in patients receiving dialysis. Renal failure patients often display high PTH levels owing to poor retention of calcium, and are at risk of developing osteitis fibrosa (soft bones) as a result. Excess aluminum causes osteomalacia by inhibiting release of parathyroid hormone. The finding of low PTH would not be expected with low serum calcium unless aluminum poisoning was present. Malignancy, hypervitaminosis D, and acidosis are associated with high serum calcium.

Point-of-care-tests (POCTs) for whole-blood glucose monitoring are based mainly on the use of: A. Glucose oxidase as the enzyme B. Amperometric detection C. Immunochromatography D. Peroxidase coupling reactions

B. Amperometric detection B All POCT devices for monitoring blood glucose use either glucose dehydrogenase (GDH) or glucose oxidase and are amperometric. For glucose oxidase methods, the electrons derive from the oxidation of hydrogen peroxide. For GDH, the electrons are transferred from one of several coenzymes that are reduced when glucose is oxidized, FAD+, NAD+, or PQQ (pyrroloquinoline quinone). Interferences depend upon which enzyme/coenzyme pair are used. For example, maltose and xylose interference can be pronounced with GDH/PQQ-based strips, but not with other GDH or glucose oxidase strips. Uric acid depresses glucose oxidase reactions but has no effect on GDH reactions.

Which of the following enzymes is activated by calcium ions? A. CK B. Amylase C. ALP D. LD

B. Amylase B Most enzymes require metals as activators or cofactors. CK and ALP require Mg+2 for full activity, and amylase requires Ca+2. Metals required for activity should be components of the substrate used for enzyme analysis. The substrate must also contain anions required (e.g., Cl- for amylase) and should not contain inhibiting cations or anions (e.g., Zn+2 and Mn+2 for CK)

The response of a sodium electrode to a 10-fold increase in sodium concentration should be: A. A 10-fold drop in potential B. An increase in potential of approximately 60 mV C. An increase in potential of approximately 10 mV D. A decrease in potential of approximately 10 mV

B. An increase in potential of approximately 60 mV B The Nernst equation predicts an increase of approximately 60 mV per 10-fold increase in sodium activity. For sodium: E = E° + RT/nF × 2.3 log10[Na+] RT/nF × 2.3 = 60 mV at 37°C. Therefore: E = E° + 60 mV × log10[Na+]. If sodium concentration is 10 mmol/L, then: E = E° + 60 mV × log10[10] = E° + 60 mV. If sodium concentration increases from 10 mmol/L to 100 mmol/L, then: E = E° + 60 mV × log10[100] = E° + 60 mV × 2 = E° + 120 mV.

Which of the following mechanisms accounts for the elevated plasma level of β lipoproteins seen in familial hypercholesterolemia (formerly type II hyperlipoproteinemia)? A. Hyperinsulinemia B. ApoB-100 receptor defect C. ApoC-II activated lipase deficiency D. ApoE3 deficiency

B. ApoB-100 receptor defect B The production of excess insulin leads to hypertriglyceridemia and is one mechanism responsible for familial endogenous hypertriglyceridemia. ApoC-II is an activator of lipoprotein lipase, and a homozygous deficiency results in high plasma chylomicrons and VLDL. ApoE3 deficiency is synonymous with inheritance of two apo-E2 alleles that lead to β dyslipoproteinemia. Familial hypercholesterolemia is inherited as an autosomal dominant trait. The classical form results from one of many mutations affecting the LDL receptor that cause it to have a lower affinity for LDL. A related hypercholesterolemia common in people of European ancestry results from a mutation of the apo-B100 gene that causes LDL to have a lower affinity for the LDL receptor. Together, they make familial hypercholesterolemia the most common inherited hyperlipoproteinemia with a frequency over 1:500.

Ion selective analyzers using undiluted samples have what advantage over analyzers that use a diluted sample? A. Can measure over a wider range of concentration B. Are not subject to pseudohyponatremia caused by high lipids C. Do not require temperature equilibration D. Require less maintenance

B. Are not subject to pseudohyponatremia caused by high lipids B Ion-selective analyzers measure the electrolyte dissolved in the fluid phase of the sample in millimoles per liter of plasma water. When undiluted blood is assayed, the measurement is independent of colloids such as protein and lipid. Hyperlipemic samples cause falsely low sodium measurements when assayed by flame photometry and ion-selective analyzers requiring dilution because lipids displace plasma water containing the electrolytes. One drawback to undiluted or direct measuring systems is that the electrodes require more frequent deproteinization and usually have a shorter duty cycle.

Cholesterol esterase is used in enzymatic assays to: A. Oxidize cholesterol to form peroxide B. Hydrolyze fatty acids bound to the third carbon atom of cholesterol C. Separate cholesterol from apoproteins A-I and A-II by hydrolysis D. Reduce NAD+ to NADH

B. Hydrolyze fatty acids bound to the third carbon atom of cholesterol B Approximately two-thirds of the serum cholesterolhas a fatty acid esterified to the hydroxyl group ofthe third carbon atom of the cholesterol molecule. Cholesterol esterase hydrolyzes fatty acids and is required because cholesterol oxidase cannot utilize esterified cholesterol as a substrate

Which of the following conditions will cause erroneous Cai results? Assume that the samples are collected and stored anaerobically, kept at 4°C until measurement, and stored for no longer than 1 hour. A. Slight hemolysis during venipuncture B. Assay of whole blood collected in sodium oxalate C. Analysis of serum in a barrier gel tube stored at 4°C until the clot has formed D. Analysis of whole blood collected in sodium heparin, 20 U/mL (low-heparin tube)

B. Assay of whole blood collected in sodium oxalate B Unlike Pi, the intracellular calcium level is not significantly different from plasma calcium, and calcium is not greatly affected by diet. Whole blood collected with 5-20 U/mL heparin and stored on ice no longer than 2 hours is the sample of choice for Cai. Blood gas syringes prefilled with 100 U/mL heparin should not be used because the high heparin concentration will cause low results. Citrate, oxalate, and ethylenediaminetetraacetic acid (EDTA) must not be used because they chelate calcium. Serum may be used provided that the sample is iced, kept capped while clotting, and assayed within 2 hours (barrier gel tubes may be stored longer).

Which of the following processes occurs when iron is in the oxidized (Fe3+) state? A. Absorption by intestinal epithelium B. Binding to transferrin and incorporation into ferritin C. Incorporation into protoporphyrin IX to form functional heme D. Reaction with chromogens in colorimetric assays

B. Binding to transferrin and incorporation into ferritin B Intestinal absorption occurs only if the iron is in the reduced (Fe+2) state. After absorption, Fe+2 is oxidized to Fe+3 by gut mucosal cells. Transferrin and ferritin bind iron efficiently only when in the oxidized state. Iron within Hgb binds to O2 by coordinate bonding, which occurs only if the iron is in the reduced state. Likewise, in colorimetric methods, Fe+2 forms coordinate bonds with carbon and nitrogen atoms of the chromogen.

Persistent noise from an ion selective electrode is most often caused by: A. Contamination of sample B. Blocked junction at the salt bridge C. Overrange from high concentration D. Improper calibration

B. Blocked junction at the salt bridge B Electrode noise most often results from an unstable junction potential. Most reference electrodes contain a high concentration of KCl internal solution used to produce the reference potential. This forms a salt bridge with the measuring half cell by contacting sample, but is kept from equilibrating via a barrier called a junction. When this junction becomes blocked by salt crystals, the reference potential will be unstable, resulting in fluctuation in the analyzer readout.

Which of the following statements about the aminotransferases (AST and ALT) is true? A. Isoenzymes of AST and ALT are not found in humans B. Both transfer an amino group to α-ketoglutarate C. Both require NADP+ as a coenzyme D. Both utilize four carbon amino acids as substrates

B. Both transfer an amino group to α-ketoglutarate B ALT catalyzes the transfer of an amino group from alanine, a three-carbon amino acid, to α-ketoglutarate (2 oxoglutarate), forming pyruvate. AST catalyzes the transfer of an amino group from aspartate (four carbons) to α-ketoglutarate, forming oxaloacetate. The reactions are highly reversible and regulate the flow of aspartate into the urea cycle. Both transaminases require P-5'-P as an intermediate amino acceptor (coenzyme). Cytoplasmic and mitochondrial isoenzymes are produced but are not differentiated in clinical practice

Which of the following dyes is the most specific for measurement of albumin? A. Bromcresol green (BCG) B. Bromcresol purple (BCP) C. Tetrabromosulfophthalein D. Tetrabromphenol blue

B. Bromcresol purple (BCP) B. Bromcresol purple (BCP) B Tetrabromphenol blue and tetrabromosulfophthalein are dyes that change pKa in the presence of protein. Although they have greater affinity for albumin than globulins, they are not sufficiently specific to apply to measurement of serum albumin. BCG and BCP are anionic dyes that undergo a spectral shift when they bind albumin at acid pH. BCP is more specific for albumin than BCG. Reaction of both dyes with globulins requires a longer incubation time than with albumin, and reaction times are kept at 30 seconds or less to increase specificity. Both dyes are free of interference from bilirubin. However, BCG is the method used most often. One reason for this is that renal dialysis patients produce an organic acid that competes with BCP for the binding site on albumin, causing a falsely low result

Which of the following tumor markers is used to monitor persons with breast cancer for recurrence of disease? A. Cathepsin-D B. CA-15-3 C. Retinoblastoma gene D. Estrogen receptor (ER)

B. CA-15-3 B CA-15-3 shares the same antigenic determinant as CA 27.29. Both are present on MUC1, a mucinous protein on the cell membrane of various tissues. The markers are used to monitor treatment and recurrence of breast cancer. However, abnormal plasma levels are seen in many nonmalignant conditions, and the test is not used for diagnostic purposes. CA-125 is a glycoprotein antigen shed by approximately 75% of ovarian cancers. It is an FDA-approved tumor marker for monitoring recurrence of ovarian cancer and evaluating the effectiveness of chemotherapy. Cathepsin-D and ER assays are performed to determine the prognosis of persons with breast cancer. Overexpression of cathepsin-D is associated with a higher relapse rate. Breast tissue that is negative for ER is poorly responsive to hormone suppression (tamoxifen) therapy. The retinoblastoma gene (RB) is a tumor-suppressor gene found to be missing in persons with retinoblastoma. Various mutations of the gene have been reported in breast, lung, bladder, and other cancers

Which method is considered the candidate reference method for triglyceride measurement? A. Glycerol kinase-ultraviolet B. CDC modification of van Handel and Zilversmit C. Hantzsch condensation D. Glycerol kinase coupled to peroxidase

B. CDC modification of van Handel and Zilversmit B Enzymatic methods for triglyceride measurement are widely used because they eliminate the need for extraction and saponification. However, they are subject to positive interference from endogenous glycerol and variations in the efficiency of lipase, which can result in under- or overestimation of triglycerides. The most accurate method for triglyceride assay is the nonenzymatic method based upon reaction of formaldehyde with chromotropic acid. In this method, extraction with silicic acid and chloroform separates triglycerides from lipoproteins, phospholipids, and glycerol. Saponification with alcoholic potassium hydroxide (KOH) produces glycerol, which is oxidized to formaldehyde by periodate. The formaldehyde reacts with chromotropic acid to form a pink product.

Which of the following enzymes is a transferase? A. ALP B. CK C. Amylase D. LD

B. CK B Enzymes are identified by a numeric system called the EC (Enzyme Commission) number. The first number refers to the class of the enzyme. There are six classes; in order, these are oxidoreductases, transferases, hydrolases, lyases, isomerases, and ligases. Dehydrogenases are oxidoreductases, whereas kinases and transaminases are transferases. CK is EC number 2.7.3.2, which distinguishes it from other kinases.

When preparing a patient for an oral glucose tolerance test (OGTT), which of the following conditions will lead to erroneous results? A. The patient remains ambulatory for 3 days prior to the test B. Carbohydrate intake is restricted to below 150 g/day for 3 days prior to test C. No food, coffee, tea, or smoking is allowed 8 hours before and during the test D. Administration of 75 g of glucose is given to an adult patient following a 10-12-hour fast

B. Carbohydrate intake is restricted to below 150 g/day for 3 days prior to test B Standardized OGTTs require that patients receive at least 150 grams of carbohydrate per day for 3 days prior to the test in order to stabilize the synthesis of inducible glycolytic enzymes. The 2-hour OGTT test is no longer recommended for screening and should be reserved for confirmation of diabetes in cases that are difficult to diagnose, such as persons who lack symptoms and signs of fasting hyperglycemia.

Uric acid is derived from the: A. Oxidation of proteins B. Catabolism of purines C. Oxidation of pyrimidines D. Reduction of catecholamines

B. Catabolism of purines B Uric acid is the principal product of purine (adenosine and guanosine) metabolism. Oxidation of proteins yields urea along with CO2, H2O, and inorganic acids Catecholamines are oxidized, forming vanillylmandelic acid (VMA) and homovanillic acid (HVA).

Which stationary phase is used for the measurement of hemoglobin A1c by high performance liquid chromatography? A. Octadecylsilane (C18) B. Cation exchanger C. Anion exchanger D. Polystyrene divinylbenzene

B. Cation exchanger B HPLC methods for measuring Hgb A1c are performed by diluting whole blood with an acid buffer that hemolyzes the sample. Normal hemoglobin A has a weak positive charge at an acidic pH and binds weakly to the resin. Glycated hemoglobin has an even weaker positive charge and is eluted before hemoglobin A. Abnormal hemoglobin molecules S, D, E, and C have a higher positive charge than hemoglobin A and are retained longer on the column. Elution is accomplished by increasing the ionic strength of the mobile phase. Cations in the buffer displace the hemoglobin pigments from the column.

The fragments typically produced and analyzed in methods employing mass spectroscopy are typically: A. Of low molecular size ranging from 10-100 daltons B. Cations caused by electron loss or proton attachment C. Anions caused by bombarding the molecule with an electron source D. Neutral species formed after excited molecules form a stable resonance structure

B. Cations caused by electron loss or proton attachment B In almost all MS applications, cations of the molecule are measured. Cations can be formed by various methods, the most common of which is electron bombardment (electron ionization). The energy transferred to the molecule causes ejection of an outer shell electron. MS can analyze sizes from trace metals through macromolecules. Proteins are measured following conversion to cations by ionization procedures such as matrix-assisted laser desorption ionization (MALDI) in which energy from a nitrogen laser causes transfer of a proton from the matrix (an acid) to the protein.

SITUATION: A patient has the following electrolyte results: Na = 130 mmol/L K = 4.8 mmol/L Cl = 105 mmol/L TCO2 = 26 mmol/L Assuming acceptable QC, select the best course of action. A. Report these results B. Check the albumin, total protein, Ca, P, and Mg results; if normal, repeat the sodium test C. Request a new sample D. Recalibrate and repeat the potassium test

B. Check the albumin, total protein, Ca, P, and Mg results; if normal, repeat the sodium test B The anion gap of this sample is < 4 mmol/L. This may result from laboratory error, retention of an unmeasured cation (e.g., calcium), or low level of unmeasured anion such as phosphorus or albumin. The sodium is inappropriately low for the chloride and bicarbonate and should be repeated if no biochemical cause is apparent.

SITUATION: A 2 2S QC error occurs for serum calcium by atomic absorption. Fresh standards prepared in 5.0% w/v albumin are found to be linear, but repeating the controls with fresh material does not improve the QC results. Select the most likely cause of this problem. A. Matrix effect caused by a viscosity difference between the standards and QC sera B. Chemical interference caused incomplete atomization C. Incomplete deconjugation of protein-bound calcium D. Ionization interference caused by excessive heat

B. Chemical interference caused incomplete atomization B Poor recovery of calcium by atomic absorption is often caused by failure to break thermostable bonds between calcium and phosphate (a form of chemical interference). This may be caused by failure to add lanthanum to the diluent or by low atomizer temperature. The use of 5.0 % w/v albumin in the calibrator produces viscosity and protein-binding characteristics similar to plasma, helping to eliminate matrix interference.

Which statement about iron methods is true? A. Interference from Hgb can be corrected by a serum blank B. Colorimetric methods measure binding of Fe2+ to a ligand such as ferrozine C. Atomic absorption is the method of choice for measurement of serum iron D. Serum iron can be measured by potentiometry

B. Colorimetric methods measure binding of Fe2+ to a ligand such as ferrozine B Atomic absorption is not the method of choice for serum iron because matrix error and variation of iron recovered by extraction cause bias and poor precision. Most methods use HCl to deconjugate Fe3+ from transferrin followed by reduction to Fe2+. This reacts with a neutral ligand such as ferrozine, tripyridyltriazine (TPTZ), or bathophenanthroline to give a blue complex. Anodic stripping voltammetry can also be used to measure serum iron. Hemolysis must be avoided because RBCs contain a much higher concentration of iron than does plasma.

In double immunodiffusion reactions, the precipitin band is: A. Invisible before the equivalence point is reached B. Concave to the protein of greatest molecular weight C. Closest to the well containing the highest level of antigen D. Located in an area of antibody excess

B. Concave to the protein of greatest molecular weight B In double immunodiffusion (Ouchterlony), the molecules of lower molecular weight move fastest through the gel, causing a visible precipitin arc when antigen and antibody approach equivalence. At equivalence the precipitin arc remains stationary. If the concentration of antisera is constant, the distance of the precipitin arc from the antigen well is proportional to antigen concentration.

SITUATION: Results of biochemistry tests are: Na = 138 mmol/L K = 4.2 mmol/L Cl = 94 mmol/L TCO2 = 20 mmol/L glucose = 100 mg/dL T bili = 1.2 mg/dL BUN = 6.8 mg/dL creat = 1.0 mg/dL albumin = 4.8 g/dL T protein = 5.1 g/dL What should be done next? A. Request a new specimen B. Repeat the total protein C. Repeat all tests D. Perform a protein electrophoresis

B. Repeat the total protein B All results are normal except total protein. The albumin level cannot be 94% of the total protein, and a random error in total protein measurement should be assumed.

Which of the following statements accurately characterizes the coulometric titration of chloride? A. The indicator electrodes generate voltage B. Constant current must be present across the generator electrodes C. Silver ions are formed at the generator cathode D. Chloride concentration is inversely proportional to titration time

B. Constant current must be present across the generator electrodes B The Cotlove chloridometer is based upon the principle of coulometric titration with amperometric detection. Charge in the form of silver ions is generated by oxidation of silver wire at the generator anode. Silver ions react with chloride ions, forming insoluble silver chloride (AgCl). When all of the chloride is titrated, free silver ions are detected by reduction back to elemental silver, which causes an increase in current across the indicator electrodes (a pair of silver electrodes with a voltage difference of about 1.0 V DC). Charge or titration time is directly proportional to chloride concentration as long as the rate of oxidation remains constant at the generator anode.

Upon which principle is the biuret method based? A. The reaction of phenolic groups with CuIISO4 B. Coordinate bonds between Cu+2 and carbonyl and imine groups of peptide bonds. C. The protein error of indicator effect producing color when dyes bind protein D. The reaction of phosphomolybdic acid with protein

B. Coordinate bonds between Cu+2 and carbonyl and imine groups of peptide bonds. B Biuret is a compound with two carbonyl groups and three amino groups and forms coordinate bonds with Cu+2 in the same manner as does protein. Therefore, proteins and peptides are both measured in the biuret reaction. The biuret reagent consists of an alkaline solution of copper II sulfate. Tartrate salts are added to keep the copper in solution and prevent turbidity. Potassium iodide prevents autoreduction of Cu+2.

A single-point calibration is performed between each blood gas sample in order to: A. Correct the electrode slope B. Correct electrode and instrument drift C. Compensate for temperature variance D. Prevent contamination by the previous sample

B. Correct electrode and instrument drift B Calibration using a single standard corrects the instrument for error at the labeled value of the calibrator but does not correct for analytic errors away from the set point. A two-point calibration adjusts the slope response of the electrode, eliminating proportional error caused by poor electrode performance.

Which of the following statements regarding adrenal cortical dysfunction is true? A. Patients with Cushing's syndrome usually have hyperkalemia B. Cushing's syndrome is associated with glucose intolerance C. Addison's disease is associated with hypernatremia D. Addison's disease is caused by elevated levels of cortisol

B. Cushing's syndrome is associated with glucose intolerance B Patients with Cushing's syndrome have elevated levels of cortisol and other adrenal corticosteroids. This causes the characteristic cushingoid appearance that includes obesity, acne, and humpback posture. Osteoporosis, hypertension, hypokalemia, and glycosuria are characteristics. Addison's disease results from adrenal hypoplasia and produces the opposite symptoms including hypotension, hyperkalemia, and hypoglycemia.

What substance may be measured as an alternative to creatinine for evaluating GFR? A. Plasma urea B. Cystatin C C. Uric acid D. Potassium

B. Cystatin C B Although all of the analytes listed are increased in chronic kidney disease as a result of low GFR, potassium, urea, and uric acid may be increased by other mechanisms and therefore, they are not specific for glomerular function. Cystatin C is an inhibitor of cysteine proteases. Being only 13 kilodaltons, it is completely filtered by the glomerulus then reabsorbed by the tubules. The plasma level is highly correlated to GFR because little is eliminated by nonrenal routes. Plasma levels are not influenced by diet, age, gender, or nutritional status. Low GFR causes retention of cystatin C in plasma and levels become abnormally high at clearance rates below 90 mL/min, making the test more sensitive than creatinine.

What is the first day in the second half of the month that patient results would be rejected? A. Day 16 B. Day 17 C. Day 18 D. Day 19

B. Day 17 B The 41s rule is broken across QC levels on day 17. This means that four consecutive controls are greater than ±1s from the mean. QC rules that are sensitive to SE are applied across both runs and levels to increase the probability of error detection. These are 22s , 41s, and 10×.

Which set of results is most likely in a female with hypogonadotropic ovarian failure? A. Increased LH, FSH, and estrogen B. Decreased LH, FSH, and estrogen C. Decreased prolactin and estrogen D. Increased LH and FSH, and decreased estrogen

B. Decreased LH, FSH, and estrogen B Hypogonadotropic ovarian failure is the result of pituitary dysfunction. It may be caused by low levels of both LH and FSH, or it may be caused by high levels of prolactin as in prolactinoma, since prolactin will inhibit LHRH and result in low LH and FSH

Which of the following statements regarding the diagnosis of pancreatitis is correct? A. Amylase and lipase are as predictive in chronic as in acute pancreatitis B. Diagnostic sensitivity is increased by assaying both amylase and lipase C. Measuring the urinary amylase:creatinine ratio is useful only when patients have renal failure D. Serum lipase peaks several hours before amylase after an episode of acute pancreatitis

B. Diagnostic sensitivity is increased by assaying both amylase and lipase B Amylase is not increased in all persons with pancreatitis and can be increased in several nonpancreatic conditions. Lipase adds both sensitivity and specificity to the diagnosis of acute pancreatitis. Plasma or serum lipase becomes abnormal within 6 hours, peaks at approximately 24 hours, and remains abnormal for about 1 week following an episode of acute pancreatitis. In acute pancreatitis, the rate of urinary amylase excretion increases, and the amylase:creatinine clearance ratio is helpful in diagnosing some cases of pancreatitis. The normal A:C clearance ratio is 1%-4%. In acute pancreatitis, the ratio is usually above 4% and can be as high as 15%. In chronic pancreatitis, acinar cell degeneration often occurs, resulting in loss of amylase and lipase production. This lowers the sensitivity of amylase and lipase in detecting chronic disease to below 50%. Patients with chronic disease have pancreatic insufficiency giving rise to increased fecal fat, and decreased pancreatic digestive enzymes such as trypsin, chymotrypsin, and elastin.

SITUATION: A serum osmolality measured in the emergency department is 326 mOsm/kg. Two hours later, chemistry results are: Na = 135 mmol/L BUN = 18 mg/dL glucose = 72 mg/dL measured osmolality = 318 mOsm/kg What do these results suggest? A. Laboratory error in electrolyte or glucose measurement B. Drug or alcohol intoxication C. Specimen misidentification D. Successful rehydration of the patient

B. Drug or alcohol intoxication B The osmolal gap is the difference between calculated and measured osmolality. Here, the osmolal gap is 38 mOsm/kg. When the osmolal gap is greater than 10 mOsm/kg, an unmeasured solute is present or an analytical error occurred when measuring the osmolality, electrolytes, urea, or glucose. The reference range for serum osmolality is 280-295 mOsm/kg. Both osmolality measurements are above the URL. These results point to the presence of an unmeasured solute. A significant osmolal gap in samples from emergency department patients usually results from alcohol or drug consumption. The difference in osmolality between the two samples is 8 mOsm/kg and can be explained by alcohol metabolism during the 2 hours between samples

Select the main estrogen produced by the ovaries and used to evaluate ovarian function. A. Estriol (E3 ) B. Estradiol (E2 ) C. Epiestriol D. Hydroxyestrone

B. Estradiol (E2) B E2 is the major estrogen produced by the ovaries and gives rise to both estrone (E1) and E3. E2 is used to evaluate both ovarian function and menstrual cycle dysfunction

When establishing QC limits, which of the following practices is inappropriate? A. Using last month's QC data to determine current target limits B. Exclusion of any QC results greater than ±2s from the mean C. Using control results from all shifts on which the assay is performed D. Using limits determined by reference laboratories using the same method

B. Exclusion of any QC results greater than ±2s from the mean B Data between ±2 and ±3s must be included in calculations of the next month's acceptable range. Elimination of these values would continuously reduce the distribution of QC results, making "out-of-control" situations a frequent occurrence. Generally, QC results greater than 3s are not used to calculate next month's mean.

In hepatitis, the rise in serum conjugated bilirubin can be caused by: A. Secondary renal insufficiency B. Failure of the enterohepatic circulation C. Enzymatic conversion of urobilinogen to bilirubin D. Extrahepatic conjugation

B. Failure of the enterohepatic circulation B Conjugated bilirubin is increased in hepatitis and other causes of hepatic necrosis due to failure to re-excrete conjugated bilirubin reabsorbed from the intestine. Increased direct bilirubin can also be attributed to accompanying intrahepatic obstruction, which blocks the flow of bile.

Which of the following situations is likely to cause an error when weighing with an electronic analytical balance? A. Failure to keep the knife edge clean B. Failure to close the doors of the balance before reading the weight C. Oxidation on the surface of the substitution weights D. Using the balance without allowing it to warm up for at least 10 minutes

B. Failure to close the doors of the balance before reading the weight B Electronic balances do not use substitution weights or knife edges to balance the weight on the pan. Instead, they measure the displacement of the pan by the weight on it using electromagnetic force to return it to its reference position. Regardless of the type of balance used, all need to be located on a firm weighing table free of vibration. Doors must be closed to prevent air currents from influencing the weighing, and the pan and platform must be clean and free of dust and chemical residue

The most commonly used detector for clinical gas-liquid chromatography (GLC) is based upon: A. Ultraviolet light absorbance at 254 nm B. Flame ionization C. Refractive index D. Thermal conductance

B. Flame ionization B Volatile solutes can be detected in GLC using flame ionization, thermal conductivity, electron capture, and mass spectroscopy. In flame ionization, energy from a flame is used to excite the analytes as they elute from the column. The flame is made by igniting a mixture of hydrogen, carrier gas, and air. Current is produced when an outer shell electron is ejected from the excited analyte

Which of the following protein methods has the highest analytical sensitivity? A. Refractometry B. Folin-Lowry C. Turbidimetry D. Direct ultraviolet absorption

B. Folin-Lowry B The Folin-Lowry (Lowry's) method uses both biuret reagent and phosphotungstic and molybdic acids to oxidize the aromatic side groups on proteins. The acids oxidize the phenolic rings of tyrosine and tryptophan. These, in turn, reduce the Cu+2 in the biuret reagent, increasing sensitivity about 100-fold

Which statement about EMIT is true? A. Enzyme activity is inversely proportional to drug level B. Formation of NADH is monitored at 340 nm C. ALP is the commonly used conjugate D. Assay use is restricted to serum

B. Formation of NADH is monitored at 340 nm B EMIT is a homogenous immunoassay, meaning that free antigen does not have to be separated from bound antigen. Most EMIT assays use a two-reagent system. Reagent A contains substrate (usually glucose-6-PO4), coenzyme (NAD+), and antibody to the drug. Reagent B contains enzyme-labeled drug (usually G-6-PD-drug) and buffer. The rate of NADH production is proportional to the drug concentration. EMIT assays are commonly used to test for drugs of abuse in urine. In such cases, the enzyme activity of the low calibrator (drug concentration equal to U.S. Substance Abuse and Mental Health Services Administration minimum for a positive test) is used as the cutoff.

Which statement regarding creatinine is true? A. Serum levels are elevated in early renal disease B. High serum levels result from reduced glomerular filtration C. Serum creatine has the same diagnostic utility as serum creatinine D. Serum creatinine is a more sensitive measure of renal function than creatinine clearance

B. High serum levels result from reduced glomerular filtration B Serum creatinine is a specific but not a sensitive measure of glomerular function. About 60% of the filtration capacity of the kidneys is lost when serum creatinine becomes elevated. Because urine creatinine diminishes as serum creatinine increases in renal disease, the creatinine clearance is more sensitive than serum creatinine in detecting glomerular disease. A creatinine clearance below 60 mL/min indicates loss of about 50% functional nephron capacity and is classified as moderate (stage 3) chronic kidney disease.

Which of the following statements regarding PSA is true? A. Complexed PSA in plasma is normally less than free PSA B. Free PSA below 25% is associated with malignant disease C. A total PSA below 4 ng/mL rules out malignant disease D. A total PSA above 10 ng/mL is diagnostic of malignant disease

B. Free PSA below 25% is associated with malignant disease B In normal plasma, 55%-95% of the PSA is bound to protease inhibitors, primarily α1-antichymotrypsin, and the remainder is called free PSA. At a cutoff of 4 ng/mL commonly used for the upper reference limit, total PSA has a sensitivity of approximately 60%, and 22% of men with a PSA below 4 ng/mL have evidence of early prostate cancer on biopsy. For this reason, some laboratories prefer a cutoff of 2.5 ng/mL for total PSA. However, based upon this cutoff alone, the number of false-positive findings (unnecessary biopsies) would be extremely high. A PSA of 2.6 ng/mL that was 2.6 ng/mL the previous year would not likely be significant; however, a PSA of 2.6 ng/mL that was only 1.6 ng/mL the previous year would warrant further testing. In persons with a total PSA between 2.6 and 10.0 ng/mL, a low ratio of free PSA:total PSA (<25% fPSA) or a high level of complexed PSA increases the diagnostic sensitivity and specificity. Persons with a PSA between 2.6 and 10.0 ng/mL are selected for biopsy if either the fPSA is low or the cPSA is high. Initial studies also indicate that the incomplete cleavage of the proenzyme of PSA (proPSA) in persons with cancer results in a high ratio of proPSA to fPSA. This ratio was reported to have better diagnostic sensitivity and specificity than the percentage of fPSA alone. The probability of cancer when the total PSA is higher than 10 ng/mL is approximately 50%, and this necessitates a biopsy to determine if the prostate is malignant.

Which enzyme is measured in whole blood? A. Chymotrypsin B. Glucose-6-phosphate dehydrogenase C. Glycogen phosphorylase D. Lipase

B. Glucose-6-phosphate dehydrogenase B Glucose-6-phosphate dehydrogenase deficiency is the most common inherited RBC enzyme deficiency and is X linked. The enzyme is measured on a wholeblood hemolysate using glucose-6 phosphate as the substrate, and forms 6-phosphogluconate as NADP+ is converted to NADPH. Persons with the deficiency are prone to a hemolytic episode upon exposure to certain oxidative drugs and fava beans and as a result of infections. Heinz bodies from in the RBCs, bite cells are seen in the circulation, and plasma haptoglobin is reduced in severe cases.

In the ultraviolet enzymatic method for BUN, the urease reaction is coupled to a second enzymatic reaction using: A. AST B. Glutamate dehydrogenase C. Glutamine synthetase D. Alanine aminotransferase (ALT)

B. Glutamate dehydrogenase B BUN is most frequently measured by the urease-UV method in which the urease reaction is coupled to the glutamate dehydrogenase reaction, generating NAD+. Urea + H2O Urease 2NH3 + CO2 2-Oxoglutarate + NH3 + NADH + H+ GLD Glutamate + NAD+ + H2O When the urease reaction is performed under first-order conditions, the decrease in absorbance at 340 nm is proportional to the urea concentration

Which of the following hormones promotes hyperglycemia? A. Calcitonin B. Growth hormone C. Aldosterone D. Renin

B. Growth hormone B Growth hormone and cortisol promote gluconeogenesis and epinephrine stimulates glycogenolysis. Excess thyroid hormone causes hyperglycemia by increasing glucagon and inactivation of insulin, thereby promoting both gluconeogenesis and glycogenolysis. An increase in any of these hormones can cause hyperglycemia. Calcitonin opposes the action of parathyroid hormone. Aldosterone is the primary mineralocorticoid hormone and stimulates sodium reabsorption and potassium secretion by the kidneys. Renin is released from the kidney due to ineffective arterial pressure and promotes activation of angiotensinogen and aldosterone secretion.

Which of the following is a potential source of error in the hexokinase method? A. Galactosemia B. Hemolysis C. Sample collected in fluoride D. Ascorbic acid

B. Hemolysis B The hexokinase method can be performed on serum or plasma using heparin, EDTA, citrate, or oxalate. RBCs contain glucose-6-PO4 and intracellular enzymes that generate NADH, causing positiveinterference. Therefore, hemolyzed samples require a serum blank correction (subtraction of the reaction rate with hexokinase omitted from the reagent).

Which of the following is the reference method for measuring serum glucose? A. Somogyi-Nelson B. Hexokinase C. Glucose oxidase D. Glucose dehydrogenase

B. Hexokinase B The hexokinase method is considered more accurate than glucose oxidase methods because the coupling reaction using glucose-6-phosphate dehydrogenase (G-6-PD) is highly specific. The hexokinase method may be done on serum or plasma collected using heparin, EDTA, fluoride, oxalate, or citrate. The method can also be used for urine, cerebrospinal fluid, and serous fluids.

Which of the following abnormal types of Hgb migrates to the same position as Hgb S on agarose or cellulose acetate at pH 8.6? A. Hgb C B. Hgb DPunjab C. Hgb OArab D. Hgb E

B. Hgb DPunjab B Hgb DPunjab migrates with Hgb S on cellulose acetate or agarose at pH 8.6-9.2. Hgb C, E, OArab, and CHarlem migrate to the same position as Hgb A2 on cellulose acetate or agarose at pH 8.6-9.2. Hgb S may be differentiated from Hgb DPunjab using citrate (acid) agar at pH 6.2. Using this technique, Hgb S migrates further toward the anode than Hgb DPunjab

Which Hgb is a β-δ chain hybrid and migrates to the same position as Hgb S at pH 8.6? A. Hgb CHarlem B. HgbLepore C. Hgb GPhiladelphia D. Hgb DPunjab

B. HgbLepore B HgbLepore results from translocation of β and δ globin genes, resulting in a polypeptide chain that migrates midway between Hgb A2 and Hgb A. The chain is transcribed more slowly than the β polypeptide chain, causing the quantity of HgbLepore to be less than 15%. HgbLepore is suspected when Hgb migrating in the "S" zone comprises less than 20% of the total Hgb. In Hgb S trait, the AS phenotype produces 20%-40% Hgb S.

Which statement best describes the clinical utility of plasma homocysteine? A. Levels are directly related to the quantity of LDL cholesterol in plasma B. High plasma levels are associated with atherosclerosis and increased risk of thrombosis C. Persons who have an elevated plasma homocysteine will also have an increased plasma Lp(a) D. Plasma levels are increased only when there is an inborn error of amino acid metabolism

B. High plasma levels are associated with atherosclerosis and increased risk of thrombosis B Homocysteine includes the monomeric amino acid as well as the dimers such as homocystine that contain homocysteine. Plasma levels are measured as an independent risk factor for coronary artery disease. High levels of homocysteine are toxic to vascular endothelium and promote inflammation and plaque formation. Plasma levels are independent of LDL and other cholesterol fractions and help explain why approximately 35% of people with first-time AMI have LDL cholesterol levels < 130 mg/dL

Which of the following is the primary mechanism for vasopressin (ADH) release? A. Hypovolemia B. Hyperosmolar plasma C. Renin release D. Reduced renal blood flow

B. Hyperosmolar plasma B ADH is released by the posterior pituitary in response to increased plasma osmolality. Normally, this is triggered by release of aldosterone caused by ineffective arterial pressure in the kidney. Aldosterone causes sodium reabsorption, which raises plasma osmolality; release of ADH causes reabsorption of water, which increases blood volume and restores normal osmolality. A deficiency of ADH (diabetes insipidus) results in dehydration and hypernatremia. An excess of ADH (syndrome of inappropriate ADH release [SIADH]) results in dilutional hyponatremia. This may be caused by regional hypovolemia, hypothyroidism, central nervous system injury, drugs, and malignancy.

Electrophoretic movement of proteins toward the anode will decrease by increasing the: A. Buffer pH B. Ionic strength of the buffer C. Current D. Voltage

B. Ionic strength of the buffer B Electrophoresis is the migration of charged molecules in an electric field. Increasing the strength of the field by increasing voltage (or current) increases migration. However, increasing ionic strength decreases the migration of proteins. Counterions (cations) in the buffer move with the proteins, reducing their electromagnetic attraction for the anode.

The freezing point osmometer differs from the vapor pressure osmometer in that only the freezing point osmometer: A. Cools the sample B. Is sensitive to ethanol C. Requires a thermoelectric module D. Requires calibration with aqueous standards

B. Is sensitive to ethanol B Alcohol enters the vapor phase so rapidly that it evaporates before the dew point of the sample is reached. Therefore, ethanol does not contribute to osmolality as measured using the vapor pressure osmometer. Freezing-point osmometers measure alcohol and can be used in emergency department settings to estimate ethanol toxicity.

Creatinine is considered the substance of choice to measure endogenous renal clearance because: A. The rate of formation per day is independent of body size B. It is completely filtered by the glomeruli C. Plasma levels are highly dependent upon diet D. Clearance is the same for both men and women

B. It is completely filtered by the glomeruli B Creatinine concentration is dependent upon muscle mass, but varies by less than 15% per day. Creatinine is not metabolized by the liver, or dependent on diet, and is 100% filtered by the glomeruli. It is not reabsorbed significantly but is secreted slightly, especially when filtrate flow is slow. Plasma creatinine and cystatin C are the two substances of choice for evaluating the glomerular filtration rate (GFR).

Why is vacuum necessary in the mass filter of a mass spectrometer? A. Ionization does not occur at atmospheric pressure B. It prevents collision between fragments C. It removes electrons from the ion source D. It prevents contamination

B. It prevents collision between fragments B Vacuum is needed in the mass filter of the MS to prevent random collisions between ions that would alter their trajectory or time of flight. It is also needed in CG-MS instruments that use electron ionization. The vacuum prevents collision between the carrier gas molecules and the ions. In spectrometers that use electrospray ionization, chemical ionization, and laser desorption ionization (MALDI and SELDI TOF), the ion source is not under vacuum.

Which of the following statements regarding the Philadelphia chromosome is true? A. It is seen exclusively in chronic myelogenous leukemia B. It results from a translocation C. It appears as a short-arm deletion of chromosome 21 D. It is associated with a poor prognosis

B. It results from a translocation B The Philadelphia chromosome (Ph1) is formed by translocation of the long arms of chromosomes 9 and 22. The result is that part of the ABL gene of chromosome 9 becomes inserted into the BCR gene of chromosome 22. The ABL gene is an oncogene and the product of the hybrid gene is a tyrosine kinase that signals cell proliferation. The Ph1 chromosome appears on karyotyping as a long-arm deletion of chromosome 22 because only the terminal end of the long arm of chromosome 9 is exchanged for most of the long arm of chromosome 22. The BCR/ABL translocation can be detected using FISH hybridization probes. Approximately 95% of persons with chronic myelogenous leukemia have the Ph1 chromosome. Those patients who do not demonstrate Ph1 have a poorer prognosis. It is also present in the lymphocytes of up to 25% of adults with acute lymphocytic leukemia (ALL) and in a small number of children with ALL and persons with acute myelogenous leukemia

Which statement best characterizes serum bilirubin levels in the first week following delivery? A. Serum bilirubin 24 hours after delivery should not exceed the upper reference limit for adults B. Jaundice is usually first seen 48-72 hours postpartum in neonatal hyperbilirubinemia C. Serum bilirubin above 5.0 mg/dL occurring 2-5 days after delivery indicates hemolytic or hepatic disease D. Conjugated bilirubin accounts for about 50% of the total bilirubin in neonates

B. Jaundice is usually first seen 48-72 hours postpartum in neonatal hyperbilirubinemia B Bilirubin levels may reach as high as 2-3 mg/dL in the first 24 hours after birth owing to the trauma of delivery, such as resorption of a subdural hematoma. Neonatal hyperbilirubinemia occurs 2-3 days after birth due to increased hemolysis at birth and transient deficiency of the microsomal enzyme, UDP-glucuronyl transferase. Normally, levels rise to about 5-10 mg/dL but may be greater than 15 mg/dL, requiring therapy with UV light to photo-oxidize the bilirubin. Neonatal jaundice can last up to 1 week in a mature neonate and up to 2 weeks in prematures babies. Neonatal bilirubin is almost exclusively unconjugated

What is the purpose of LD in the kinetic method of Henry for AST? A. Forms NADH, enabling the reaction to be monitored at 340 nm B. Rapidly exhausts endogenous pyruvate in the lag phase C. Reduces oxaloacetate, preventing product inhibition D. Generates lactate, which activates AST

B. Rapidly exhausts endogenous pyruvate in the lag phase B Patients with liver disease often have high levels of pyruvate and LD. The LD can catalyze the reaction of pyruvate with NADH in the substrate, forming NAD+ and lactate. This would give a falsely high rate for AST because NAD+ is the product measured. Adding LD to the substrate causes pyruvate to be depleted in the first 30 seconds, before AST and MD reactions reach steady state

An HPLC assay for procainamide gives an internal standard peak that is 15% greater in area and height for sample 1 than sample 2. The technologist should suspect that: A. The column pressure increased while sample 2 was being analyzed B. Less recovery from sample 2 occurred in the extraction step C. The pH of the mobile phase increased during chromatography of sample 2 D. There was more procainamide in sample 1 than sample 2

B. Less recovery from sample 2 occurred in the extraction step B The internal standard compensates for variation in extraction, evaporation, reconstitution, and injection volume. The same amount of internal standard is added to all samples and standards prior to assay. Increased column pH or pressure usually alters retention time, and may not affect peak quantitation.

Which statement best describes the clinical utility of plasma or serum myoglobin? A. Levels greater than 100 µg/L are diagnostic of AMI B. Levels below 100 µg/L on admission and 2-4 hours postadmission help to exclude a diagnosis of AMI C. Myoglobin peaks after the cardiac troponins but is more sensitive D. The persistence of myoglobin > 110 µg/L for 3 days following chest pain favors a diagnosis of AMI

B. Levels below 100 µg/L on admission and 2-4 hours postadmission help to exclude a diagnosis of AMI B Myoglobin is a heme-containing pigment in both skeletal and cardiac muscle cells. The upper limit of normal is approximately 90 μg/L for males and 75 μg/L for females. The plasma myoglobin is a sensitive marker for AMI. Over 95% of affected persons have a value higher than the cutoff (typically >110 μg/L). However, specificity is approximately 75%-85% owing to skeletal muscle injury or renal insufficiency. For this reason, a plasma myoglobin below the cutoff on admission, and within the first 3 hours following chest pain helps to rule out AMI. A value above the cutoff must be confirmed using a cardiac specific assay such as TnI or TnT.

Select the anticoagulant of choice for blood gas studies. A. Sodium citrate 3.2% B. Lithium heparin 100 U/mL blood C. Sodium citrate 3.8% D. Ammonium oxalate 5.0%

B. Lithium heparin 100 U/mL blood B Heparin is the only anticoagulant that does not alter the pH of blood; heparin salts must be used for pH and blood gases. Solutions of heparin are air equilibrated and must be used sparingly to prevent contamination of the sample by gas in the solution.

Which form of jaundice occurs within days of delivery and usually lasts 1-3 weeks, but is not due to normal neonatal hyperbilirubinemia or hemolytic disease of the newborn? A. Gilbert syndrome B. Lucey -Driscoll syndrome C. Rotor syndrome D. Dubin-Johnson syndrome

B. Lucey -Driscoll syndrome B Lucey-Driscoll syndrome is a rare form of jaundice caused by unconjugated bilirubin that presents within 2-4 days of birth and can last several weeks. It is caused by an inhibitor of UDP-glucuronyl transferase in maternal plasma that crosses the placenta. Jaundice is usually severe enough to require treatment.

Select the coupling enzyme used in the kinetic AST reaction of Henry. A. LD B. Malate dehydrogenase C. Glutamate dehydrogenase D. G-6-PD

B. Malate dehydrogenase B The method of Henry for AST uses malate dehydrogenase (MD) to reduce oxaloacetate to malate. The electrons come from NADH forming NAD+. Aspartate + α-ketoglutarate AST Oxaloacetate + Glutamate Oxaloacetate + NADH + H+ MD Malate + NAD+

Interference in atomic absorption spectrophotometry caused by differences in viscosity is called: A. Absorption interference B. Matrix effect C. Ionization interference D. Quenching

B. Matrix effect B Significant differences in aspiration and atomization result when the matrix of sample and unknowns differ. Differences in viscosity and protein content are major causes of matrix error. Matrix effects can be reduced by using protein-based calibrators and diluting both standards and samples prior to assay.

Which statement best describes the use of the Hgb A1C test? Peak / Calibrated % Area / % Area / Retention Time / Peak Area Alb / 0.60/ 0.25/ 12500 F/ 0.50/ 0.50/ 11300 LA1c/ 0.75/ 0.70/ 15545 A1c/ 6.2/ 0.90/ 45112 P3/ 2.6 /1.60 /57489 Ao /48.0/ 1.8/ 994813 C/ 43.0 /2.00/ 926745 A. Should be used for monitoring glucose control only B. May be used for both diagnosis and monitoring C. Should be used only to monitor persons with type 1 diabetes D. May be used only to monitor persons with type 2 diabetes

B. May be used for both diagnosis and monitoring B The ADA now recommends that the hemoglobin A1c test be used for both diagnosis and monitoring blood glucose levels. The cutpoint for diabetes is an A1c of 6.5. Persons with an A1c of 5.7%-6.4% are classified as being at high risk for diabetes within 5 years. An A1c between 4.0%-5.5% is defined as within normal limits

A laboratory is establishing a reference range for a new analyte and wants the range to be determined by the regional population of adults age 18 and older. The analyte concentration is known to be independent of race and gender. Which is the most appropriate process to follow? A. Determine the mean and standard deviation of the analyte from 40 healthy adults and calculate the ±2s limit B. Measure the analyte in 120 healthy adults and calculate the central 95th percentile C. Measure the analyte in 120 healthy adults and use the lowest and highest as the reference range limits D. Measure the analyte in 60 healthy adults and 60 adults with conditions that affect the analyte concentration; calculate the concentration of least overlap

B. Measure the analyte in 120 healthy adults and calculate the central 95th percentile B Since the concentration of an analyte may not be normally distributed in a population, the reference range should not be determined from the standard deviation. It is more appropriate to determine the central 95th percentile (the range that encompasses 95% of the results). A minimum of 120 samples is needed for statistical significance. Results are rank ordered from lowest to highest. The 3rd result is the lowest value and the 118th is the highest value in the reference range. The laboratory can verify a preexisting reference range (e.g., as determined by the manufacturer's study) by testing 20 healthy persons. If no more than 10% fall outside the range, it can be considered valid for the patient population.

When a therapeutic drug is suspected of causing toxicity, which specimen is the most appropriate for an initial investigation? A. Trough blood sample B. Peak blood sample C. Urine at the time of symptoms D. Gastric fluid at the time of symptoms

B. Peak blood sample B When a drug is suspected of toxicity, the peak blood sample (sample after absorption and distribution are complete) should be obtained because it is most likely to exceed the therapeutic limit. If the peak level is above the upper therapeutic limit, then toxicity is confirmed, and the drug dose is lowered. If the peak drug concentration is within the therapeutic range, toxicity is less likely, but cannot be ruled out. A high concentration of free drug, the presence of active metabolites, and abnormal response to the drug are causes of drug toxicity that may occur when the blood drug level is within the published therapeutic range.

Which method is most often used to measure fractionated catecholamines (epinephrine, norepinephrine, and dopamine)? A. Measurement of fluorescence following oxidation by potassium ferricyanide B. Measurement by HPLC with electrochemical detection C. Measure of radioactivity after conversion by catechol-O-methyltransferase (COMT) to tritiated metanephrines D. Measurement by HPLC with fluorescence detection

B. Measurement by HPLC with electrochemical detection B HPLC-ECD separates catecholamines by reversephase chromatography, then detects them by oxidizing the aromatic ring at +0.8 V to a quinone ring. Current is proportional to epinephrine and norepinephrine concentration. Fluorescent methods employing ferricyanide (trihydroxyindole method) or ethylenediamine (EDA method) show interference by Aldomet and several other drugs and are obsolete. The radioenzymatic assay of catecholamines is a specific alternative to HPLC but requires a liquid scintillation counter. The method uses the enzyme COMT to transfer a tritiated methyl group from S adenosyl methionine to the catecholamines. This results in formation of radiolabeled metanephrines that are measured. HPLC with fluorescence detection is not as sensitive as HPLC-ECD. Electrospray ionization tandem-mass spectroscopy is an alternative to HPLC-EDC for measurement of fractionated catecholamines and metanephrines.

In which condition is the measurement of acid phosphatase clinically useful? A. Measuring the prostatic isoenzyme to screen for prostate cancer B. Measuring the enzyme in a vaginal swab extract C. The diagnosis of hemolytic anemia D. As a marker for bone regeneration

B. Measuring the enzyme in a vaginal swab extract B The PSA test is clinically more sensitive than prostatic acid phosphatase in detecting prostatic cancer. The clinical use of prostatic acid phosphatase is confined to the investigation of sexual assault. Acid phosphatase activity > 50 IU/L establishes the presence of seminal fluid in the vaginal sample. Tartrate-resistant acid phosphatase is used as a cytochemical marker for hairy-cell leukemia, and may be measured in serum to identify diseases with increased osteoclast activity, particularly malignancies involving bone

A patient's blood gas results are: pH = 7.50 PCO2 = 55 mm Hg HCO3 - = 40 mmol/L These results indicate: A. Respiratory acidosis B. Metabolic alkalosis C. Respiratory alkalosis D. Metabolic acidosis

B. Metabolic alkalosis B A pH above 7.45 corresponds with alkalosis. Both bicarbonate and PCO2 are elevated. Bicarbonate is the conjugate base and is under metabolic (renal) control, while PCO2 is an acid and is under respiratory control. Increased bicarbonate (but not increased CO2) results in alkalosis; therefore, the classification is metabolic alkalosis, partially compensated by increased PCO2.

Which of the following processes is part of the normal metabolism of bilirubin? A. Both conjugated and unconjugated bilirubin are excreted into the bile B. Methene bridges of bilirubin are reduced by intestinal bacteria forming urobilinogens C. Most of the bilirubin delivered into the intestine is reabsorbed D. Bilirubin and urobilinogen reabsorbed from the intestine are mainly excreted by the kidneys

B. Methene bridges of bilirubin are reduced by intestinal bacteria forming urobilinogens B Most of the conjugated bilirubin delivered into the intestine is deconjugated by β-glucuronidase and then reduced by intestinal flora to form three different reduction products collectively called urobilinogens. The majority of bilirubin and urobilinogen in the intestine are not reabsorbed. Most of that which is reabsorbed is re-excreted by the liver. The portal vein delivers blood from the bowel to the sinusoids. Hepatocytes take up about 90% of the returned bile pigments and secrete them again into the bile. This process is called the enterohepatic circulation

Which statement about colorimetric bilirubin methods is true? A. Direct bilirubin must react with diazo reagent under alkaline conditions B. Most methods are based upon reaction with diazotized sulfanilic acid C. Ascorbic acid can be used to eliminate interference caused by Hgb D. The color of the azobilirubin product is independent of pH

B. Most methods are based upon reaction with diazotized sulfanilic acid B Unconjugated bilirubin is poorly soluble in acid, and therefore, direct bilirubin is assayed using diazotized sulfanilic acid diluted in weak HCl. The direct diazo reaction should be measured after no longer than 3 minutes to prevent reaction of unconjugated bilirubin, or the diazo group can be reduced using ascorbate or hydroxylamine preventing any further reaction.

Urinary HVA is most often assayed to detect: A. Pheochromocytoma B. Neuroblastoma C. Adrenal medullary carcinoma D. Psychiatric disorders such as manic depression

B. Neuroblastoma B HVA is the major metabolite of dopa, and urinary HVA is elevated in more than 75% of neuroblastoma patients. Neuroblastomas also usually produce VMA from norepinephrine. Thus, HVA and VMA are assayed together and this increases the sensitivity of detection to around 90%.

Select the products formed from the forward reaction of AST. A. Alanine and α-ketoglutarate B. Oxaloacetate and glutamate C. Aspartate and glutamine D. Glutamate and NADH

B. Oxaloacetate and glutamate B AST forms oxaloacetate and glutamate from aspartate and α-ketoglutarate (2-oxoglutarate). Both transaminases use α ketoglutarate and glutamate as a common sub trate and product pair. Both aspartate and alanine can be used to generate glutamate in the central nervous system, where it acts as a neurotransmitter.

Which element is reduced at the cathode of a Clark polarographic electrode? A. Silver B. Oxygen C. Chloride D. Potassium

B. Oxygen B The Clark electrode is designed to measure oxygen. O2 diffuses through a gas-permeable membrane covering the electrode. It is reduced at the cathode, which is made of platinum or other inert metal. Electrons are supplied by the anode, which is made of silver. The net reaction is: 4 KCl + 2 H2O + O2 + 4 Ag° →4 AgCl + 4 KOH

In uncompensated metabolic acidosis, which of the following will be normal? A. Plasma bicarbonate B. PCO2 C. p50 D. Total CO2

B. PCO2 B The normal compensatory mechanism for metabolic acidosis is respiratory hyperventilation. In uncompensated cases, the PCO2 is not reduced, indicating a concomitant problem in respiratory control.

SITUATION: A lipemic specimen collected from an adult after a 12-hour fast was assayed for total cholesterol, triglycerides, and HDL cholesterol using a direct HDL method. Following are the results: Total cholesterol = 220 mg/dL HDL cholesterol = 40 mg/dL Triglyceride = 420 mg/dL The physician requests an LDL cholesterol assay after receiving the results. How should the LDL cholesterol be determined? A. Dilute the specimen 1:10 and repeat all tests; calculate LDL cholesterol using the Friedewald equation B. Perform a direct LDL cholesterol assay C. Ultracentrifuge the sample and repeat the HDL cholesterol on the infranate. Use the new result to calculate the LDL cholesterol D. Repeat the HDL cholesterol using the manganese heparin precipitation method. Use the new result to calculate the LDL cholesterol

B. Perform a direct LDL cholesterol assay B An accurate LDL cholesterol can be reported, if thedirect (detergent) method for LDL cholesterol is employed. These methods are not subject to interference by triglycerides at a concentration below 700 mg/dL.

In peroxidase-coupled glucose methods, which reagent complexes with the chromogen? A. Nitroprusside B. Phenol C. Tartrate D. Hydroxide

B. Phenol B The coupling step in the Trinder glucose oxidase method uses peroxidase to catalyze the oxidation of a dye by H2O2. Dyes such as 4 aminophenozone or 4-aminoantipyrine are coupled to phenol to form a quinoneimine dye that is red and is measured at about 500 nm.

A person is suspected of having testicular cancer. Which type of hCG test would be most useful? A. Plasma immunoassay for intact hCG only B. Plasma immunoassay for intact hCG and the β-hCG subunit C. Plasma immunoassay for the free alpha and β-hCG subunits D. Urine assay for hCG β core

B. Plasma immunoassay for intact hCG and the β-hCG subunit B In addition to testicular cancer, hCG is produced by trophoblastic tumors and choriocarcinomas. Some of these tumors secrete the β-subunit without intact hCG. This is especially true after treatment when hCG is used to monitor for recurrence. The use of an immunoassay that measures both the intact and free β hCG will have greater sensitivity than an assay for intact hCG or an assay for only free subunits. Free α hCG subunits may be produced in persons with testicular and urinary bladder (urothelial) cancer, but the incidence of α hCG subunit secretion only is relatively low. Urinary β core (urinary gonadotropin peptide) is a metabolic product of the β subunit and has been used to monitor for persistence of trophoblastic disease and recurrence of some hCG-producing tumors

Which support medium can be used to determine the molecular weight of a protein? A. Cellulose acetate B. Polyacrylamide gel C. Agar gel D. Agarose gel

B. Polyacrylamide gel B Polyacrylamide gels separate by molecular sieving as well as charge. Sodium dodecyl sulfate (SDS) is a nonionic detergent that binds to proteins, neutralizing their charge. Polyacrylamide gel electrophoresis (PAGE) after treating with SDS separates proteins on the basis of molecular size. The smaller proteins become trapped in the pores of the gel and migrate more slowly.

Which enzyme deficiency is most commonly associated with familial hypertriglyceridemia associated with fasting plasma cholomicrons (formerly type I hyperlipoproteinemia)? A. β Glucocerebrosidase deficiency B. Post-heparin-activated lipoprotein lipase deficiency C. Apo-B deficiency D. Apo-C-III deficiency

B. Post-heparin-activated lipoprotein lipase deficiency B Deficiency of capillary endothelial lipase is the most common cause of fasting chylomicronemia. This lipase is also known as post heparinactivated lipase and apo C-II-activated lipase. β Glucocerebrosidase deficiency results in accumulation of glucocerebrosides and is the cause of Gaucher's disease. ApoC-II deficiency results in decreased activity of peripheral and hepatic lipases and is associated with hypertriglyceridemia. Apo-B deficiency resulting from a point mutation in the apo-B gene, is responsible for hypobetalipoproteinemia, and is inherited as an autosomal dominant trait. LDL levels are about half normal in heterozygotes, and this reduces their risk of coronary artery disease.

Which of the following statements about the diagnosis of Addison's disease is true? A. Patients with primary Addison's disease show a normal response to ACTH stimulation B. Primary and secondary Addison's disease can often be differentiated by plasma ACTH C. Twenty-four-hour urinary free cortisol is normal in Addison's disease D. Pituitary ACTH reserves are normal in secondary Addison's disease

B. Primary and secondary Addison's disease can often be differentiated by plasma ACTH B ACTH (Cortrosyn) stimulation is used as a screening test for Addison's disease. A 250-μg dose of Cortrosyn is given intravenously. Normal patients show a 2-5 times increase in serum cortisol. A subnormal response occurs in both primary and secondary Addison's disease. Plasma ACTH is high in primary but is low in secondary Addison's disease. Patients with secondary Addison's disease (pituitary failure) do not respond to metyrapone because their ACTH reserve is diminished.

When the magnitude of error increases with increasing sample concentration, it is called: A. Constant error B. Proportional error C. Random error D. Bias

B. Proportional error B Proportional error (slope or percent error) results in greater absolute error (deviation from the target value) at higher sample concentration. Constant error refers to a difference between the target value and the result, which is independent of sample concentration. For example, if both level 1 and level 2 controls for laboratory A average 5 mg/dL below the cumulative mean reported by all other laboratories using the same method, then laboratory A has a constant error of -5 mg/dL for that method.

Which of the following would cause a "response" error from an ion-selective electrode for sodium when measuring serum but not the calibrator? A. Interference from other electrolytes B. Protein coating the ion-selective membrane C. An overrange in sodium concentration D. Protein binding to sodium ions

B. Protein coating the ion-selective membrane B Response is the time required for an electrode to reach maximum potential. Ion-selective analyzers use a microprocessor to monitor electrode response, slope, drift, and noise. When an electrode gives an acceptable response time when measuring an aqueous calibrator, but not when measuring serum, the cause is often protein buildup on the membrane.

One mole per kilogram H2O of any solute will cause all of the following except: A. Lower the freezing point by 1.86°C B. Raise vapor pressure by 0.3 mm Hg C. Raise the boiling point by 0.52°C D. Raise osmotic pressure by 22.4 atm

B. Raise vapor pressure by 0.3 mm Hg B Both freezing point and vapor pressure are lowered by increasing solute concentration. Boiling point and osmotic pressure are raised. Increasing solute concentration of a solution opposes a change in its physical state and lowers the concentration of H2O molecules.

SITUATION: An amylase result is 550 U/L. A 1:4 dilution of the specimen in NaCl gives 180 U/L (before mathematical correction for dilution). The dilution is repeated with the same results. The technologist should: A. Report the amylase as 550 U/L B. Report the amylase as 720 U/L C. Report the amylase as 900 U/L D. Dilute the sample 1:10 in distilled water and repeat

B. Report the amylase as 720 U/L B A 1:4 dilution refers to 1 part serum and 3 parts diluent; the result is multiplied by 4 to determine the serum concentration. Serum may contain wheat germ gluten or other natural amylase inhibitors that, when diluted, result in increased enzyme activity. Serum for amylase should always be diluted with normal saline because chloride ions are needed for amylase activity.

SITUATION: A patient who has a positive urinalysis for glucose and ketones has a glycated Hgb of 4.0%. A fasting glucose performed the previous day was 180 mg/dL. Assuming acceptable QC, you would: A. Report the glycosylated Hgb B. Request a new specimen and repeat the glycosylated Hgb C. Perform a Hgb electrophoresis on the sample D. Perform a glucose measurement on the sample

B. Request a new specimen and repeat the glycosylated Hgb B The glycated Hgb is at the lowest normal limit (4%-5.5%), but the fasting glucose indicates frank diabetes mellitus. Although the glycosylated Hgb reflects the average blood glucose 2-3 months earlier, the value reported is inconsistent with the other laboratory results. A high probability of sample misidentification or analytical error necessitates that the test be repeated.

Select the primary reagent used in the Jaffe method for creatinine. A. Alkaline copper II sulfate B. Saturated picric acid and NaOH C. Sodium nitroprusside and phenol D. Phosphotungstic acid

B. Saturated picric acid and NaOH B The Jaffe method uses saturated picric acid, which oxidizes creatinine in alkali, forming creatinine picrate. The reaction is nonspecific; ketones, ascorbate, proteins, and other reducing agents contribute to the final color. Alkaline CuSO4 is used in the biuret method for protein

Quantitation of a drug by gas chromatography-mass spectroscopy (GC-MS) is usually performed in which mode? A. Total ion chromatography B. Selective ion monitoring C. Ion subtraction D. Selective reaction monitoring

B. Selective ion monitoring B Most GC-MS instruments use an electron beam to split the drug emerging from the column into its component ions. These are drawn into the mass analyzer, usually a vacuum chamber containing two pairs of charged rods (a positive pair and a negative pair) called a quadrupole analyzer. By changing the potential and radio frequency applied to the rods, the travel of ions will vary depending upon their mass to charge (m/z) ratio. As ions emerge from the mass filter, they are detected by an electron multiplier tube. CG-MS instruments can be operated in two modes, total ion chromatography and selective ion monitoring. A total ion chromatograph displays the retention time of all ions detected and their abundance. It is primarily used for identification of unknown compounds. SIM mode measures the abundance of one or more principal ions that provides sufficient specificity to eliminate potential interfering substances and greater quantitative sensitivity. For example, tetrahydrocannabinol (THC) can be identified by ions m/z 371.3, 372.3, and 473.3

The modification of diet in renal disease (MDRD) formula for calculating eGFR requires which four parameters? A. Urine creatinine, serum creatinine, height, weight B. Serum creatinine, age, gender, race C. Serum creatinine, height, weight, age D. Urine creatinine, gender, weight, age

B. Serum creatinine, age, gender, race B The National Kidney Foundation recommends screening for chronic kidney disease using the estimated glomerular filtration rate (eGFR) because of the high frequency of sample collection errors associated with measuring creatinine clearance. The eGFR should be calculated according to the MDRD formula, and reported along with the serum or plasma creatinine. eGFR (mL/min/1.73m2) = 186 x Plasma Cr -1.154 x Age -0.203 x 0.742 (if female) x 1.21 (if Black)

Which formula provides the best estimate of serum TIBC? A. Serum transferrin in mg/dL × 0.70 = TIBC (µg/dL) B. Serum transferrin in mg/dL × 1.43 = TIBC (µg/dL) C. Serum iron (µg/dL)/1.2 + 0.06 = TIBC (µg/dL) D. Serum Fe (µg/dL) × 1.25 = TIBC (µg/dL)

B. Serum transferrin in mg/dL × 1.43 = TIBC (µg/dL) B Transferrin, a β-globulin, has a molecular size of about 77,000. Transferrin is the principal iron transport protein, and TIBC is determined by the serum transferrin concentration. One mole of transferrin binds two moles of Fe+3, so the transferrin concentration can be used to predict the TIBC. Since the direct measurement of TIBC requires manual pretreatment to remove the excess iron added and is prone to overestimation if all of the unbound iron is not removed, some labs prefer to measure transferrin immunochemically and calculate TIBC. This formula may underestimate TIBC because albumin and other proteins will bind iron when the percent iron saturation of transferrin is abnormally high.

Which of the following total quality management tools can be used to calculate the analytical error rate for an analyte in the clinical laboratory? A. LEAN B. Six sigma C. ISO 9000 D. Laboratory information system

B. Six sigma B All four of the answer choices are total quality management (TQM) tools used in the clinical laboratory to improve performance. Six sigma is a measurement of the frequency of product defects. In clinical laboratories, it refers to the frequency of an erroneous result. At the six-sigma level, the analytical process has such small variance that an error of six times the standard deviation would still be within acceptable limits for total allowable error. For example, a six-sigma process for an analyte produces a significant error in test result only 3.4 out of 1 million times the test is performed. Conversely, a method performing at the three-sigma level would give 66,807 errors per million. The sigma of a method is calculated by subtracting its bias from the total method error and dividing by its standard deviation. It is the method's random error divided by its standard deviation.

The term RT/nF in the Nernst equation defines the: A. Potential at the ion-selective membrane B. Slope of the electrode C. Decomposition potential D. Isopotential point of the electrode

B. Slope of the electrode B In the term RT/nF, R = the molar gas constant, T = temperature in degrees Kelvin, F = Faraday's constant, and n = the number of electrons donated per atom of reductant. The slope depends upon the temperature of the solution and the valence of the reductant. At room temperature, the slope is 59.2 mV for a univalent ion and 29.6 mV for a divalent ion.

Which statement best describes the level of GH in patients with pituitary adenoma associated with acromegaly? A. The fasting GH level is always elevated at least twofold B. Some patients will require a glucose suppression test to establish a diagnosis C. A normal fasting GH level rules out acromegaly D. Patients produce a lower concentration of insulin-like growth factor I (IGF-1) than expected from their GH level

B. Some patients will require a glucose suppression test to establish a diagnosis B Approximately 90% of patients with acromegaly will have an elevated fasting GH level, but 10% will not. In addition, a single measurement is not sufficient to establish a diagnosis of acromegaly because various metabolic and nutritional factors can cause an elevated serum GH in the absence of pituitary disease. The glucose suppression test is used to diagnose acromegaly. An oral dose of 100 g of glucose will suppress the serum GH level at 1 hour (postadministration) to below 1 μg/L in normal patients, but not in patients with acromegaly. Patients with acromegaly also have high levels of IGF-1, also called somatomedin C, which is overproduced by the liver in response to excess release of GH

Which statement regarding thyroid hormones is true? A. Circulating levels of T3 and T4 are about equal B. T3 is about 10-fold more active than T4 C. The rate of formation of monoiodotyrosine and diiodotyrosine is about equal D. Most of the T3 present in plasma is from its direct release from thyroid storage sites

B. T3 is about 10-fold more active than T4 B The rate of DIT synthesis is twice that of MIT and the rate of coupling favors formation of T4. Levels of T4 are about 50 times those of T3, but T3 is approximately 10 times more active physiologically. Eighty percent of circulating T3 is derived from enzymatic conversion of T4 by T4 5´-deiodinase

Which statement about TSH and T4 in early pregnancy is correct? A. TSH and thyroid hormones fall B. TSH falls and thyroid hormones rise C. TSH and thyroid hormones both rise D. TSH rises and thyroid hormones fall

B. TSH falls and thyroid hormones rise B Estrogens released in pregnancy cause an increase in TBG, which causes an increase in total T4 and T3. In early pregnancy, the hCG produced by the placenta stimulates the thyroid, causing an increase in free thyroid hormones. This suppresses TSH production. In the second trimester, as hCG diminishes, free T4 levels fall, and may be lower than 0.8 ng/dL, the lower limit of the adult reference range due to expansion of the blood volume. Therefore, both TSH and free T4 should be evaluated during pregnancy using trimester-specific reference ranges. In early pregnancy, a TSH above the first- trimester reference range should be followed up with free T4 and thyroid peroxidase antibody levels to assess the need for thyroid treatment.

SITUATION: A plasma sample from a person in a coma as a result of an automobile accident gave the following results: Total CK 480 IU/L CK-MB 8 µg/L Myoglobin 800 µg/L Troponin I 0.02 µg/L What is the best interpretation of these results? A. The person had a heart attack that caused the accident B. The accident caused traumatic injury, but no heart attack occurred C. A heart attack occurred in addition to a stroke D. It is not possible to tell whether a heart attack occurred because of the extensive trauma

B. The accident caused traumatic injury, but no heart attack occurred B The automobile accident caused both brain damage (coma) and muscle damage (myoglobin). The sandwich assay for MB uses antibodies to both the M and B subunits of CK-MB and therefore, is not subject to interference from CK-BB that could have resulted from brain injury. The CK relative index is 1.6, which is lower than would be expected if the CK-MB were derived from heart damage. Since the TnI is within normal limits, the slight increase in CK-MB is due to the gross release of CK from skeletal muscle.

SITUATION: A GC-MS analysis using nitrogen as the carrier gas shows an extensively noisy baseline. A sample of the solvent used for the extraction procedure, ethyl acetate, was injected and showed the same noise. Results of an Autotune test showed the appearance of a base peak at 16 with two smaller peaks at 17 and 18. Tese results indicate: A. The solvent is contaminated B. The carrier gas is contaminated C. There is electrical noise in the detector D. The ion source is dirty

B. The carrier gas is contaminated B All of these situations are sources of baseline noise in GC-MS. However, the peak at 16 indicates the presence of oxygen in the carrier gas. Oxygen in the atmosphere also contains small quantities of two isotopes with molecular weights of 17 and 18 owing to one and two extra neutrons, respectively.

In tandem-mass spectroscopy, the first mass filter performs the same function as: A. The ion source B. The chromatography column C. Extraction D. The vacuum system

B. The chromatography column B A tandem mass spectrometer uses two or more mass filters in sequence. The first filter functions as an ion trap. Once the sample is ionized, the filter selects molecular or parent ions of interest by excluding ions outside a specified size range. Therefore, it effectively separates the analyte(s) of interest from unwanted compounds. Tandem MS uses ESI to introduce the sample into the first mass filter, usually a quadrapole. The RF and DC voltages of the quadrapole are set to optimize the trajectory of the parent ions of interest and cause ejection of unwanted ions. The parent ions are drawn into a second mass filter where they are bombarded by argon atoms. The collisions result in the formation of mass fragments called daughter ions. This process is called collision-induced dissociation and the second filter is called a collision chamber. The process can be repeated in a third mass filter that generates granddaughter ions. A total-ion chromatogram is produced from these, enabling the compound of interest to be identified and quantified. Tandem MS is used to screen for inborn errors of fatty acid, amino acid, and organic acid metabolism.

In the coulometric chloride titration: A. Acetic acid in the titrating solution furnishes the counter ion for reduction B. The endpoint is detected by amperometry C. The titrating reagent contains a phosphate buffer to keep pH constant D. Nitric acid (HNO3) is used to lower the solubility of AgCl

B. The endpoint is detected by amperometry B Reduction of Ag+ back to Ag° generates the current, which signals the endpoint. The titrating reagent contains HNO3, acetic acid, H2O, and either gelatin or polyvinyl alcohol. The HNO3 furnishes nitrate, which is reduced at the generator cathode, forming ammonium ions. The ammonium becomes oxidized back to nitrate at the indicator anode. Gelatin or polyvinyl alcohol is needed to prevent pitting of the generator anode. Acetic acid lowers the solubility of AgCl, preventing dissociation back to Ag+

Which statement regarding cloned enzyme donor immunoassay (CEDIA) is true? A. The enzyme used is glucose-6-phosphate dehydrogenase B. The enzyme donor and acceptor molecules are fragments of β-galactosidase C. Drug concentration is inversely related to fluorescence D. The antibody is covalently linked to the enzyme donor

B. The enzyme donor and acceptor molecules are fragments of β-galactosidase B CEDIA is a homogenous enzyme immunoassay that is commonly used to measure drugs of abuse. Drug conjugated to a fragment of β-galactosidase that is catalytically inactive competes with drug in the sample for a limited number of antibodies to the drug. The fragment, called the enzyme donor (ED), and substrate (chlorophenol red-β-D-galactopyranose) are mixed with the sample. A second reagent containing monoclonal antibody and a second fragment of β-galactosidase called the enzyme acceptor (EA) is added. If the antibody is neutralized by drug from the sample, the ED and EA combine forming an active enzyme. The concentration of drug in the sample is directly proportional to the amount of chlorophenol red formed.

The following chart compares the monthly total bilirubin mean of Laboratory A to the monthly mean of Laboratory B, which uses the same control materials, analyzer, and method. Level 1 Control- Level 2 Control- Mean(mg/dL) CV (%) Lab A 1.1 mg/dL 2.1%- 6.7 mg/dL 3.2% Mean (mg/dL) CV (%) Lab B 1.4 mg/dL 2.2% 7.0 mg/dL 3.6% Both laboratories performed controls at the beginning of each shift using commercially prepared liquid QC serum stored at -20°C. Which of the following conditions would explain these differences? A. Improper handling of the control material by Laboratory A resulted in loss of bilirubin due to photodegradation B. The laboratories used a different source of bilirubin calibrator C. Laboratory B obtained higher results because its precision was poorer D. Carryover from another reagent falsely elevated the results of Laboratory B

B. The laboratories used a different source of bilirubin calibrator B Interlaboratory variation in bilirubin results is often caused by differences in the assigned value of the calibrator used. Bilirubin calibrators are either serum-based material that have been reference assayed or unconjugated bilirubin stabilized by addition of alkali and albumin. Calibrator differences result in bias and should be suspected when the laboratory's mean differs significantly from the peer group's mean. The bias in this example is due to constant rather than proportional error. When bilirubin calibrator error is suspected, the molar absorptivity of the calibrator should be measured and the bilirubin concentration calculated. Photodegradation generally results in a greater loss of bilirubin at higher concentration and also contributes to random error.

AST ALT ALP LD CK GGT TP ALB TBIL GLU TG CA InP BUN U/L U/L U/L U/L U/L U/L g/dL g/dL mg/dL mg/dL mg/dL mg/dL mg/dL mg/dL Day 1 20 15 40 100 15 40 8.2 3.6 0.8 84 140 8.7 4.2 16 Day 2 22 14 65 90 20 36 8.3 3.8 1.0 128 190 8.8 5.2 26 SITUATION: Biochemistry tests are performed 24 hours apart on a patient and delta-check flag is reported for inorganic phosphorus by the laboratory information system. Given the results shown in the table above, identify the most likely cause. A. Results suggest altered metabolic status caused by poor insulin control B. The patient was not fasting when the sample was collected on day 2 C. The samples were drawn from two different patients D. The delta-check limit is invalid when samples are collected 24 or more hours apart

B. The patient was not fasting when the sample was collected on day 2 B The delta check compares the difference of the patient's two most recent laboratory results within a 3-day period to a delta limit usually determined as a percentage difference. The purpose of the delta check is to detect sample identification errors. A delta-check flag can also be caused by random analytical errors and interfering substances such as hemolysis, icterus, and lipemia, and by metabolic changes associated with disease or treatment. Therefore, results should be carefully considered before determining the cause. In this case, hemolysis and icterus can be ruled out because enzymes sensitive to hemolysis interference (AST, ALT, and LD) and bilirubin are within normal limits. Tests showing a significant difference are inorganic phosphorus, ALP, triglycerides, and glucose. These four tests are elevated by diet (the ALP from postprandial secretion of intestinal ALP). All other tests show a high level of agreement between days, and the differences are attributable to normal physiological and analytical variation

In mass spectroscopy, the term base peak typically refers to: A. The peak with the lowest mass B. The peak with the most abundance C. A natural isotope of the molecular ion D. The first peak to reach the mass detector

B. The peak with the most abundance B The base peak is typically the "molecular ion" or parent ion, meaning that it is the initial fragment made by releasing an electron. The cation thus formed has a charge of +1, and therefore, its m/z ratio is equal to its mass. The base peak is used for selective ion monitoring (SIM). It is the most abundant and most stable ion, and gives the best sensitivity for quantitative analysis.

Which statement about methods for measuring LD is true? A. The formation of pyruvate from lactate (forward reaction) generates NAD+ B. The pyruvate-to-lactate reaction proceeds at about twice the rate as the forward reaction C. The lactate-to-pyruvate reaction is optimized at pH 7.4 D. The negative-rate reaction is preferred

B. The pyruvate-to-lactate reaction proceeds at about twice the rate as the forward reaction B Although the rate of the reverse reaction (P → L) is faster, the L → P reaction is more popular because it produces a positive rate (generates NADH), is not subject to product inhibition, and is highly linear. The pH optimum for the forward reaction is approximately 8.8

Polarographic methods for glucose analysis are based upon which principle of measurement? A. Nonenzymatic oxidation of glucose B. The rate of O2 depletion C. Chemiluminescence caused by formation of adenosine triphosphate (ATP) D. The change in electrical potential as glucose is oxidized

B. The rate of O2 depletion B Polarographic glucose electrodes measure the consumption of O2 as glucose is oxidized. Glucose oxidase in the reagent catalyzes the oxidation of glucose by O2 under first order conditions, forming hydrogen peroxide (H2O2). As the dissolved O2 decreases, less is reduced at the cathode, resulting in a decrease in current proportional to glucose concentration. It is important that the H2O2 not breakdown to re-form O2. This is prevented by adding molybdate and iodide that react with H2O2, forming iodine and water, and by adding catalase and ethanol that react with H2O2, forming acetaldehyde and water

BUN is determined electrochemically by coupling the urease reaction to measurement of: A. Potential with a urea-selective electrode B. The timed rate of increase in conductivity C. The oxidation of ammonia D. Carbon dioxide

B. The timed rate of increase in conductivity B A conductivity electrode is used to measure the increase in conductance of the solution as urea is hydrolyzed by urease in the presence of sodium carbonate. Urea + H2O → 2NH3 + CO2 2NH3 + 2H2O + Na2CO3 → 2NH4 + + CO3 -2 + 2NaOH Ammonium ions increase the conductance of the solution. The timed rate of current increase is proportional to the BUN concentration. Alternatively, the ammonium ions produced can be measured using an ion-selective electrode

What is the recommended troponin T and I cutoff (upper limit of normal) for detecting myocardial infarction? A. The cutoff varies with the method of assay but should be no lower than 0.2 ng/mL B. The upper 99th percentile or lowest level that can be measured with 10% CV C. The concentration corresponding to the lowest level of calibrator used D. The highest value fitting under the area of the curve for the 95% confidence interval

B. The upper 99th percentile or lowest level that can be measured with 10% CV B The American College of Cardiology recommends the cutoff for an abnormal troponin test be set at the 99th percentile of the normal population, 0.013 ng/mL, or if the assay precision at this level is >10% then the cutoff should be the lowest value measurable with a CV of 10% (typically 0.03 ng/mL). An abnormal result (0.04 ng/mL or higher) in a patient with other evidence of ischemic changes indicates cardiac damage. This typically occurs when a pattern of increasing troponin concentration is seen over the first 6 hours after initial testing.

Which statement best summarizes the relationship between the new BUN method and reference method based upon the following linear regression scatterplot? A. The methods agree very well but show a high standard error of estimate B. There is little or no constant error, but some proportional error C. There will be a significant degree of uncertainty in the regression equation D. There is significant constant and proportional error but little

B. There is little or no constant error, but some proportional error B The scatterplot shows that each sample produces a coordinate (x corresponds to the reference result and y to the candidate method result) that is very close to the regression line. This means that the variance of regression is low and there is a high degree of certainty that the predicted value of y will be close to its measured value. Near-zero concentration there is good agreement between methods; however, the higher the result, the greater the difference between x and y. The regression equation for this scatterplot is y = -0.01 + 0.90 x, indicating a proportional error of -10%

Which of the following statements about the phosphatases is true? A. They hydrolyze adenosine triphosphate and related compounds B. They are divided into two classes based upon pH needed for activity C. They exhibit a high specificity for substrate D. They are activated by Pi

B. They are divided into two classes based upon pH needed for activity B Phosphatases are classified as either alkaline or acid depending upon the pH needed for optimum activity. The phosphatases hydrolyze a wide range of monophosphoric acid esters. ALP is inhibited by phosphorus (product inhibition). The International Federation of Clinical Chemistry (IFCC) recommended method employs 2-amino-2-methyl-1-propanol, a buffer that binds Pi.

Why do many optical systems in chemistry analyzers utilize a reference light path? A. To increase the sensitivity of the measurement B. To minimize error caused by source lamp fluctuation C. To obviate the need for wavelength adjustment D. To reduce stray light effects

B. To minimize error caused by source lamp fluctuation B A reference beam is used to produce an electrical signal at the detector to which the measurement of light absorption by the sample is compared. This safeguards against measurement errors caused power fluctuations that change the source lamp intensity. Although reference beams increase the accuracy of measurements, they do so at the expense of optical sensitivity since some of the incident light must be used to produce the reference beam

Which of the following plots is best for comparison of precision and accuracy among laboratories? A. Levy-Jennings B. Tonks-Youden C. Cusum D. Linear regression

B. Tonks-Youden B The Tonks-Youden plot is used for interlaboratory comparison of monthly means. The method mean for level 1 is at the center of the y axis and mean for level 2 at the center of the x axis. Lines are drawn from the means of both levels across the graph, dividing it into four equal quadrants. If a laboratory's monthly means both plot in the lower left or upper right, then systematic error (SE) exists in its method

Which of the following statements regarding thyroid hormones is true? A. Both protein-bound and free T3 and T4 are physiologically active B. Total T3 and T4 are influenced by the level of thyroxine-binding globulin C. Variation in thyroxine-binding protein levels affects both free T3 and T4 D. An elevated serum total T4 and T3 is diagnostic of hyperthyroidism

B. Total T3 and T4 are influenced by the level of thyroxine-binding globulin B Total serum T4 and T3 are dependent upon both thyroid function and the amount of thyroxine-binding proteins such as thyroxine-binding globulin (TBG). Total T4 or T3 may be abnormal in a patient with normal thyroid function, if the TBG level is abnormal. For this reason, free T3 and T4 are more specific indicators of thyroid function than are measurements of total hormone. Only free hormone is physiologically active.

Examine the Levy-Jennings chart at the bottom of the previous page and identify the QC problem that occurred during the first half of the month. A. Shift B. Trend C. Random error D. Kurtosis

B. Trend B A trend is characterized by six consecutive decreasing or increasing control results. The value for both controls becomes progressively higher from day 4 to day 9. Trends are caused by changes to the test system that increase over time, such as deterioration of reagents or calibrators, progressive changes in temperature, evaporation, light exposure, and bacterial contamination. A trend is a type of SE because all results are affected. Conversely, RE affects some results (but not others) in an unpredictable manner. Control rules affected by RE are 13s and R4s.

Which of the following cardiac markers is consistently increased in persons who exhibit unstable angina? A. Troponin C B. Troponin T C. CK-MB D. Myoglobin

B. Troponin T B Persons with unstable angina (angina at rest) who have an elevated TnT or TnI are at eight times greater risk of having an MI within the next 6 months. This property is being used to identify short-term risk patients who should be considered for coronary angioplasty. The reference range for troponin is very low (0-0.03 ng/mL); persons with unstable angina usually have values between 0.04 and 0.1 ng/mL without clinical evidence of AMI. CK-MB and myoglobin have not been useful in identifying persons with unstable angina.

Which statement regarding the measurement of urinary catecholamines is true? A. An increased excretion of total urinary catecholamines is specific for pheochromocytoma B. Twenty-four-hour urinary catecholamine assay avoids pulse variations associated with measurement of plasma catecholamines C. Total urinary catecholamine measurement provides greater specificity than measurement of urinary free catecholamines D. Total urinary catecholamines are not affected by exercise

B. Twenty-four-hour urinary catecholamine assay avoids pulse variations associated with measurement of plasma catecholamines B Measurement of total urinary catecholamines is not a specific test for pheochromocytoma. Urine levels may be increased by exercise and in muscular diseases. Catecholamines in urine may also be derived from dietary sources rather than endogenous production. Most catecholamines are excreted as the glucuronide, and the urinary free catecholamines increase only when there is increased secretion. Measurement of free hormone in urine is equal in clinical sensitivity and specificity to measurement of metanephrines. Twenty-four hour urine is the sample of choice because plasma levels are subject to pulse variation and affected by the patient's psychological and metabolic condition at the time of sampling

The term R4S means that: A. Four consecutive controls are greater than ±1 standard deviation from the mean B. Two controls in the same run are greater than 4s units apart C. Two consecutive controls in the same run are each greater than ±4s from the mean D. There is a shift above the mean for four consecutive controls

B. Two controls in the same run are greater than 4s units apart B The R4s rule is applied to two control levels withinthe same run. The rule is violated when the algebraic difference between them (level 1 - level 2) exceeds 4s. The rule is never applied across different runs. The R4s rule detects random error (error due to poor precision).

Which enzyme is responsible for the conjugation of bilirubin? A. β-Glucuronidase B. UDP-glucuronyl transferase C. Bilirubin oxidase D. Biliverdin reductase

B. UDP-glucuronyl transferase B UDP-glucuronyl transferase esterifies glucuronic acid to unconjugated bilirubin, making it water soluble. Most conjugated bilirubin is diglucuronide; however, the liver makes a small amount of monoglucuronide and other glycosides. β Glucuronidase hydrolyzes glucuronide from bilirubin, hormones, or drugs. It is used prior to organic extraction to deconjugate urinary metabolites (e.g., total cortisol). Biliverdin reductase forms bilirubin from biliverdin (and heme oxygenase forms biliverdin from heme). Bilirubin oxidase is used in an enzymatic bilirubin assay in which bilirubin is oxidized back to biliverdin and the rate of biliverdin formation is measured at 410 nm.

A blood sample is left on a phlebotomy tray for 4.5 hours before it is delivered to the laboratory. Which group of tests could be performed? A. Glucose, Na, K, Cl, TCO2 B. Uric acid, BUN, creatinine C. Total and direct bilirubin D. CK, ALT, ALP, AST

B. Uric acid, BUN, creatinine B Glucose in serum is metabolized by cells at a rate of about 7% per hour. Bilirubin levels will fall if the sample is exposed to sunlight. Transaminases should be measured within 4 hours and ALP within 2 hours if the sample is stored at room temperature. Uric acid, BUN, and creatinine are least likely to be affected.

Which statement is true regarding the volume distribution (Vd) of a drug? A. Vd is equal to the peak blood concentration divided by the dose given B. Vd is the theoretical volume in liters into which the drug distributes C. The higher the Vd, the lower the dose needed to reach the desired blood level of drug D. The Vd is the principal determinant of the dosing interval

B. Vd is the theoretical volume in liters into which the drug distributes B The Vd of a drug represents the dilution of the drug after it has been distributed in the body. The Vd is used to estimate the peak drug blood level expected after a loading dose is given. The peak blood level equals the dose multiplied by f ÷ Vd. The Vd can be calculated by dividing the dose, Xo, by the initial plasma drug concentration, Co, (Vd = Xo/Co) or by dividing the clearance rate by K, the elimination rate constant (K = 0.693 divided by drug half-life). The greater the Vd, the higher the dose that will be needed to achieve the desired blood concentration of drug. The Vd is the principal determinant of the dose, and the clearance rate is the principal determinant of the dosing interval.

A chromatogram for blood alcohol (GC) gives broad trailing peaks and increased retention times for ethanol and internal standard. This is most likely caused by: A. A contaminated injection syringe B. Water contamination of the column packing C. Carrier gas flow rate that is too fast D. Oven temperature that is too high

B. Water contamination of the column packing B Increased oven temperature or gas flow rate will shorten retention times and decrease peak widths. Syringe contamination may cause the appearance of ghost peaks. Water in a PEG column such as Carbowax used for measuring volatiles causes longer retention times and loss of resolution.

Which of the following statements regarding the identification of monoclonal proteins by IFE is true? A. The monoclonal band must be present in the γ region B. When testing for a monoclonal gammopathy, both serum and urine must be examined C. A diagnosis of monoclonal gammopathy is based upon quantitation of IgG, IgA, and IgM D. A monoclonal band always indicates a malignant disorder

B. When testing for a monoclonal gammopathy, both serum and urine must be examined B Quantitation of IgG, IgA, IgM, or IgD indicates the concentration of each class of immunoglobulin but does not distinguish monoclonal from polyclonal gammopathies. Monoclonal characteristics are determined by demonstrating restricted electrophoretic mobility, indicating that all immunoglobulins in the band are of the same amino acid sequence. Monoclonal light chains can be demonstrated in about 60% of monoclonal gammopathies. In up to 25% of multiple myeloma patients, a heavy chain gene deletion results in production of monoclonal light chains only. Because these are filtered by the glomerulus, the procedure must be performed on urine as well as serum. Some patients with a monoclonal protein fail to develop malignant plasma cell proliferation. This state is called a monoclonal gammopathy of undetermined significance (MGUS). Within 10-15 years, 15%-20% of persons with MGUS develop some form of lymphoproliferative disease.

Which set of results is consistent with uncompensated respiratory alkalosis? A. pH 7.70 HCO3 30 mmol/L PCO2 25 mm Hg B. pH 7.66 HCO3 22 mmol/L PCO2 20 mm Hg C. pH 7.46 HCO3 38 mmol/L PCO2 55 mm Hg D. pH 7.36 HCO3 22 mmol/L PCO2 38 mm Hg

B. pH 7.66 HCO3 22 mmol/L PCO2 20 mm Hg B Respiratory alkalosis is caused by hyperventilation, inducing low PCO2. Very often, in the early phase of an acute respiratory disturbance, the kidneys have not had time to compensate, and the bicarbonate is within normal limits. In answer A, the bicarbonate is high and PCO2 low; thus, both are contributing to alkalosis and this would be classified as a combined acid-base disturbance. In answer C, the pH is almost normal, and both bicarbonate and PCO2 are increased. This can occur in the early stage of a metabolic acid base disturbance when full respiratory compensation occurs or in a combined acid-base disorder. In answer D, both bicarbonate and PCO2 are within normal limits (22-26 mmol/L, 35-45 mm Hg, respectively) as is the pH.

Which of the following represents the Henderson Hasselbalch equation as applied to blood pH? A. pH = 6.1 + log HCO3-/PCO2 B. pH = 6.1 + log HCO3-/ (0.03 × PCO2) C. pH = 6.1 + log dCO2/HCO3- D.pH=6.1 + log (0.03×PCO2) / HCO3-

B. pH = 6.1 + log HCO3-/ (0.03 × PCO2) B The Henderson-Hasselbalch equation describes the pH of a buffer comprised of a weak acid and its salt. pH = pKa + log salt/acid, where pKa is the negative logarithm of the dissociation constant of the acid. In this case, the salt is sodium bicarbonate and the acid is the dissolved CO2, which is equal to 0.03 (mmol/L per mm Hg) x PCO2. The pKa includes both the hydration and dissociation constant for dissolved CO2 in blood, 6.1 and is termed pK´.

Which of the following findings is characteristic of all forms of clinical hypoglycemia? A. A fasting blood glucose value below 55 mg/dL B. High fasting insulin levels C. Neuroglycopenic symptoms at the time of low blood sugar D. Decreased serum C peptide

C Clinical hypoglycemia can be caused by insulinoma, drugs, alcoholism, and reactive hypoglycemia. Neuroglycopenic symptoms at the time of low blood sugar C. Reactive hypoglycemia is characterized by delayed or excessive insulin output after eating and is very rare. Fasting insulin is normal but postprandial levels are increased. High fasting insulin levels (usually > 6 μg/L) are seen in insulinoma, and patients with insulinoma almost always display fasting hypoglycemia, especially when the fast is extended to 48 72 hours. C peptide is a subunit of proinsulin that is hydrolyzed when insulin is released. In hypoglycemia, low levels indicate an exogenous insulin source, whereas high levels indicate overproduction of insulin.

In thin-layer chromatography (TLC), the distance the solute migrates divided by the distance the solvent migrates is the: A. tR B. Kd C Rf D. pK

C Rf C Rf is the distance migrated by the solute divided by the distance migrated by the solvent. The tR refers to the retention time of the solute in HPLC or gas chromatography (GC). The Kd is the partition coefficient, and is a measure of the relative affinity of solutes for the stationary phase. The solute with the greater Kd will be retained longer. The pK is the negative logarithm of K, the ionization constant, and is a measure of ionization.

Select the order of mobility of lipoproteins electrophoresed on cellulose acetate or agarose at pH 8.6. A. - Chylomicrons→pre-β →β→α+ B. - β→pre-β→α→chylomicrons + C. - Chylomicrons →β→pre-β→α + D. - α→β→pre-β→chylomicrons +

C. - Chylomicrons →β→pre-β→α + C Although pre-β lipoprotein is lower in density than β lipoprotein, it migrates faster on agarose or cellulose acetate owing to its more negative apoprotein composition. When lipoproteins are separated on polyacrylamide gel, pre-β moves slower than β lipoprotein. Molecular sieving causes migration to correlate with lipoprotein density when PAGE is used.

Select the correct order of Hgb migration on agarose or cellulose acetate at pH 8.6. A. - C→F→S→A + B. - S→C→A→F + C. - C→S→F→A + D. - S→F→A→C +

C. - C→S→F→A + C Hgb A2 is the slowest of the normal Hgbs, and Hgb A is the fastest. Hgb F migrates just behind Hgb A. Hgb S migrates midway between Hgb A2 and Hgb A. Hgbs C, CHarlem (Georgetown), O, and E migrate with Hgb A2. Hgbs G and DPunjab and Hgb OArab migrate with Hgb S.

A new method for BUN is evaluated by comparing the results of 40 paired patient samples to the urease-UV method. Normal and high controls were run on each shift for 5 days, five times per day. The results are as follows: Linear Regression/ Low Control/ High Control ŷ = -0.3 + 0.90x/ -x = 14.2 mg/dL; s=1.24/ -x = 48.6 mg/dL; s=1.12 What is the total analytical error estimate for a sample having a concentration of 50 mg/dL? A. -2.2 mg/dL B. -2.8 mg/dL C. -7.5 mg/dL D.-10.0 mg/dL

C. -7.5 mg/dL C Linear regression analysis gives an estimate of SE, which is equal to (ŷ - xc) where xc is the expected concentration, and ŷ is the value predicted by the linear regression equation. SE = [-0.3 + (0.9 × 50 mg/dL)] - 50.0 mg/dL = 44.7-50.0 = -5.3 mg/dL The standard deviation of the new method for the high control is used to estimate the RE because the mean of this control is nearest to the expected concentration of 50 mg/dL. RE is estimated by ±1.96 × s. RE = 1.96 × 1.12 = ± 2.2 mg/dL Total analytical error (TE) is equal to the sum of SE and RE. TE = SE + RE = -5.3 mg/dL + (-2.2 mg/dL) = -7.5 mg/dL

Which statement regarding reflectometry is true? A. The relation between reflectance density and concentration is linear B. Single-point calibration can be used to determine concentration C. 100% reflectance is set with an opaque film called a white reference D. The diode array is the photodetector of choice

C. 100% reflectance is set with an opaque film called a white reference C Reflectometry does not follow Beer's law, but the relationship between concentration and reflectance can be described by a logistic formula or algorithm that can be solved for concentration. For example, K/S = (1 - R) 2/2R, where K = Kubelka-Munk absorptivity constant, S = scattering coefficient, R = reflectance density. K/S is proportional to concentration. The white reference is analogous to the 100%T setting in spectrophotometry and serves as a reference signal. Dr = log R0/R1, where Dr is the reflectance density, R0 is the white reference signal, and R1 is the photodetector signal for the test sample.

Given the following real-time PCR amplification curve, what is the threshold cycle? A. 15 B. 20 C. 25 D. 30

C. 25 C The maximum curvature of the plot approximates the threshold cycle. A line is drawn from the threshold value on the y-axis through the curve, and a perpendicular dropped to the x-axis. The Ct is determined by the intersection point on the x-axis. The threshold is usually determined by an algorithm but can be calculated manually as 10 times the average standard deviation of the RFUs for cycles 2-10.

What is the most appropriate fasting procedure when a lipid study of triglyceride, total cholesterol, HDL cholesterol, and LDL cholesterol tests are ordered? A. 8 hours; nothing but water allowed B. 10 hours; water, smoking, coffee, tea (no sugar or cream) allowed C. 12 hours; nothing but water allowed D. 16 hours; water, smoking, coffee, tea (no sugar or cream) allowed

C. 12 hours; nothing but water allowed C Lipid orders that include triglyceride and LDL cholesterol should always be performed using a plasma or serum specimen collected after a 12-14 hour fast. The patient should be instructed to drink nothing but water during this period.Fasting specimens are preferred for total and HDL cholesterol as well, but nonfasting specimens may be used for initial screening purposes.

How many milliliters of HNO3 (purity 68.0%, specific gravity 1.42) are needed to prepare 1.0 L of a 2.0 N solution? Atomic weights: H = 1.0; N = 14.0; O = 16.0 A. 89.5 mL B. 126.0 mL C. 130.5 mL D. 180.0 mL

C. 130.5 mL C The molecular weight of HNO3 is 63.0 g. Because the valance of the acid is 1 (1 mol of hydrogen is produced per mole of acid), the equivalent weight is also 63.0 g. The mass is calculated by multiplying the normality (2.0 N) by the equivalent weight (63.0 g) and volume (1.0 L); therefore, 126.0 g of acid are required. Because the purity is 68.0% and the specific gravity 1.42, the amount of HNO3 in grams per milliliter is 0.68 × 1.42 g/mL or 0.9656 g/mL. The volume required to give 126.0 g is calculated by dividing the mass needed (grams) by the grams per milliliter. mL HNO3 = 126.0 g ÷ 0.9656 g/mL = 126.0 g × 1.0 mL/0.9656 g = 130.5 mL

What is the PO2 of calibration gas containing 20.0% O2, when the barometric pressure is 30 in.? A. 60 mm Hg B. 86 mm Hg C. 143 mm Hg D. 152 mm Hg

C. 143 mm Hg C Convert barometric pressure in inches to mm Hg by multiplying by 25.4 (mm/in.). Next, subtract the vapor pressure of H2O at 37°C, 47 mm Hg, to give dry gas pressure. Multiply dry gas pressure by the %O2: 25.4 mm/in. × 30 in. = 762 mm Hg 762 mm Hg - 47 mm Hg (vapor pressure) = 715 mm Hg (dry gas pressure) 0.20 × 715 mm Hg = 143 mm Hg PO2

A biuret reagent requires preparation of a stock solution containing 9.6 g of copper II sulfate (CuSO4) per liter. How many grams of CuSO4 • 5H2O are needed to prepare 1.0 L of the stock solution? Atomic weights: H = 1.0; Cu = 63.6; O = 16.0; S = 32.1 A. 5.4 g B. 6.1 g C. 15.0 g D. 17.0 g

C. 15.0 g C Determine the mass of CuSO4 • 5H2O containing 9.6 g of anhydrous CuSO4. First, calculate the percentage of CuSO4 in the hydrate, then divide the amount needed (9.6 g) by the percentage. % CuSO4 = molecular weight CuSO4 ÷ molecular weight CuSO4 • 5H2O × 100 = (159.7 ÷ 249.7) × 100 = 63.96% Grams CuSO4 • 5H2O = 9.6 g ÷ 0.6396 = 15.0 g A convenient formula to use is: g hydrate = (MW hydrate ÷ MW anhydrous salt) × g anhydrous salt

Which of the following 2-hour glucose challenge results would be classified as impaired glucose tolerance (IGT)? Two-hour serum glucose: A. 130 mg/dL B. 135 mg/dL C. 150 mg/dL D. 204 mg/dL

C. 150 mg/dL C With the exception of pregnant females, impaired glucose tolerance is defined by the ADA as a serum or plasma glucose at 2 hours following a 75-g oral glucose load of ≥140 mg/dL and < 200 mg/dL. Persons who have a fasting plasma glucose of ≥100 but < 126 mg/dL are classified as having impaired fasting glucose (IFG). Both IGT and IFG are risk factors for developing diabetes later in life. Such persons are classified as having prediabetes and should be tested annually.

Which formula is most accurate in predicting plasma osmolality? A. Na + 2(Cl) + BUN + glucose B. 2(Na) + 2(Cl) + glucose + urea C. 2(Na) + (glucose ÷ 18) + (BUN ÷ 2.8) D. Na + Cl + K + HCO3

C. 2(Na) + (glucose ÷ 18) + (BUN ÷ 2.8) C Calculated plasma osmolality is based upon measurement of sodium, glucose, and urea. Because sodium associates with a counter ion, two times the sodium estimates the millimoles per liter of electrolytes. Some laboratories multiply by 1.86 instead of 2 to correct for undissociated salts. Dividing glucose by 18 converts from milligrams per deciliter to millimoles per liter. Dividing blood urea nitrogen (BUN) by 2.8 converts from milligrams per deciliter BUN to millimoles per liter urea.

Which of the following buffers is used in the IFCC recommended method for ALP? A. Glycine B. Phosphate C. 2-Amino-2-methyl-1-propanol D. Citrate

C. 2-Amino-2-methyl-1-propanol C The Szasz modification of the Bowers-McComb method measures the hydrolysis of p-nitrophenyl phosphate, and continuously monitors the formation of p nitrophenol at 405 nm. AMP buffer chelates phosphorus, preventing product inhibition; Zn+2 and Mg+2 are added to the substrate to activate ALP. HEDTA is used to chelate the excess Zn+2, which is inhibitory at high concentrations

Convert 2.0 mEq/L magnesium (atomic weight = 24.3) to milligrams per deciliter. A. 0.8 mg/dL B. 1.2 mg/dL C. 2.4 mg/dL D. 4.9 mg/dL

C. 2.4 mg/dL C To convert from milliequivalent per liter to milligrams per deciliter, first calculate the milliequivalent weight (equivalent weight expressed in milligrams), which is the atomic mass divided by the valence. Because magnesium is divalent, each mole has the charge equivalent of 2 mol of hydrogen. Then, multiply the milliequivalent per liter by the milliequivalent weight to convert to milligrams per liter. Next, divide by 10 to convert milligrams per liter to milligrams per deciliter. Milliequivalent weight Mg = 24.3 ÷ 2 = 12.15 mg/mEq 2.0 mEq/L × 12.15 mg/mEq = 24.3 mg/L 24.3 mg/L × 1.0 L/10.0 dL = 2.4 mg/dL

Convert 10.0 mg/dL calcium (atomic weight = 40.1) to International System of Units (SI). A. 0.25 B. 0.40 C. 2.5 D. 0.4

C. 2.5 C The SI unit is the recommended method of reporting clinical laboratory results. The SI unit for all electrolytes is millimole per liter. To convert from milligrams per deciliter to millimoles per liter, multiply by 10 to convert to milligrams per liter, then divide by the atomic mass expressed in milligrams. 10.0 mg/dL × 10.0 dL/1.0 L = 100.0 mg/L 100.0 mg/L × 1.0 mmol/40.1 mg = 2.5 mmol/L

Which of the following values is the threshold critical value (alert or action level) for low plasma potassium? A. 1.5 mmol/L B. 2.0 mmol/L C. 2.5 mmol/L D. 3.5 mmol/L

C. 2.5 mmol/L C The reference range for potassium is 3.6-5.4 mmol/L. However, values below 2.5 mmol/L require immediate intervention because below that level there is a grave risk of cardiac arrhythmia, which can lead to cardiac arrest. The upper alert level for potassium is usually 6.5 mmol/L, except for neonatal and hemolyzed samples. Above this level, there is danger of cardiac failure

What is the normal ratio of bicarbonate to dissolved carbon dioxide (HCO3 - :dCO2) in arterial blood? A. 1:10 B. 10:1 C. 20:1 D. 30:1

C. 20:1 C When the ratio of HCO3 -:dCO2 is 20:1, the log of salt/acid becomes 1.3. Substituting this in the Henderson-Hasselbalch equation and solving for pH gives pH = 6.1 + log 20; pH = 6.1 + 1.3 = 7.4. Acidosis results when this ratio is decreased, and alkalosis when it is increased.

Select the reagent needed in the coupling enzyme reaction used to generate a colored product in the cholesterol oxidase method for cholesterol. A. Cholestahexaene B. H2O2 C. 4-Aminoantipyrine D. Cholest-4-ene-3-one

C. 4-Aminoantipyrine C In the cholesterol oxidase method, cholesterol ester hydrolase converts cholesterol esters to free cholesterol by hydrolyzing the fatty acid from the C3-OH group. Cholesterol oxidase catalyzes the oxidation of free cholesterol at the C3-OH group forming cholest-4-ene-3-one and hydrogen peroxide. The peroxide is used in a peroxidase reaction to oxidize a dye (e.g., 4-aminoantipyrine), which couples to phenol, forming a red quinoneimine complex.

The protein composition of HDL is what percentage by weight? A. Less than 2% B. 25% C. 50% D. 90%

C. 50% C About 50% of the weight of HDL is protein, largely apo A-I and apo A-II. The HDL is about 30% phospholipid and 20% cholesterol by weight. The HDL binds and esterifies free cholesterol from cells and transports it to the liver, where it can be eliminated in the bile.

What is the PCO2 if the dCO2 is 1.8 mmol/L? A. 24 mm Hg B. 35 mm Hg C. 60 mm Hg D. 72 mm Hg

C. 60 mm Hg C Dissolved CO2 is calculated from the measured PCO2 × 0.0306, the solubility coefficient for CO2 gas in blood at 37°C. dCO2 = PCO2 × 0.03. Therefore, PCO2 = dCO2 /0.03 PCO2 = 1.8 mmol/L ÷ 0.03 mmol/L per mm Hg = 60 mm Hg

Select the most appropriate adult reference range for fasting blood glucose. A. 40-105 mg/dL (2.22-5.82 mmol/L) B. 60-140 mg/dL (3.33-7.77 mmol/L) C. 65-99 mg/dL (3.61-5.50 mmol/L) D. 75-150 mg/dL (4.16-8.32 mmol/L)

C. 65-99 mg/dL (3.61-5.50 mmol/L) C Reference ranges vary slightly depending upon method and specimen type. Enzymatic methods specific for glucose have an upper limit of normal no greater than 99 mg/dL. This is the cutoff value for impaired fasting plasma glucose (prediabetes) recommended by the American Diabetes Association. Although 65 mg/dL is considered the 2.5 percentile, a fasting level below 50 mg/dL is often seen without associated clinical hypoglycemia, and neonates have a lower limit of approximately 40 mg/dL owing to maternal insulin

What is the blood pH when the partial pressure of carbon dioxide (PCO2) is 60 mm Hg and the bicarbonate concentration is 18 mmol/L? A. 6.89 B. 7.00 C. 7.10 D. 7.30

C. 7.10 C Solve using the Henderson-Hasselbalch equation. pH = pK´ + log HCO3 -/(0.03 × PCO2), where pK´, the negative logarithm of the combined hydration and dissociation constants for dissolved CO2 and carbonic acid, is 6.1 and 0.03 is the solubility coefficient for CO2 gas. pH = 6.1 + log 18/(0.03 × 60) = 6.1 + log 18/1.8 pH = 6.1 + log 10. Because log 10 = 1, pH = 7.10

How many grams of sodium hydroxide (NaOH) are required to prepare 150.0 mL of a 5.0% w/v solution? A. 1.5 g B. 4.0 g C. 7.5 g D. 15.0 g

C. 7.5 g C A percent solution expressed in w/v (weight/volume) refers to grams of solute per 100.0 mL of solution. To calculate, multiply the percentage (as grams) by the volume needed (mL), then divide by 100.0 (mL) (5.0 g × 150.0 mL) ÷ 100.0 mL = 7.5 g

A new tumor marker for ovarian cancer is evaluated for sensitivity by testing serum samples from patients who have been diagnosed by staging biopsy as having malignant or benign lesions. The following results were obtained: Number of malignant patients who are positive for CA 125 = 21 out of 24 Number of benign patients who are negative for CA 125 = 61 out of 62 What is the sensitivity of the new CA 125 test? A. 98.4% B. 95.3% C. 87.5% D. 85.0%

C. 87.5% C Sensitivity is defined as the percentage of persons with the disease who have a positive test result. It is calculated as true positives (TP) divided by the sum of TP and false negatives (FN). % Sensitivity = (TP × 100) /TP + FN Sensitivity = (21 × 100) ÷ (21 + 3) = 87.5%

Calculate the pH of a solution of 1.5 × 10-5 M NH4OH. A. 4.2 B. 7.2 C. 9.2 D. 11.2

C. 9.2 C First, calculate the pOH of the solution. pOH = -Log [OH-] pOH = - Log 1.5 x 10-5 = 4.82 pH = 14 - pOH pH = 14 - 4.8 = 9.2

Which statement best describes the clinical utility of B-type natriuretic peptide (BNP)? A. Abnormal levels may be caused by obstructive lung disease B. A positive test indicates prior myocardial damage caused by AMI that occurred within the last 3 months C. A normal test result (<100 pg/mL) helps rule out congestive heart failure in persons with symptoms associated with coronary insufficiency D. A level above 100 pg/mL is not significant if evidence of congestive heart failure is absent

C. A normal test result (<100 pg/mL) helps rule out congestive heart failure in persons with symptoms associated with coronary insufficiency C B-type natriuretic peptide is a hormone produced by the ventricles in response to increased intracardiac blood volume and hydrostatic pressure. It is formed in the heart from a precursor peptide (preproBNP) by enzymatic hydrolysis, first forming proBNP followed by BNP and NT (N-terminal) proBNP which is not physiologically active. Both BNP and NT-proBNP are increased in persons with congestive heart failure (CHF). Levels are not increased in pulmonary obstruction, hypertension, edema associated with renal insufficiency, and other conditions that cause physical limitation and symptoms that overlap CHF. At a cutoff of <100 pg/mL the BNP test is effective in ruling out CHF. Diagnostic accuracy in distinguishing CHF from nonCHF ranges from 83%-95%. In addition, persons with ischemia who have an increased BNP are at greater risk for MI. The NTpro-BNP assay is similar in clinical value, and can be used for persons being treated with nesiritide, a recombinant form of BNP used to treat CHF.

Which instrument requires a highly regulated DC power supply? A. A spectrophotometer with a barrier layer cell B. A colorimeter with multilayer interference filters C. A spectrophotometer with a photomultiplier tube D. A densitometer with a photodiode detector

C. A spectrophotometer with a photomultiplier tube C When AC voltage regulators are used to isolate source lamp power, light output fluctuates as the voltage changes. Because this occurs at 60 Hz, it is not detected by eyesight or slow-responding detectors. Photomultiplier tubes are sensitive enough to respond to the AC frequency and require a DC-regulated power supply.

Which of the following statements regarding transaminases is true? A. ALT is often increased in muscular disease, pancreatitis, and lymphoma B. ALT is increased in infectious mononucleosis, but AST is usually normal C. ALT is far more specific for liver diseases than is AST D. Substrate depletion seldom occurs in assays of serum from hepatitis cases

C. ALT is far more specific for liver diseases than is AST D. Substrate depletion C ALT is far more specific for liver disease than AST. High ALT may result from nonhepatic causes such as AMI, muscle injury or disease, and severe hemolysis, but nonhepatic sources can be ruled out by a high direct bilirubin. Elevated ALT (e.g., >65 IU/L) is used along with immunologic tests for hepatitis to disqualify blood donors. AST is increased in muscle disease, MI, pancreatitis, and lymphoma. Both transaminases are moderately increased in infectious mononucleosis

What is the typical time course for plasma myoglobin following an AMI? A. Abnormal before 1 hour; peaks within 3 hours; returns to normal in 8 hours B. Abnormal within 3 hours; peaks within 6 hours; returns to normal in 18 hours C. Abnormal within 2 hours; peaks within 12 hours; returns to normal in 36 hours D. Abnormal within 6 hours; peaks within 24 hours; returns to normal in 72 hours

C. Abnormal within 2 hours; peaks within 12 hours; returns to normal in 36 hour C After AMI, myoglobin usually rises above the cutoff within 1-2 hours, peaks within 8-12 hours and returns to normal within 36 hours. Typically, levels reach a peak concentration that is 10-fold the upper reference limit. Since myoglobin is the first marker to become abnormal after an AMI, it should be measured on admission and if negative, again 2 hours later. If both samples are below the cutoff, the probability of an AMI having occurred is low. If the myoglobin is above the cutoff, a cardiac specific marker such as TnI, or TnT must be performed at some point to confirm the diagnosis

What is the typical time course for plasma TnI or TnT following an AMI? A. Abnormal within 3 hours; peaks within 12 hours; returns to normal in 24 hours B. Abnormal within 4 hours; peaks within 18 hours; returns to normal in 48 hours C. Abnormal within 4 hours; peaks within 24 hours; returns to normal in 1 week D. Abnormal within 6 hours; peaks within 36 hours; returns to normal in 5 days

C. Abnormal within 4 hours; peaks within 24 hours; returns to normal in 1 week C Troponin is a complex of three polypeptides that function as a regulator of actin and tropomyosin. The three subunits are designated TnC, TnI, and TnT. All are present in both cardiac and some skeletal muscles, but cardiac and skeletal isoforms of TnI and TnT can be differentiated by specific antiseras. TnI and TnT cardiac isoforms in plasma will at least double within 4-6 hours after MI, peak within 24 hours and usually remain elevated for 7-10 days. TnT and TnI have the same sensitivity, but TnT is more commonly elevated in renal failure patients. Both are increased slightly in unstable angina (chest pain while at rest) and cardiac ischemia.

In which condition is the LD most likely to be within normal limits? A. Hepatic carcinoma B. Pulmonary infarction C. Acute appendicitis D. Crush injury

C. Acute appendicitis C LD is increased slightly to moderately in most causes of liver disease. Smallest elevations are seen in obstructive jaundice and highest in hepatic carcinoma and toxic hepatitis, where levels can reach 10-fold the upper reference limit. LD is also increased in crush injury and muscular dystrophies due to skeletal muscle damage, and in pulmonary infarction owing to embolism formation. Amylase is increased in a majority of persons with acute appendicitis, but LD is not.

Which of the following procedures can be used to detect proportional error in a new method for glucose? A. Compare the standard deviation of 40 patient samples to the hexokinase method B. Measure a mixture made from equal parts of normal and high-QC sera C. Add 5.0 mg of glucose to 1.0 mL of a serum of known concentration and measure D. Compare the mean of 40 normal samples to the hexokinase method

C. Add 5.0 mg of glucose to 1.0 mL of a serum of known concentration and measure C Proportional error is percentage deviation from the expected result, and affects the slope of the calibration curve. It causes a greater absolute error (loss of accuracy) as concentration increases. It is measured by a recovery study in which a sample is spiked with known amounts of analyte. In the example, the concentration should increase by 500 mg/dL.

Which of the following is the most effective serial sampling time for ruling out AMI using both myoglobin and a cardiac specific marker in an emergency department environment? A. Admission and every hour for the next 3 hours or until positive B. Admission, 2 hours, 4 hours, and 6 hours or until positive C. Admission, 3 hours, 6 hours, and a final sample within 12 hours D. Admission and one sample every 8 hours for 48 hours

C. Admission, 3 hours, 6 hours, and a final sample within 12 hours C Since the time between the onset of symptoms and arrival in the emergency department is often speculative, serial measurement of cardiac markers is required in order to rule out AMI. Since myoglobin is the first marker to rise after AMI, if used it should be measured on admission. Since TnI, TnT, and CK-MB are more cardiac specific, at least one should be measured starting at 3 hours postadmission, and again at 6 hours postadmission. If all results are negative to this point, a final assay should be performed 6-12 hours postadmission to conclusively rule out the possibility of AMI and evaluate the short-term risk of AMI

Which of the following laboratory tests is a marker for ischemic heart disease? A. CK-MB isoforms B. Myosin light chain 1 C. Albumin cobalt binding D. Free fatty acid binding protein

C. Albumin cobalt binding C When heart muscle suffers reversible damage as a result of oxygen deprivation, free radicals are released from the cells and bind to circulating albumin. The albumin is modified at the N-terminus, causing a reduced ability to bind certain metals. This ischemia-modified albumin can be measured by its inability to bind cobalt. An excess of cobalt is incubated with plasma followed by addition of dithiothreitol. The sulfhydryl compound complexes with the free cobalt, forming a colored complex. The absorbance of the reaction mixture is directly proportional to the ischemia-modified albumin concentration. In addition to ischemia-modified albumin, glycogen phosphorylase-BB is a marker for ischemia because it is released from heart muscle during an ischemic episode. Myosin light chains and fatty acid-binding protein are released from necrotic heart tissue in the early stages of AMI.

The term δ-bilirubin refers to: A. Water-soluble bilirubin B. Free unconjugated bilirubin C. Bilirubin tightly bound to albumin D. Direct-reacting bilirubin

C. Bilirubin tightly bound to albumin C HPLC separates bilirubin into four fractions: α = unconjugated, β = monoglucuronide, γ = diglucuronide, and δ = irreversibly albumin bound. δ Bilirubin is a separate fraction from the unconjugated bilirubin, which is bound loosely to albumin. δ Bilirubin and conjugated bilirubin react with diazo reagent in the direct bilirubin assay

Which enzyme is most likely to be elevated in the plasma of a person suffering from a muscle wasting disorder? A. 5´-Nucleotidase B. Pseudocholinesterase C. Aldolase D. Glutamate dehydrogenase

C. Aldolase C 5'-Nucleotidase is increased primarily in obstructive liver disease and liver cancer. When elevated along with ALP, it identifies the liver as the source of ALP. Glutamate dehydrogenase is increased in necrotic liver diseases along with transaminases, but because of its distribution it is elevated to a greater extent in toxic hepatitis and therefore is useful as a marker for halothane (anesthesia) toxicity. Aldolase is found in all tissues and is increased in many conditions including myocardial infarction, viral hepatitis, and myelocytic leukemia. However, like CK, the greatest increase is seen in skeletal muscle-wasting disease such as muscular dystrophies.

A person presents with a cushingoid appearance and an elevated 24-hour urinary cortisol level. The plasma adrenocotropic hormone (ACTH) is very elevated, and the physician suspects the cause is ectopic ACTH production. Which test would be most useful in substantiating this diagnosis? A. Plasma cortisol B. CA-50 C. Alkaline phosphatase isoenzymes D. AFP

C. Alkaline phosphatase isoenzymes C Most often, ectopic ACTH production occurs in lung cancer. Tumors of the lung are often associated with the production of placental-like alkaline phosphatase, and a positive finding would support the diagnosis of an ectopic (nonpituitary) source of ACTH. Many other tumor markers, including neuron-specific enolase and parathyroid hormone-related protein, are also increased in lung cancers. CA-50 (along with CA-19-9) shares the same antigenic determinant as Lewis A and is a marker for recurrence and treatment of gastrointestinal and pancreatic cancers. AFP is the predominant protein produced by the fetus, and plasma levels are increased primarily in yolk sac, liver, and testicular tumors.

SITUATION: A specimen for CK performed on an automated analyzer using an optimized Oliver-Rosalki method gives an error flag indicating substrate depletion. The sample is diluted 1:2 and 1:4 by the serial dilution technique and reassayed. After correcting for the dilution, the results are as follows: 1:2 Dilution = 3,000 IU/L 1:4 Dilution = 3,600 IU/L Dilutions are made a second time and assayed again but give identical results. What is the most likely explanation? A. The serum became contaminated prior to making the 1:4 dilution B. The wrong pipet was used to make one of the dilutions C. An endogenous competitive inhibitor is present in the serum D. An error has been made in calculating the enzyme activity of one of the two dilutions

C. An endogenous competitive inhibitor is present in the serum C When a competitive inhibitor is present in the serum, a dilution of the sample will cause an increase in the reaction rate by reducing the concentration of the inhibitor. Dilution of serum frequently increases the activity of CK and amylase. The same effect will occur when a smaller volume of serum is used in the assay because less inhibitor will be present in the reaction mixture.

SITUATION: A digoxin result from a stable patient with a normal electrocardiogram (EKG) is reported as 7.4 ng/mL (URL 2.6 ng/mL) using an immunofluorescent method. Renal function tests were normal and the patient was not taking any other medications. The assay was repeated and results were the same. The sample was frozen and sent to a reference laboratory for confirmation. The result was 1.6 ng/mL measured by a competitive chemiluminescent procedure. Which best explains the discrepancy in results? A. The fluorescent immunoassay was performed improperly B. Digoxin was lower by the chemiluminescent method because it is less sensitive C. An interfering substance was present that cross-reacted with the antibody in the fluorescent immunoassay D. Freezing the specimen caused lower results by converting the digoxin to an inactive metabolite

C. An interfering substance was present that cross-reacted with the antibody in the fluorescent immunoassay C An error was suspected because there was a discrepancy between the test result and the patient's clinical status (i.e., signs of digoxin toxicity such as ventricular arrhythmia were not present.) Somesubstances called DLIFs (digoxin-like immunologic factors) can cross-react with antibodies used to measure digoxin. The extent of interference varies with the source of anti-digoxin used. In addition, falsely elevated digoxin results may result from accidental ingestion of plant poisons such as oleandrin and from administration of Digibind, a Fab fragment against digoxin that is used to reverse digoxin toxicity.

The method for measuring iron or lead by plating the metal and then oxidizing it is called: A. Polarography B. Coulometry C. Anodic stripping voltometry D. Amperometry

C. Anodic stripping voltometry C Anodic stripping voltometry is used to measure lead and iron. The cation of the metal is plated onto a mercury cathode by applying a negative charge. The voltage of this electrode is reversed until the plated metal is oxidized back to a cation. Current produced by oxidation of the metal is proportional to concentration.

When should progesterone be measured when evaluating an adult female for anovulation? A. At the onset of menses B. During the first 7 days of the menstrual cycle C. At the midcycle just after LH peaks D. At the end of the menstrual cycle

C. At the midcycle just after LH peaks C Progesterone is often measured along with LH, FSH, estrogen, and prolactin to evaluate female infertility and dysmenorrhea. Progesterone is produced by the corpus luteum and levels are very low during the early follicular phase of the cycle. Progesterone is released by the corpus luteum following the LH surge that occurs 1-2 days prior to ovulation and is an indication that ovulation occurred. Low progesterone at midcycle indicates that ovulation did not occur. This is often the case in polyovarian cyst syndrome

Which statement about the measurement of carboxyhemoglobin is true? A. Treatment with alkaline dithionite is used to convert carboxyhemoglobin to oxyhemoglobin B. Oxyhemoglobin has no absorbance at 540 nm, but carboxyhemoglobin does C. Bichromatic analysis is required in order to eliminate interference by oxyhemoglobin D. Carboxyhemoglobin can be measured by potentiometry

C. Bichromatic analysis is required in order to eliminate interference by oxyhemoglobin C The absorbance spectras of oxy- and carboxyhemoglobin pigments overlap, and bichromatic or multichromatic analysis is required in order to accurately measure carboxyhemoglobin. In bichromatic analysis, oxyhemoglobin and methemoglobin are converted to deoxyhemoglobin by the addition of alkaline sodium dithionite. The ratio of absorbance at 541:555 nm is directly proportional to carboxyhemoglobin concentration. Percent carboxyhemoglobin is commonly determined from simultaneous absorbance measurements at 548, 568, and 578 nm, or other wavelength combinations, a process called oximetry.

Drugs rapidly infused intravenously usually follow which elimination model? A. One compartment, first order B. One compartment, logarithmic C. Biphasic or two compartment with serum level rapidly falling in the first phase D. Michaelis-Menton or concentration-dependent elimination

C. Biphasic or two compartment with serum level rapidly falling in the first phase C Drugs rapidly infused intravenously follow a two-compartment model of elimination. The central compartment is the blood and tissues that are well perfused. The second consists of tissues for which distribution of drug is time dependent. In determining the loading dose, the desired serum concentration should be multiplied by the volume of the central compartment to avoid toxic levels.

SITUATION: A peak blood level for orally administered theophylline (therapeutic range 8-20 mg/L) measured at 8 a.m. is 5.0 mg/L. The preceding trough level was 4.6 mg/L. What is the most likely explanation of these results? A. Laboratory error made on peak measurement B. Specimen for peak level was collected from wrong patient C. Blood for peak level was drawn too soon D. Elimination rate has reached maximum

C. Blood for peak level was drawn too soon C Sample collection time is critical for accurate therapeutic drug monitoring. Blood for trough levels must be collected immediately before the next dose. Blood collection time for peak levels must not occur prior to complete absorption and distribution of drug. This usually requires 1-2 hours for orally administered drugs. The therapeutic range for theophylline is 8-20 mg/L. These results are most consistent with a peak sample having been drawn prior to complete absorption of the drug.

Which statement best describes the relationship between luteinizing hormone (LH) and follicle-stimulating hormone (FSH) in cases of dysmenorrhea? A. Both are usually increased when there is pituitary adenoma B. Increases in both hormones and a decrease in estrogen signal a pituitary cause of ovarian failure C. Both hormones normally peak 1-2 days before ovulation D. In menopause, the LH level at the midcycle peak is higher than the level of FSH

C. Both hormones normally peak 1-2 days before ovulation C In women, serum or urine LH and FSH are measured along with estrogen and progesterone to evaluate the cause of menstrual cycle abnormalities and anovulation. Both hormones show a pronounced serum peak 1-2 days prior to ovulation and urine peak 20-44 hours before ovulation. Normally, the LH peak is sharper and greater than the FSH peak; however, in menopause, the FSH usually becomes higher than LH. In patients with primary ovarian failure, the LH and FSH are elevated because low estrogen levels stimulate release of luteinizing hormone-releasing hormone (LHRH) from the hypothalamus. Conversely, in pituitary failure, levels of FSH and LH are reduced, and this reduction causes a deficiency of estrogen production by the ovaries

The best method of analysis for serum PTH involves using antibodies that detect: A. The amino-terminal fragment of PTH B. Te carboxy-terminal end of PTH C. Both the amino-terminal fragment and intact PTH D. All fragments of PTH as well as intact hormone

C. Both the amino-terminal fragment and intact PTH C PTH is a polypeptide comprised of 84 amino acids. The biological activity of the hormone resides in the N-terminal portion of the polypeptide, but the hormone is rapidly degraded and produces N-terminal, middle, and C-terminal fragments. Fragments lacking the N-terminal portion are inactive. Immunoassays for PTH using antibodies to different portions of the polypeptide will give different results. The assay of choice is a two-site double-antibody sandwich method that measures only intact PTH and active fragments. Methods that use single antibodies may detect inactive as well as active PTH fragments and are not as specific for parathyroid disease.

Which of the following is the best analyte to monitor for recurrence of ovarian cancer? A. CA-15-3 B. CA-19-9 C. CA-125 D. CEA

C. CA-125 C CA-125 is an oncofetal antigen, meaning that it is produced by genes that are active during fetal development but minimally active after birth except in malignant tissues. This group includes α-fetoprotein (AFP), CEA, PSA, and the carbohydrate-associated antigens (CA). CA-15-3 (which shares the same antigenic determinant as CA-27.29) is used mainly to monitor breast cancer treatment and recurrence. CA-19-9 (which shares the same antigenic determinant as CA-50) is a glycoprotein shed from the surface of gastric, pancreatic, and colorectal cancer cells

Which of the following statements regarding the clinical use of CK-MB (CK-2) is true? A. CK-MB becomes elevated before myoglobin after an AMI B. CK-MB levels are usually increased in cases of cardiac ischemia C. CK-MB is more specific than myoglobin D. An elevated CK-MB is always accompanied by an elevated total CK

C. CK-MB is more specific than myoglobin C Serum myoglobin becomes abnormal within 1-2 hours after an acute myocardial infarction (AMI) before troponin and CK-MB. CK-MB becomes abnormal shortly after troponin I (TnI) or troponin T (TnT) when a URL of 4 μg/L is used, and peaks at around the same time following AMI. However, cardiac troponins remain elevated significantly longer than CK-MB after AMI, are not increased in crush injuries. There is less than 5 μg/L CK-MB in the serum of healthy adults, while the total CK ranges from 10-110 U/L. Consequently, an abnormal CK-MB can occur in the absence of an elevated total CK

Which statement regarding IFE is true? A. Serum containing a monoclonal protein should have a κ:λ ratio of 0.5 B. A monoclonal band seen with monospecific antiserum should not be visible in the lane where polyvalent antiserum or sulfosalicylic acid was added C. CSF should be concentrated 50- to100-fold before performing IFE D. When oligoclonal bands are seen in the CSF, they must also be present in serum to indicate multiple sclerosis

C. CSF should be concentrated 50- to100-fold before performing IFE C Any monoclonal precipitin band formed when heavy- or light-chain-specific antiserum reacts with a sample should also be found in the same position when sample is fixed with sulfosalicylic acid or reacted with polyvalent antihuman Ig. The normal free κ:λ ratio can vary between 0.26 and 1.65. In a monoclonal gammopathy, this ratio always heavily favors the light chain type of M protein. A diagnosis of multiple sclerosis is usually confirmed by demonstration of oligoclonal banding in the CSF, which is not present in the serum. CSF is usually concentrated 50-100 times to increase sensitivity.

Which reagent is used in the Jendrassik-Grof method to solubilize unconjugated bilirubin? A. 50% methanol B. N-butanol C. Caffeine D. Acetic acid

C. Caffeine C A polarity modifier is required to make unconjugated bilirubin soluble in diazo reagent. The Malloy-Evelyn method uses 50% methanol to reduce the polarity of the diazo reagent. Caffeine is used in the Jendrassik-Grof method. This method is recommended because it is not falsely elevated by hemolysis and gives quantitative recovery of both conjugated and unconjugated bilirubin.

A stat plasma lithium determined using an ion-selective electrode is measured at 14.0 mmol/L. Select the most appropriate course of action. A. Immediately report this result B. Check sample for hemolysis C. Call for a new specimen D. Rerun the lithium calibrators

C. Call for a new specimen C Lithium in excess of 2.0 mmol/L is toxic (in some laboratories 1.5 mmol/L is the upper therapeutic limit). A level of 14 mmol/L would not occur unless the sample were contaminated with lithium. This would most likely result from collection in a green-stoppered tube containing the lithium salt of heparin.

In which circumstances is a validation study (versus performing routine quality control) required? A. Instrument recalibration B. Source lamp or ion selective electrode change C. Change in reagent lot D. Change in calibrator lot

C. Change in reagent lot C All of the listed conditions except a change in the reagent lot number can be validated by assaying two levels of control material following the procedure. A change in reagent lot number may alter the test system more dramatically, especially when the reagent was subjected to storage and shipping conditions that alter its performance. Therefore, both controls and patient samples should be analyzed and the results compared to the reagent in current use using criteria determined by the total allowable error for the analyte.

In addition to phenylketonuria, maple syrup urine disease, and homocystinuria, what other aminoaciduria can be detected by tandem MS? A. Alkaptonuria B. Hartnup disease C. Citrullinemia D. Cystinuria

C. Citrullinemia C Most states use electrospray ionizationn tandem-mass spectroscopy (MS/MS), which can detect over 20 inborn errors of metabolism from a single blood spot. Typically, this includes phenylketonuria, tyrosinemia, maple syrup urine disease, homocystinuria, citrullinemia, and argininosuccinate acidemia. The latter two are errors of the urea cycle

Weighthich of the following conditions is characterized by primary hyperaldosteronism caused by adrenal adenoma, carcinoma, or hyperplasia? A. Cushing's syndrome B. Addison's disease C. Conn's syndrome D. Pheochromocytoma

C. Conn's syndrome C Conn's syndrome is characterized by hypertension, hypokalemia, and hypernatremia with increased plasma and urine aldosterone and decreased renin. Cushing's syndrome results from excessive production of cortisol, and Addison's disease from deficient production of adrenal corticosteroids. Pheochromocytoma is a tumor of chromaffin cells (usually adrenal) that produces catecholamines

What is the purpose of the saponification step used in the Abell-Kendall method for cholesterol measurement? A. Remove phospholipids B. Reduce sterol molecules structurally similar to cholesterol C. Convert cholesterol esters to free cholesterol D. Remove proteins that can interfere with color formation

C. Convert cholesterol esters to free cholesterol C The Abell-Kendall method is the reference method for cholesterol assay because differences in esterase activity and interference in the peroxidase step are potential sources of error in enzymatic assays. Saponification is performed to hydrolyze the fatty acid esters of cholesterol, forming free cholesterol. This is required because the reagents react more intensely with cholesterol esters than with free cholesterol. Saponification is followed by extraction of cholesterol in petroleum ether to separate it from proteins and interfering substances. The extract is reacted with sulfuric acid, acetic anhydride, and acetic acid (Liebermann-Burchard reagent), which oxidizes the cholesterol and forms a colored product.

An international unit (IU) of enzyme activity is the quantity of enzyme that: A. Converts 1 µmol of substrate to product per liter B. Forms 1 mg of product per deciliter C. Converts 1 µmol of substrate to product per minute D. Forms 1 µmol of product per liter

C. Converts 1 µmol of substrate to product per minute C The IU is a rate expressed in micromoles per minute. Activity is reported as IUs per liter (IU/L) or mIU/mL. The SI unit for enzyme activity is the katal (1 katal converts 1 mol of substrate to product in 1 second).

Analysis of normal and abnormal QCs performed at the beginning of the evening shift revealed a 22s error across levels for triglyceride. Both controls were within the 3s limit. The controls were assayed again, and one control was within the acceptable range and the other was slightly above the 2s limit. No further action was taken and the patient results that were part of the run were reported. Which statement best describes this situation? A. Appropriate operating procedures were followed B. Remedial evaluation should have been taken, but otherwise, the actions were appropriate C. Corrective action should have been taken before the controls were repeated D. The controls should have been run twice before reporting results

C. Corrective action should have been taken before the controls were repeated C Quality control limits are chosen to achieve a low probability of false rejection. For example, a 22s error occurs only once in 1,600 occurrences by chance. Therefore, such an error can be assumed to be significant. However, this does not mean the error will occur if the controls are repeated again. The error detection rate (power function) of the 22s rule is only about 30% for a single run. This means that there is a greater chance the repeated controls will be within range than outside acceptable limits. Therefore, controls should never be repeated until the test system is evaluated for potential sources of error. Calibration should have been performed prior to repeating the controls, and patient samples should have been evaluated to determine the magnitude of the error before reporting

Which form of hyperbilirubinemia is caused by an inherited absence of UDP-glucuronyl transferase? A. Gilbert's syndrome B. Rotor syndrome C. Crigler-Najjar syndrome D. Dubin-Johnson syndrome

C. Crigler-Najjar syndrome C Crigler-Najjar syndrome is a rare condition that occurs in two forms. Type 1 is inherited as an autosomal recessive trait and causes a total deficiency of UDP-glucuronyl transferase. Life expectancy is less than 1 year. Type 2 is an autosomal dominant trait and is characterized by lesser jaundice and usually the absence of kernicterus. Bilirubin levels can be controlled with phenobarbital, which promotes bilirubin excretion. Gilbert's syndrome is an autosomal recessive condition characterized by decreased bilirubin uptake and decreased formation of bilirubin diglucuronide. It is the most common form of inherited jaundice. UDP glucuronyl transferase activity is reduced owing to an increase in the number of AT repeats in the promoter region of the gene. Dubin-Johnson and Rotor syndromes are autosomal recessive disorders associated with defective delivery of bilirubin into the biliary system.

Which plot will give the earliest indication of a shift or trend? A. Levy-Jennings B. Tonks-Youden C. Cusum D. Histogram

C. Cusum C Cusum points are the algebraic sum of the difference between each QC result and the mean. The y axis is the sum of differences and the x axis is the run number. The center of the y axis is 0. Because QC results follow a random distribution, the points should distribute about the zero line. Results are out of control when the slope exceeds 45° or a decision limit (e.g., ±2.7s) is exceeded.

Referring to the Levy-Jennings chart, what is the first day in the month when the run should be rejected and patient results should be repeated? A. Day 6 B. Day 7 C. Day 8 D. Day 9

C. Day 8 C Although the trend is apparent across QC levels by day 7, the patient results would not be rejected until day 8 when the 41s rule is broken. An advantage to plotting control data is that trends can be identified before results are out of control and patient data must be rejected. In this case, corrective steps should have been implemented by day 7 to avoid the delay and expense associated with having to repeat the analysis of patient samples.

In polarography, the voltage needed to cause depolarization of the cathode is called the: A. Half-wave potential B. Isopotential point C. Decomposition potential D. Polarization potentia

C. Decomposition potential C In polarography, a minimum negative voltage must be applied to the cathode to cause reduction of metal ions (or O2) in solution. This is called the decomposition potential. It is concentration dependent (dilute solutions require greater negative voltage), and can be determined using the Nernst equation.

The onset of menopause is usually associated with what hormone changes? A. Decreased estrogen, testosterone, and androgens B. Decreased estrogen, FSH, LH, and progesterone C. Decreased estrogen and progesterone, and increased LH and FSH D. Decreased estrogen and progesterone, normal LH and FSH

C. Decreased estrogen and progesterone, and increased LH and FSH C In menopause, estrogen production decreases to where the menstrual cycle and ovarian folliclematuration stop. The decreased estrogen causes the pituitary release of LH and FSH. In menopause, the FSH at midcycle is higher than LH. The increased LH causes the ovaries to secrete testosterone and androgens.

Which of the following is likely to occur first in iron deficiency anemia? A. Decreased serum iron B. Increased TIBC C. Decreased serum ferritin D. Increased transferrin

C. Decreased serum ferritin C Body stores must be depleted of iron before serum iron falls. Thus, serum ferritin falls in the early stages of iron deficiency, making it a more sensitive test than serum iron in uncomplicated cases. Ferritin levels are low only in iron deficiency. However, concurrent illness such as malignancy, infection, and inflammation may promote ferritin release from the tissues, causing

What role do CTx and NTx play in the management of osteoporosis? A. Increased urinary excretion is diagnostic of early stage disease B. Increased levels indicate a low risk of developing osteoporosis C. Decreased urinary excretion indicates a positive response to treatment D. The rate of urinary excretion correlates with the stage of the disease

C. Decreased urinary excretion indicates a positive response to treatment C Markers for both bone formation and resorption are used to monitor treatment for osteoporosis. Serum and urinary measurements of CTx and NTx and urinary deoxypyridinoline are used to monitor medications such as biphosphonates that inhibit bone resorption. Levels fall with successful treatment. DEXA scan, an x-ray procedure based on subtraction of surrounding tissue, is the most sensitive diagnostic test for osteoporosis and can show bone loss as small as 1%. However, it takes months before a DEXA scan shows increased bone remodeling following treatment.

Which of the following serum protein electrophoresis results suggests an acute inflammatory process? Albumin/ α-1 /α2 /β /γ A. Decreased, Increased, Decreased, Normal, Normal B. Normal, Increased, Normal, Increased, Increased C. Decreased, Increased, Increased, Normal, Normal D. Increased, Increased, Increased, Increased, Increased

C. Decreased, Increased, Increased, Normal, Normal C Acute inflammation is characterized by increased production of acute phase proteins. These include α1-antitrypsin, α1-acid glycoprotein, α1-antichymotrypsin, and haptoglobin. Albumin is slightly decreased. γ- and β fractions are normal.

Which of the following methods is most useful in order to detect sample misidentification? A. Cumulative summation B. Critical limit C. Delta limit D. Significant change limit

C. Delta limit C Cumulative summation is a statistical method used in quality assurance to detect a trend in QC results. Critical limits are used to define when medical intervention is likely needed to prevent injury or death. The significant change limit is the difference in test results that is medically significant, or that which cannot be attributed to the sum of normal physiological and analytical variation. The delta limit (delta check) determines whether the difference between two measurements usually 24-48 hours apart exceeds the expected. Although this can result from an abrupt change in the patient's status, other causes are sample misidentification, contamination, and random error. Delta limits are expressed in percent and vary depending on analyte stability.

A gastric fluid from a patient suspected of having taken an overdose of amphetamine is sent to the laboratory for analysis. The technologist should: A. Perform an EMIT assay for amphetamine B. Refuse the sample and request serum or urine C. Dilute 1:10 with H2O and filter; perform TLC for amphetamines D. Titrate to pH 7.0, then follow procedure for measuring amphetamine in urine

C. Dilute 1:10 with H2O and filter; perform TLC for amphetamines C The gastric sample can be measured by TLC, but such a sample should not be used in place of serum or urine without documentation of acceptability by the reagent manufacturer or laboratory. A positive amphetamine result by a screening test such as TLC or immunoassay may be caused by a related drug which interferes, and therefore, the result should be confirmed by GC-MS if there is a medicolegal implication.

Which condition is a common cause of stray light? A. Unstable source lamp voltage B. Improper wavelength calibration C. Dispersion from second-order spectra D. Misaligned source lamp

C. Dispersion from second-order spectra C Stray light is caused by the presence of any light other than the wavelength of measurement reaching the detector. It is most often caused by second-order spectra, deteriorated optics, light dispersed by a darkened lamp envelope, and extraneous room light.

If too small a peak-trough difference is seen for a drug given orally, then: A. The dose should be decreased B. Time between doses should be decreased C. Dose interval should be increased D. Dose per day and time between doses should be decreased

C. Dose interval should be increased C Increasing the dosing interval will reduce the trough concentration of the drug, and increasing the dose will increase the peak concentration of the drug, resulting in a greater peak trough difference. The peak-trough ratio is usually adjusted to 2 with the dose interval set to equal the drug half-life. Under these conditions, both peak and trough levels often fall within the therapeutic range.

Which condition is caused by deficient secretion of bilirubin into the bile canaliculi? A. Gilbert's disease B. Neonatal hyperbilirubinemia C. Dubin-Johnson syndrome D. Crigler-Najjar syndrome

C. Dubin-Johnson syndrome C Dubin-Johnson syndrome is an autosomal recessive condition arising from mutation of an ABC transporter gene. It produces mild jaundice from accumulation of conjugated bilirubin that is not secreted into the bile canaliculi. Total and direct bilirubin are elevated, but other liver function is normal. Rotor syndrome is an autosomal recessive condition that also results in retention of conjugated bilirubin. The mechanism in Rotor syndrome is unknown, and like Dubin-Johnson syndrome it is commonly asymptomatic. It can be differentiated from Dubin-Johnson syndrome by the pattern of urinary coproporphyrin excretion and because it produces no black pigmentation in the liver.

Which method is the most useful when screening for errors of amino and organic acid metabolism? A. Two-dimensional thin-layer chromatography B. Gas chromatography C. Electrospray ionization tandem-mass spectroscopy D. Inductively charged coupled-mass spectroscopy

C. Electrospray ionization tandem-mass spectroscopy C While two-dimensional thin-layer chromatography can separate both amino and organic acids, it is not sufficiently sensitive for newborn screening. Electrospray ionization allows a small alcohol-extracted whole-blood sample to be analyzed by two mass spectrometers without prior separation by liquid or gas chromatography. Disorders of both organic and fatty acid metabolism are identified by the specific pattern of acylcarnitine ions produced. Amino acids are detected as amino species that have lost a carboxyl group during ionization, a process called neutral loss.

Which of the following statements regarding ALP is true? A. All isoenzymes of ALP are antigenically distinct and can be identified by specific antibodies B. Highest serum levels are seen in intrahepatic obstruction C. Elevated serum ALP seen with elevated GGT suggests a hepatic source D. When jaundice is present, an elevated ALP suggests acute hepatitis

C. Elevated serum ALP seen with elevated GGT suggests a hepatic source C ALP isoenzymes can result from different genes or from modification of a common gene product in the tissues. Some differ mainly in carbohydrate content and cannot be identified by immunologic methods. Highest levels of ALP are seen in Paget's disease of bone, where ALP can be as high as 25 times the URL. GGT in serum is derived from the hepatobiliary system and is increased in alcoholic hepatitis and hepatobiliary obstruction. It is not increased in diseases of bone or in pregnancy. When the increase in GGT is twofold higher than the increase in ALP, the liver is assumed to be the source of the elevated ALP. Serum ALP is a sensitive marker for extrahepatic obstruction, which causes an increase of approximately 10 times the URL. A lesser increase is seen in intrahepatic obstruction. ALP is only mildly elevated in acute hepatitis as a result of accompanying obstruction

The EMIT for drugs of abuse uses an: A. Antibody conjugated to a drug B. Enzyme conjugated to an antibody C. Enzyme conjugated to a drug D. Antibody bound to a solid phase

C. Enzyme conjugated to a drug C In EMIT, enzyme-labeled drug competes with drug in the sample for a limited amount of reagent antibodies. When antibody binds to the enzyme-drug conjugate, it blocks the catalytic site of the enzyme. Enzyme activity is directly proportional to sample drug concentration because the quantity of unbound drug-enzyme conjugate will be highest when drug is present in the sample.

A serum thyroid panel reveals an increase in total T4, normal TSH, and normal free T4. What is the most likely cause of these results? A. Primary hyperthyroidism B. Secondary hyperthyroidism C. Euthyroid with increased thyroxine-binding protein D. Subclinical hypothyroidism

C. Euthyroid with increased thyroxine-binding protein C Patients with a normal TSH are euthyroid, and most commonly an increase in total T4 in these patients is caused by an increase in TBG. An increase in TBG causes an increase in total T4 but not free T4. Subclinical hypothyroidism is usually associated with a high TSH, but normal free T3 and free T4. When TSH is indeterminate, the diagnosis is made by demonstrating an exaggerated response to the TRH stimulation test.

When calibrating a pH meter, unstable readings occur for both pH 7.00 and 4.00 calibrators, although both can be set to within 0.1 pH unit. Select the most appropriate course of action. A. Measure the pH of the sample and report to the nearest 0.1 pH B. Replace both calibrators with unopened buffers and recalibrate C. Examine the reference electrode junction for salt crystals D. Move the electrodes to another pH meter and calibrate

C. Examine the reference electrode junction for salt crystals C Noise in pH measurements often results from a blocked junction between the reservoir of the reference electrode and test solution. This occurs when salt crystals collect at the junction or when KCl concentration in the reservoir increases due to evaporation of water. The fluid in the reference electrode should be replaced with warm deionized water. After the crystals have dissolved, the water is replaced with fresh reference electrolyte solution.

A lab measures total bilirubin by the Jendrassik-Grof bilirubin method with sample blanking. What would be the effect of moderate hemolysis on the test result? A. Falsely increased due to optical interference B. Falsely increased due to release of bilirubin from RBCs C. Falsely low due to inhibition of the diazo reaction by hemoglobin D. No effect due to correction of positive interference by sample blanking

C. Falsely low due to inhibition of the diazo reaction by hemoglobin C The sample blank measures the absorbance of the sample and reagent in the absence of azobilirubin formation and corrects the measurement for optical interference caused by hemoglobin absorbing the wavelength of measurement. However, hemoglobin is an inhibitor of the diazo reaction and will cause falsely low results in a blank corrected sample. For this reason, direct bichromatic spectrophotometric methods are preferred when measuring bilirubin in neonatal samples, which are often hemolyzed.

Which instrument requires a primary and secondary monochromator? A. Spectrophotometer B. Atomic absorption spectrophotometer C. Fluorometer D. Nephelometer

C. Fluorometer C A fluorometer uses a primary monochromator to isolate the wavelength for excitation, and a secondary monochromator to isolate the wavelength emitted by the fluorochrome.

What is the confirmatory method for measuring drugs of abuse? A. HPLC B. Enzyme-multiplied immunoassay technique (EMIT) C. Gas chromatography with mass spectroscopy (GC-MS) D. TLC

C. Gas chromatography with mass spectroscopy (GC-MS) C GC-MS determines the mass spectrum of the compounds eluting from the analytic column. Each substance has a unique and characteristic spectrum of mass fragments. This spectrum is compared to spectra in a library of standards to determine the percent match. A match of greater than 95% is considered confirmatory.

What type of detector is used in high-performance liquid chromatography with electrochemical detection (HPLC-ECD)? A. Calomel electrode B. Conductivity electrode C. Glassy carbon electrode D. Polarographic electrode

C. Glassy carbon electrode C HPLC-ECD uses a glassy carbon measuring electrode and a silver-silver chloride reference. The analyte is oxidized or reduced by holding the glassy carbon electrode at a positive voltage (oxidization) or negative voltage (reduction). The resulting current flow is directly proportional to concentration. Phenolic groups such as catecholamines can be measured by HPLC-ECD.

Which of the following enzymes is common to all enzymatic methods for triglyceride measurement? A. Glycerol phosphate oxidase B. Glycerol phosphate dehydrogenase C. Glycerol kinase D. Pyruvate kinase

C. Glycerol kinase C All enzymatic triglyceride methods require lipase to hydrolyze triglycerides, and glycerol kinase to phosphorylate glycerol, forming glycerol-3 phosphate. The most common method couples glycerol kinase with glycerol phosphate oxidase and peroxidase. 1. Triglyceride + H2O -Lipase-> glycerol + fatty acids 2. Glycerol + ATP -GK-> glycerol-3-phosphate + ADP 3. Glycerol-3-phosphate + O2 -GPO-> dihydroxyacetone phosphate + H2O2 H2O2 + phenol + 4-aminophenazone -Px -> quinoneimine dye + H2O GK = glycerol kinase; GPO = glycerol phosphate oxidase; Px = peroxidase

Which of the following contributes the most to the serum total CO2? A. PCO2 B. dCO2 C. HCO3- D. Carbonium ion

C. HCO3- C The total CO2 is the sum of the dCO2, H2CO3 (carbonic acid or hydrated CO2), and bicarbonate (as mainly NaHCO3). When serum is used to measure total CO2, the dCO2 is insignificant because all the CO2 gas has escaped into the air. Therefore, serum total CO2 is equivalent to the bicarbonate concentration. Total CO2 is commonly measured by potentiometry. An organic acid is used to release CO2 gas from bicarbonate and pCO2 is measured with a Severinghaus electrode. Alternately, bicarbonate can be measured by an enzymatic reaction using phosphoenol pyruvate carboxylase. The enzyme forms oxaloacetate and phosphate from phosphoenol pyruvate and bicarbonate. The oxaloacetate is reduced to malate by malate dehydrogenase and NADH is oxidized to NAD+. The negative reaction rate is proportional to plasma bicarbonate concentration.

Which tumor marker is used to determine the usefulness of trastuzumab (Herceptin) therapy for breast cancer? A. PR B. CEA C. HER-2/neu D. Myc

C. HER-2/neu C Trastuzumab is an antibody to the HER-2/neu gene product, a tyrosine kinase receptor protein. HER2/neu is an oncogene that is overexpressed in some breast cancers. Overexpression is associated with a more aggressive clinical course but responds to treatment with trastuzumab, which blocks the attachment of growth factor to the receptor. The progesterone receptor, like the ER, is used to identify persons with breast cancer who are more likely to respond to estrogen suppression therapy. Myc is a group of oncogenes that are activated in various cancers, including lung, breast, colon, stomach, leukemia, and lymphoma. HER-2/neu is measured in plasma by immunoassay. ER, PR, and myc are measured in tissue and not plasma using immunohistological stains or FISH.

Which of the following liver diseases produces the highest levels of transaminases? A. Hepatic cirrhosis B. Obstructive jaundice C. Hepatic cancer D. Alcoholic hepatitis

C. Hepatic cancer C Elevation of transaminases is greatest in acute hepatitis (20-50 × URL). Levels are moderately elevated (5-10 × URL) in hepatic cancer. They are slightly elevated (2-5 × URL) in chronic hepatitis, hepatic cirrhosis, alcoholic hepatitis, and obstructive jaundice.

Which of the following conditions is associated with "β-γ bridging"? A. Multiple myeloma B. Malignancy C. Hepatic cirrhosis D. Rheumatoid arthritis

C. Hepatic cirrhosis C Hepatic cirrhosis produces a polyclonal gammopathy associated with a high IgA level. This obliterates the valley between β and γ zones. Malignancy and rheumatoid arthritis produce polyclonal gammopathies classified as chronic inflammatory or delayed response patterns. Multiple myeloma produces a zone of restricted mobility usually in the γ, but sometimes in the β- or α2-region

Blood ammonia levels are usually measured in order to evaluate: A. Renal failure B. Acid-base status C. Hepatic coma D. Gastrointestinal malabsorption

C. Hepatic coma C Hepatic coma is caused by accumulation of ammonia in the brain as a result of liver failure. The ammonia increases central nervous system pH and is coupled to glutamate, a central nervous system neurotransmitter, forming glutamine. Blood and cerebrospinal fluid ammonia levels are used to distinguish encephalopathy caused by cirrhosis or other liver disease from nonhepatic causes and to monitor patients with hepatic coma.

Which statement about ammonia is true? A. Normally, most of the plasma ammonia is derived from peripheral blood deamination of amino acids B. Ammonia-induced coma can result from salicylate poisoning C. Hepatic coma can result from Reye's syndrome D. High plasma ammonia is usually caused by respiratory alkalosis

C. Hepatic coma can result from Reye's syndrome C Ammonia produced in the intestines from the breakdown of proteins by bacterial enzymes is the primary source of plasma ammonia. Most of the ammonia absorbed from the intestines is transported to the liver via the portal vein and converted to urea. Blood ammonia levels will rise in any necrotic liver disease including hepatitis, Reye's syndrome, and drug-induced injury such as acetaminophen poisoning. In hepatic cirrhosis, shunting of portal blood to the general circulation causes blood ammonia levels to rise. Ammonia crosses the blood- brain barrier, which accounts for the frequency of central nervous

In familial β dyslipoproteinemia (formerly type III hyperlipoproteinemia), which lipoprotein accumulates? A. Chylomicrons B. VLDL C. IDL D. VLDL

C. IDL C IDLs have roughly equal amounts of cholesterol and triglyceride. The IDL has a density of about 1.006-1.020, causing it to float on the 1.063 density potassium bromide solution used to recover LDL by ultracentrifugation. IDL has faster electrophoretic mobility on agarose than beta lipoprotein. These observations gave rise to the terms "floating beta" and "broad beta," respectively. Familial dysbetalipoproteinemia is in part caused by a polymorphism of apoE (apo E2) that has poor affinity for the apo-E receptor on hepatocytes. Not all persons with the homozygous polymorphism develop the disease; thus, other factors are necessary for the accumulation of IDL.

Hemoglobin electrophoresis performed on agarose at pH 8.8 gives the following results: A2 Position S Position F Position A Position 35% 30% 5% 30% All components of the Hgb C, S, F, A control hemolysate were within the acceptable range. What is the most likely cause of this patient's result? A. HgbLepore B. Hgb S-β-thalassemia (Hgb S/β+) C. Hgb SC disease post-transfusion D. Specimen contamination

C. Hgb SC disease post-transfusion C HemoglobinLepore results from a hybridization of the β and δ genes and produces a pattern that is similar to Hgb S trait (AS), except that the quantity of HgbLepore at the Hgb S position is below 20%. Hemoglobin S-β-thalassemia minor results in an increase in Hgb A2 (and possibly Hgb F) because there is reduced transcription of the structurall normal β chain. However, the Hgb S should be greater than the Hgb A, and the amount at the Hgb A2 is far too high. The concentration of Hgb at the A2 position is too high to result from contamination or to be considered as Hgb A2. This pattern appears to express two abnormal Hgbs (Hgb S and C) as well as the normal adult Hgb A. Because inheritance of two abnormal β genes prohibits formation of normal Hgb A, this pattern would occur only if the patient has been transfused with normal RBCs. Hemoglobin SC disease usually produces almost equal amounts of Hgb C and S (and usually a slight increase in Hgb F), and is the most likely cause of these results. This could be confirmed by acid agar electrophoresis or isofocusing to identify the abnormal Hgbs, and review of the patient's medical record for evidence of recent blood transfusion

A quantitative urine glucose was determined to be 160 mg/dL by the Trinder glucose oxidase method. The sample was refrigerated overnight. The next day, the glucose is repeated and found to be 240 mg/dL using a polarographic method. What is the most likely cause of this discrepancy? A. Poor precision when performing one of the methods B. Contamination resulting from overnight storage C. High levels of reducing substances interfering with the Trinder reaction D. Positive interference in the polarographic method caused by hematuria

C. High levels of reducing substances interfering with the Trinder reaction C Urine often contains high levels of ascorbate and other reducing substances. These may cause significant negative bias when measuring glucose using a peroxidase-coupled method. The reductants compete with chromogen for H2O2.

Which of the following statements regarding total CK is true? A. Levels are unaffected by strenuous exercise B. Levels are unaffected by repeated intramuscular injections C. Highest levels are seen in Duchenne's muscular dystrophy D. The enzyme is highly specific for heart injury

C. Highest levels are seen in Duchenne's muscular dystrophy C Total CK is neither sensitive nor specific for AMI. An infarct can occur without causing an elevated total CK. Exercise and intramuscular injections cause a significant increase in total CK. Crush injuries and muscular dystrophy can increase the total CK up to 50 times the URL.

Which of the following is characteristic of type 2 diabetes mellitus? A. Insulin levels are consistently low B. Most cases require a 3-hour oral glucose tolerance test to diagnose C. Hyperglycemia is often controlled without insulin replacement D. The condition is associated with unexplained weight loss

C. Hyperglycemia is often controlled without insulin replacement C. Hyperglycemia is often controlled without insulin replacement C Type 2, or late-onset diabetes, is associated with a defect in the receptor site for insulin. Insulin levels may be low, normal, or high. Patients are usually obese and over 40 years of age, although the incidence is increasing in both children and young adults. The American Diabetes Association (ADA) recommends screening all adults for diabetes who are overweight and have one additional risk factor and all adults over age 45, and to retest them every 3 years, if negative. Patients do not require insulin to prevent ketosis and hyperglycemia can be controlled in most patients by diet and drugs that promote insulin release. Type 2 accounts for 80%-90% of all diabetes mellitus.

A patient has the following arterial blood gas results: pH = 7.56 PCO2 = 25 mm Hg PO2 = 100 mm Hg HCO3 - = 22 mmol/L These results are most likely the result of which condition? A. Improper specimen collection B. Prolonged storage C. Hyperventilation D. Hypokalemia

C. Hyperventilation C The pH is alkaline (reference range 7.35-7.45) and this can be caused by either low PCO2 or increased bicarbonate. This patient has a normal bicarbonate (reference range 22-26 mmol/L) and a low PCO2 (reference range 35-45 mm Hg). Low PCO2 is always caused by hyperventilation, and therefore, this is a case of uncompensated respiratory alkalosis. The acute stages of respiratory disorders are often uncompensated. Prolonged storage would cause the pH and PO2 to fall, and the PCO2 to rise. Hypokalemia causes alkalosis, but usually is associated with the retention of CO2 as compensation.

Which of the following conditions involving electrolytes is described correctly? A. Pseudohyponatremia occurs only when undiluted samples are measured B. Potassium levels are slightly higher in heparinized plasma than in serum C. Hypoalbuminemia causes low total calcium but does not affect Cai D. Hypercalcemia may be induced by low serum magnesium

C. Hypoalbuminemia causes low total calcium but does not affect Cai C When serum albumin is low, the equilibrium between bound and Cai is shifted, producing increased Cai . This inhibits release of PTH by negative feedback until the Cai level returns to normal. Potassium is released from platelets and leukocytes during coagulation, causing serum levels to be higher than plasma. Pseudohyponatremia is a measurement error caused by diluting samples containing excessive fat or protein. The colloids displace plasma water, resulting in less electrolytes being delivered into the diluent. Only ion-selective electrodes that measure whole blood or undiluted serum are unaffected. Magnesium is needed for release of PTH, and PTH causes release of calcium and magnesium from bone. Therefore, hypocalcemia can be associated with either magnesium deficiency or magnesium excess.

Hyperparathyroidism is most consistently associated with: A. Hypocalcemia B. Hypocalciuria C. Hypophosphatemia D. Metabolic alkalosis

C. Hypophosphatemia C Hyperparathyroidism causes increased resorption of calcium and decreased renal retention of phosphate. Increased serum calcium leads to increased urinary excretion. The distal collecting tubule of the nephron reabsorbs less bicarbonate as well as phosphate, resulting in acidosis

Which assay is used to confirm difficult cases of hypothyroidism? A. Free T3 assay B. Free thyroxine index C. Tyrotropin-releasing hormone (TRH) stimulation test D. TBG assay

C. Tyrotropin-releasing hormone (TRH) stimulation test C The TRH stimulation test is used to confirm borderline cases of abnormal thyroid function. In normal patients, intravenous injection of 500 μg of TRH causes a peak TSH response within 30 minutes. In patients with primary hypothyroidism, there is an exaggerated response (>30 mU/L). Patients with hyperthyroidism do not show the expected rise in TSH after TRH stimulation.

In which situation is the plasma or 24-hour urinary cortisol not consistent with the clinical picture? A. In pregnant patients B. In patients with a positive overnight dexamethasone suppression test C. In congenital adrenal hyperplasia D. In Cushing's syndrome caused by ectopic ACTH producing tumors

C. In congenital adrenal hyperplasia C Congenital adrenal hyperplasia (adrenogenital syndrome) results from a deficiency of an enzyme required for synthesis of cortisol. Approximately 90% of cases are caused by a deficiency of 21-hydroxylase blocking conversion of 17-α hydroxyprogesterone to 11-deoxycortisol. Most other cases are caused by 11-hydroxylase deficiency, which blocks conversion of 11-deoxycortisol to cortisol. Precursors of cortisol, usually either 17-α hydroxyprogesterone or 11-deoxycortisol are increased. This results in low serum cortisol levels, but high levels of these intermediates (mainly 17-ketogenic steroids). The two most common features of CAH are salt wasting caused by increased mineral corticoid activity and virilization due to increased androgens.

What method is used to introduce the sample into a mass spectrometer for analysis of a trace element? A. Electrospray ionization B. Laser desorption C. Inductively charged plasma (ICP) ionization D. Direct injection

C. Inductively charged plasma (ICP) ionization C Mass spectrometers can be used to measure trace metals, but the atoms need to be vaporized and ionized like molecules before they enter the mass filter. This is done by introducing the sample into a very hot plasma (6,000-10,000°K) called a torch. The torch is made by circulating argon through inner and outer quartz tubes. The tubes are wrapped with a coil of wire that receives a radio frequency. This creates current flow through the wire and a magnetic field at the torch end. Argon atoms are excited by the current and magnetic field and ionize. When the argon is ignited by a spark, it forms the plasma. The sample is mixed with argon at the other end to create an aerosol. When it reaches the torch, the solvent is evaporated and the energy from the torch and collisions with argon ions cause ejection of outershell electrons, forming cations of the element. ICP-MS is used to measure any trace element that readily forms cations.

Which condition is associated with the lowest percent saturation of transferrin? A. Hemochromatosis B. Anemia of chronic infection C. Iron deficiency anemia D. Noniron deficiency anemia

C. Iron deficiency anemia C Percent saturation = Serum Fe × 100/TIBC. Normally, transferrin is one-third saturated with iron. In iron deficiency states, the serum iron falls but transferrin rises. This causes the numerator and denominator to move in opposite directions, resulting in very low percent saturation (about 10%). The opposite occurs in hemochromatosis and sideroblastic anemia, resulting in an increased percent saturation

Treatment recommendations for patients with coronary heart disease are based upon measurement of which analyte? A. HDL cholesterol B. Apo-B100 C. LDL cholesterol D. Total cholesterol

C. LDL cholesterol C NECP has identified LDL cholesterol as the target of therapy for reducing the risk of heart attack because lowering LDL cholesterol has proven to be an effective intervention. The greater the risk of coronary heart disease, the lower the cutpoint for intervention. For persons at high risk (a 10-year risk of heart attack > 20%) the cutpoint is ≥ 100 mg/dL for initiation of statin therapy. For highest-risk persons (those that have acute coronary syndrome, and multiple or uncontrolled risk factors) the treatment goal is LDL cholesterol below 70 mg/dL

When performing a sweat chloride collection, which of the following steps will result in analytical error? A. Using unweighed gauze soaked in pilocarpine nitrate on the inner surface of the forearm to stimulate sweating B. Collecting more than 75 mg of sweat in 30 minutes C. Leaving the preweighed gauze on the inside of the arm exposed to air during collection D. Rinsing the collected sweat from the gauze pad using chloride titrating solution

C. Leaving the preweighed gauze on the inside of the arm exposed to air during collection C The sweat chloride procedure requires the application of pilocarpine to stimulate sweating, and the use of iontophoresis (application of 0.16-mA current for 5 minutes) to bring the sweat to the surface. After iontophoresis, the skin on the inner surface of the forearm is washed with deionized water and dried, and a preweighed pair of 2-in.2 pads is taped to the skin. During the 30-minute collection of sweat, the gauze must be completely covered to prevent contamination and loss of sweat by evaporation. The Gibson-Cooke reference method for sweat chloride uses the Schales and Schales method (titration by Hg[NO3]2 with diphenylcarbazone indicator) to assay 1.0 mL of sweat eluted from the gauze with 5 mL of water. A Cotlove chloridometer is often used to measure sweat chloride. The sweat is eluted from the gauze with the titrating solution to facilitate measurement. Alternatively, a macroduct collection system may be used that does not require weighing. A minimum mass of 75 mg sweat is required for collection in gauze and 15 μL sweat for collection in macroduct tubing

Select the five pharmacological parameters that determine serum drug concentration. A. Absorption, anabolism, perfusion, bioactivation, excretion B. Liberation, equilibration, biotransformation, reabsorption, elimination C. Liberation, absorption, distribution, metabolism, excretion D. Ingestion, conjugation, integration, metabolism, elimination

C. Liberation, absorption, distribution, metabolism, excretion C Liberation is the release of the drug and absorption is the transport of drug from the site of administration to the blood. The percent of drug absorption and the rate of absorption determine the bioavailable fraction, f. This is the fraction of the dose that reaches the blood. Distribution refers to the delivery of the drug to the tissues. It involves dilution and equilibration of the drug in various fluid compartments including the blood, and is influenced by binding to proteins and blood cells. Metabolism is the process of chemical modification of the drug by cells. This results in production of metabolites with altered activity and solubility. Excretion is the process by which the drug and its metabolites are removed from the body.

A patient's biochemistry results are: Na = 125 mmol/L Cl = 106 mmol/L K = 4.5 mmol/L TCO2 = 19 mmol/L chol = 240 mg/dL triglyceride = 640 mg/dL glucose = 107 mg/dL AST = 16 IU/L ALT = 11 IU/L amylase = 200 U/L Select the most likely cause of these results. A. The sample is hemolyzed B. Serum was not separated from cells in sufficient time C. Lipemia is causing in vitro interference D. The specimen is contaminated

C. Lipemia is causing in vitro interference C The triglyceride level is about five times normal, causing the sample to be lipemic. This will cause pseudohyponatremia (unbalanced electrolytes). Lipemia may cause a falsely high rate reaction when amylase is measured by turbidimetry; however, the high amylase may be associated with pancreatitis, which results in hyperlipidemia.

What is the lipid testing protocol for adults recommended by the National Cholesterol Education Program (NCEP) to evaluate risk for atherosclerosis beginning at age 20? A. Total cholesterol, fasting or nonfasting every year B. Total cholesterol, fasting, every 2 years C. Lipid profile, fasting, every 5 years D. LDL cholesterol, fasting, every 2 years

C. Lipid profile, fasting, every 5 years C Because LDL cholesterol, HDL cholesterol, VLDL cholesterol, and triglycerides are all risk factors for coronary artery disease, NCEP recommends a fasting lipid profile to include triglycerides, total cholesterol, HDL cholesterol, and LDL cholesterol be performed every 5 years beginning at age 20. However, because LDL cholesterol is the target of treatment, therapeutic goals are based on the LDL cholesterol. New guidelines recommend an LDL cholesterol goal below 70 mg/dL for the highest-risk persons

Which isoenzyme of ALP migrates farthest toward the anode when electrophoresed at pH 8.6? A. Placental B. Bone C. Liver D. Intestinal

C. Liver C Liver ALP isoenzymes migrate farthest toward the anode, but fast and slow variants occur. The slow liver ALP band is difficult to distinguish from placental and bone ALP. The order from cathode to anode is: -Renal→Intestinal→Bone→Placental→Liver + Improved separation of bone and liver isoenzymes can be achieved by incubating the serum with neuraminidase prior to electrophoresis. The enzyme reduces the sialic content of the bone isoenzyme, causing it to migrate at a slower rate.

Which of the following conditions will prevent any migration of proteins across an electrophoretic support medium such as agarose? A. Using too high a voltage B. Excessive current during the procedure C. Loss of contact between a buffer chamber and the medium D. Evaporation of solvent from the surface of the medium

C. Loss of contact between a buffer chamber and the medium C Movement of proteins is dependent upon the presence of a salt bridge that allows current to flow via transport of ions to the electrodes across the support medium. If the salt bridge is not intact, there will be no migration, even if voltage is maintained across the electrodes. For agarose and cellulose acetate, heat causes evaporation of solvent from the buffer. This increases the ionic strength, causing current to rise during the run. Excessive heat can damage the support medium and denature proteins. Power = E (voltage) × I (current) × t (time); since E = I × R (resistance), heat is proportional to the square of current (P = I 2 × R × t). Constant current or power mode is used for long runs to prevent heat damage.

Interference from other reducing substances can be partially eliminated in the Jaffe reaction by: A. Measuring the product at 340 nm B. Measuring the product with an electrode C. Measuring the timed rate of product formation D. Performing a sample blank

C. Measuring the timed rate of product formation C The Jaffe reaction is nonspecific; proteins and other reducing substances such as pyruvate, protein, and ascorbate cause positive interference. Much of this interference is reduced by using a timed rate reaction. Ketoacids react with alkaline picrate almost immediately, and proteins react slowly. Therefore, reading the absorbance at 20 and 80 seconds and using the absorbance difference minimizes the effects of those compounds. Creatinine can be measured using an amperometric electrode However, this requires the enzymes creatininase, creatinase, and sarcosine oxidase. The last enzyme produces hydrogen peroxide from sarcosine, which is oxidized. This produces current in proportion to creatinine concentration. Performing a sample blank does not correct for interfering substances that react with alkaline picrate.

In which condition would hypochloremia be expected? A. Respiratory alkalosis B. Metabolic acidosis C. Metabolic alkalosis D. All of these options

C. Metabolic alkalosis C Chloride is the major extracellular anion and is retained or lost to preserve electroneutrality. Low chloride will occur in metabolic alkalosis because excess bicarbonate is present. Low chloride also will occur in partially compensated respiratory acidosis because the kidneys compensate by increased retention of bicarbonate.

Given the serum protein electrophoresis pattern shown, which transaminase results would you expect? A. Within normal limits for both B. Marked elevation of both (20-50-fold normal) C. Mild elevations of both (2-5-fold normal) D. Marked elevation of AST but normal AL

C. Mild elevations of both (2-5-fold normal) C The protein electrophoresis and densitometric scan show a significantly reduced albumin and polyclonal gammopathy. The densitometric scan shows beta-gamma bridging that supports a diagnosis of hepatic cirrhosis. In this condition one would expect two- to fivefold increases of both transaminases with an ALT:AST ratio below 1

The term isocratic is used in high-performance liquid chromatography (HPLC) to mean the: A. Mobile phase is at constant temperature B. Stationary phase is equilibrated with the mobile phase C. Mobile phase consists of a constant solvent composition D. Flow rate of the mobile phase is regulated

C. Mobile phase consists of a constant solvent composition C An isocratic separation uses a single mobile phase of constant composition, pH, and polarity, and requires a single pump. Some HPLC separations use a gradient mobile phase to increase distance between peaks. Gradients are made by mixing two or more solvents using a controller to change the proportions of solvent components.

Which testing situation is appropriate for the use of point-of-care whole-blood glucose methods? A. Screening for type 2 diabetes mellitus B. Diagnosis of diabetes mellitus C. Monitoring of blood glucose control in type 1 and type 2 diabetics D. Monitoring diabetics for hyperglycemic episodes only

C. Monitoring of blood glucose control in type 1 and type 2 diabetics C The ADA does not recommend the use of whole-blood glucose monitors for establishing a diagnosis of diabetes or screening persons for diabetes. The analytical measurement range of these devices varies greatly, and whole blood glucose is approximately 10% lower than serum or plasma glucose. In addition, analytical variance is greater and accuracy less than for laboratory instruments. Whole blood glucose meters should be used by diabetics and caregivers to monitor glucose control and can detect both hyper- and hypoglycemic states that result from too little or too much insulin replacement. Therefore, postprandial monitoring with such a device is recommended for all persons who receive insulin therapy.

Which type of dietary fatty acid is not associated with an increase in serum LDL cholesterol production? A. Monounsaturated trans fatty acids B. Saturated fatty acids C. Monounsaturated cis fatty acids D. Monounsaturated trans Ω-9 fatty acids

C. Monounsaturated cis fatty acids C Polyunsaturated and cis monounsaturated fatty acids are not associated with increased production of LDL cholesterol. On the other hand, saturated and trans monounsaturated fatty acids are both associated with increased LDL. Cis fatty acids are those in which the H atoms belonging to the double-bonded carbons are on the same side of the molecule. Ω-9 (n-9) fatty acids are those with a double bond located 9 carbons from the terminal methyl group. Ω Fatty acids are associated with increased cholesterol, if the hydrogens attached to the double-bonded carbons are in the trans position.

What is the principle of the transcutaneous bilirubin assay? A. Conductivity B. Amperometric inhibition C. Multiwavelength reflectance photometry D. Infrared spectroscopy

C. Multiwavelength reflectance photometry C Measurement of bilirubin concentration through the skin requires the use of multiple wavelengths to correct for absorbance by melanin and other light-absorbing constituents of skin and blood. More than 100 wavelengths and multiple reflectance measurements at various sites may be used to derive the venous bilirubin concentration in mg/dL. Such devices have been shown to have a high specificity. They can be used to identify neonates with hyperbilirubinemia, and to monitor treatment.

Which substance is used in the CK assay to activate the enzyme? A. Flavin adenine dinucleotide (FAD) B. Imidazole C. N-acetylcysteine D. Pyridoxyl-5´-phosphate

C. N-acetylcysteine C In addition to Mg+2, CK requires a thiol compound to reduce interchain disulfide bridges and bind heavy metals that inactivate the enzyme. N-acetylcysteine is an activator of CK used for this purpose in the IFCC recommended method. Pyridoxyl-5´-phosphate is a prosthetic group of AST and ALT. FAD is a prosthetic group of glucose oxidase. Imidazole is used to buffer the CK reagent.

Select the equation describing the potential that develops at the surface of an ion-selective electrode. A. van Deemter equation B. van Slyke equation C. Nernst equation D. Henderson-Hasselbalch equation

C. Nernst equation C The van Deemter equation describes the relation between the velocity of mobile phase to column efficiency in gas chromatography. The Henderson-Hasselbalch equation is used to determine the pH of a solution containing a weak acid and its salt. van Slyke developed an apparatus to measure CO2 and O2 content using a manometer.

A person has an elevated 24-hour urinary homovanillic acid (HVA) and vanillymandelic acid (VMA). Urinary metanephrines, chromogranin A, and neuron specific enolase are also elevated but 5 hydroxyindoleacetic acid is within the reference range. What is the most likely diagnosis? A. Carcinoid tumors of the intestine B. Pheochromocytoma C. Neuroblastoma D. Pancreatic cancer

C. Neuroblastoma C Neuron-specific enolase is an isoenzyme containing two gamma polypeptides that are specific for nervous tissue and are found in neuroendocrine cells. Plasma levels are increased in neuroblastomas, carcinoid tumors, thyroid medullary carcinomas, and in some lung cancers and seminomas. Urinary VMA, catecholamines, and metanephrines are increased in both pheochromocytoma (a tumor of chromaffin cells) and neuroblastoma (also a tumor of neuroectodermal cells derived from the neural crest neuroblasts of the sympathetic ganglia). Urinary HVA is increased in about 75% of persons with neuroblastoma but is not usually increased in pheochromocytoma. Chromogranin A is a protein that inhibits release of catecholamines and is increased in pheochromocytoma, neuroblastoma, and carcinoid tumors. Urinary 5 hydroxyindoleacetic acid is increased in carcinoid tumors (enterochromaffin tumors).

Which of the following diseases is caused by a deficiency of sphingomyelinase? A. Gaucher disease B. Fabry disease C. Niemann-Pick disease D. Tay-Sachs disease

C. Niemann-Pick disease C The diseases mentioned result from inborn errors of lipid metabolism (lipidoses) caused by deficiency of an enzyme needed for lipid degradation. Specific lipids accumulate in the lysosomes. Niemann-Pick disease results from a deficiency of sphingomyelinase; Gaucher disease from β-glucocerebrosidase ; Fabry disease (sex linked) from α-galactosidase A; and Tay-Sachs from N acetylglucosaminidase A

Urinary urea measurements may be used for calculation of: A. Glomerular filtration B. Renal blood flow C. Nitrogen balance D. All of these options

C. Nitrogen balance C Because BUN is handled by the tubules, serum levels are not specific for glomerular filtration rate. Urea clearance is influenced by diet and liver function as well as renal function. Protein intake minus excretion determines nitrogen balance. A negative balance (excretion exceeds intake) occurs in stress, starvation, fever, cachexia, and chronic illness. Nitrogen balance = (Protein intake in grams per day ÷ 6.25) - (Urine urea nitrogen in grams per day + 4), where 4 estimates the protein nitrogen lost in the feces per day and dividing by 6.25 converts protein to protein nitrogen

Which tumor marker is associated with cancer of the urinary bladder? A. CA-19-9 B. CA-72-4 C. Nuclear matrix protein D. Cathepsin-D

C. Nuclear matrix protein C Nuclear matrix proteins (NMPs) are RNA-protein complexes. NMP-22 is shed into the urine in persons with bladder carcinoma and is about 25-fold higher than normal in this condition. It has a clinical sensitivity of about 70% but is likely to be negative when the tumor is low grade. Other markers used for detection of bladder cancer include bladder tumor associated analytes (BTAs), a variant of the complement factor H protein; cytokeratin-20, a variant cytokeratin (fibrous protein) in the cytoplasm of malignant bladder epithelium; and telomerase, an enzyme that adds nucleotides to the ends of chromosomes, preventing telomere degradation. The specificity of these tests varies from approximately 75%-80%. Bladder cancer can also be detected by FISH because it is associated with a high incidence of ploidy and other chromosomal abnormalities that can be detected by fluorescent-labeled DNA probes. FISH specificity is over 94%, and like the immunoassays its sensitivity is higher for high-grade tumors (approximately 78% for grade 2 and 94% for grade 3 cancers).

Which of the following assays is recommended as a screening test for colorectal cancer in persons over 50 years old? A. CEA B. AFP C. Occult blood D. Fecal trypsin

C. Occult blood C Bleeding in the gastrointestinal tract occurs during the early stages of colorectal cancer when treatment can be most effective. Although occult blood can be caused by many other GI problems, it is not associated with benign polyps and has a sensitivity of over 80% for detection of colorectal cancer. CEA is elevated in less than 60% of such cases. AFP is elevated in only about 5% of colon cancers. Fecal trypsin is not a marker for colorectal cancer, but α1-antitrypsin is present in the stool in a majority of malignant colon tumors owing to intestinal protein loss.

Which of the following is the most common cause of Cushing's syndrome? A. Pituitary adenoma B. Adrenal hyperplasia C. Overuse of corticosteroids D. Ectopic adrenocorticotropic hormone (ACTH) production by tumors

C. Overuse of corticosteroids C The most common cause of Cushing's syndrome is the administration of medications with cortisol or glucocorticoid activity. Excluding iatrogenic causes, approximately 60%-70% of Cushing's syndrome results from hypothalamic-pituitary misregulation and is called Cushing's disease. Adrenal adenoma or carcinoma (non-ACTH-mediated Cushing's syndrome) comprise about 20% of cases, and ectopic ACTH production accounts for 10%-20%

Which statement regarding the use of PTH is true? A. Determination of serum PTH level is the best screening test for disorders of calcium metabolism B. PTH levels differentiate primary and secondary causes of hypoparathyroidism C. PTH levels differentiate primary and secondary causes of hypocalcemia D. PTH levels are low in patients with pseudohypoparathyroidism

C. PTH levels differentiate primary and secondary causes of hypocalcemia C Serum Cai is the best screening test to determine if a disorder of calcium metabolism is present, and will distinguish primary hyperparathyroidism (high Cai) and secondary hyperparathyroidism (low Cai). PTH levels are used to distinguish primary and secondary causes of hypocalcemia. Serum PTH is low in primary hypocalcemia (which results from parathyroid gland disease), but is high in secondary hypocalcemia (e.g., renal failure). Serum PTH is also used for the early diagnosis of secondary hypocalcemia because PTH levels rise prior to a decrease in the serum Cai . Serum PTH is used to help distinguish primary hyperparathyroidism (high PTH) and hypercalcemia of malignancy (usually low PTH), and pseudohypoparathyroidism from primary hypoparathyroidism. Pseudohypoparathyroidism results from a deficient response to PTH and is associated with normal or elevated serum PTH

Which of the following is most often elevated in hypercalcemia associated with malignancy? A. Parathyroid-derived PTH B. Ectopic PTH C. Parathyroid hormone-related protein (PTHRP) D. Calcitonin

C. Parathyroid hormone-related protein (PTHRP) C PTHRP is a peptide produced by many tissues and normally present in the blood at a very low level. The peptide has an N-terminal sequence of eight amino acids that are the same as found in PTH and that will stimulate the PTH receptors of bone. Some malignancies (e.g., squamous, renal, bladder, and ovarian cancers) secrete PTHRP, causing hypercalcemia-associated malignancy. Because the region shared with PTH is small and poorly immunoreactive, the peptide does not cross-react in most assays for PTH. For this reason, and because tumors producing ectopic PTH are rare, almost all patients who have an elevated Cai and elevated PTH have primary hyperparathyroidism. The immunoassay for PTHRP will frequently be elevated in patients who have not yet been diagnosed with malignancy but have an elevated Cai, without an elevated serum PTH. Calcitonin is a hormone produced in the medulla of the thyroid that opposes the action of PTH. However calcitonin levels do not greatly influence the serum calcium. Assay of calcitonin is used exclusively to diagnose medullary thyroid cancer, which produces very high serum levels

What is the primary means of solute separation in HPLC using a C18 column? A. Anion exchange B. Size exclusion C. Partitioning D. Cation exchange

C. Partitioning C Stationary phases (column packings) used in HPLC separate solutes by multiple means, but in reverse-phase HPLC the relative solubility between the mobile phase and stationary phase is most important and depends upon solvent polarity, pH, and ionic strength.

Which component is required in a spectrophotometer in order to produce a spectral absorbance curve? A. Multiple monochromators B. A reference optical beam C. Photodiode array D. Laser light source

C. Photodiode array C There are two ways to perform spectral scanning for compound identification. One is to use a stepping motor that continuously turns the monochromator so that the wavelength aligned with the exit slit changes at a constant rate. A more practical method is to use a diode array detector. This consists of a chip embedded with as many as several hundred photodiodes. Each photodiode is aligned with a narrow part of the spectrum produced by a diffraction grating, and produces current proportional to the intensity of the band of light striking it (usually 1-2 nm in range). The diode signals are processed by a computer to create a spectral absorbance or transmittance curve.

The serum level of which of the following laboratory tests is decreased in both VDDR and VDRR? A. Vitamin D B. Calcium C. Pi D. Parathyroid hormone

C. Pi C Persons with VDDR and VDRR have a low Pi. However, persons with VDDR have decreased serum calcium, as well. Parathyroid hormone (PTH) is increased in persons with VDDR because calcium is the primary stimulus for PTH release, but not in persons with VDRR. Vitamin D levels vary depending upon the type of rickets and the vitamin D metabolite that is measured. 1,25(OH)D, the active form of vitamin D, is low in type 1 but high in type 2 VDDR. It may be either normal or low in VDRR.

Which of the following statements regarding the catecholamines is true? A. They are derived from tryptophan B. They are produced by the zona glomerulosa of the adrenal cortex C. Plasma levels show both diurnal and pulsed variation D. They are excreted in urine primarily as free catecholamines

C. Plasma levels show both diurnal and pulsed variation C Catecholamines—epinephrine, norepinephrine, and dopamine—are produced from the amino acid tyrosine by the chromaffin cells of the adrenal medulla. Plasma and urinary catecholamines are measured in order to diagnose pheochromocytoma. Symptoms include hypertension, headache, sweating, and other endocrine involvement. Plasma catecholamines are oxidized rapidly to metanephrines and VMA; only about 2% is excreted as free catecholamines. The zona glomerulosa is the outermost portion of the adrenal cortex, where aldosterone is mainly produced.

Which statement about the biuret reaction for total protein is true? A. It is sensitive to protein levels below 0.1 mg/dL B. It is suitable for urine, exudates, and transudates C. Polypeptides and compounds with repeating imine groups react D. Hemolysis will not interfere

C. Polypeptides and compounds with repeating imine groups react C The biuret reaction is not sensitive to protein levelsb below 0.1 g/dL and, therefore, is not sensitive enough for assays of total protein in CSF, urine, or transudates. Slight hemolysis does not cause falsely high results, if the absorbance of the Cu+2 protein complexes is measured bichromatically. However, frankly hemolyzed samples contain sufficient globin to cause positive interference. The reagent reacts with peptides containing at least two peptide bonds, but due to the high concentration of proteins in plasma relative to peptides present this reactivity causes insignificant bias.

In which case would eGFR derived from the plasma creatinine likely give a more accurate measure of GFR than measurement of plasma cystatin C? A. Diabetic patient B. Chronic renal failure C. Post-renal transplant D. Chronic hepatitis

C. Post-renal transplant C Cystatin C is eliminated almost exclusively by the kidneys and plasma levels are not dependent on age, sex, or nutritional status. However, plasma levels are affected by some drugs, including those used to prevent renal transplant rejection. Increased plasma levels have been reported in chronic inflammatory diseases and cancer. Formulas are available to calculate eGFR from plasma cystatin C, but unlike for creatinine, the formulas must be matched to the method of assay. The eGFR derived from cystatin C can detect a fall in GFR sooner and may be more sensitive for diabetic and other populations at risk for chronic kidney disease. As a screening test for eGFR, it has about the same predictive value as eGFR derived from creatinine.

SITUATION: Hgb electrophoresis is performed and all of the Hgbs have greater anodal mobility than usual. A fast Hgb (Hgb H) is at the edge of the gel and bands are blurred. The voltage is set correctly, but the current reading on the ammeter is too low. Select the course of action that would correct this problem. A. Reduce the voltage B. Dilute the buffer and adjust the pH C. Prepare fresh buffer and repeat the test D. Reduce the running time

C. Prepare fresh buffer and repeat the test C Increased mobility, decreased resolution, and low current result from low ionic strength. Reducing voltage will slow migration but will not improve resolution. Diluting the buffer will reduce the current, resulting in poorer resolution.

After staining a silica gel plate to determine the L/S ratio, the technologist notes that the lipid standards both migrated 1 cm faster than usual. The technologist should: A. Repeat the separation on a new silica gel plate B. Check the pH of the developing solvent C. Prepare fresh developing solvent and repeat the assay D. Reduce solvent migration time for all subsequent runs

C. Prepare fresh developing solvent and repeat the assay C TLC plates migrate in solvent until the front comes to 1 cm of the top of the plate. Separation of lipids on silica gel is based upon adsorption. Higher Rf values indicate greater solubility of lipids in the developing solvent. This may be caused by evaporation of H2O, lowering the polarity of the solvent

A patient's BUN is 60 mg/dL and serum creatinine is 3.0 mg/dL. These results suggest: A. Laboratory error measuring BUN B. Renal failure C. Prerenal failure D. Patient was not fasting

C. Prerenal failure C BUN is affected by renal blood flow as well as by glomerular and tubular function. When blood flow to the kidneys is diminished by circulatory insufficiency (prerenal failure), glomerular filtration decreases and tubular reabsorption increases due to slower filtrate flow. Because urea is reabsorbed, BUN levels rise higher than creatinine. This causes the BUN:creatinine ratio to be greater than 10:1 in prerenal failure

Which of the following enzymes is usually depressed in liver disease? A. Elastase-1 B. GLD C. Pseudocholinesterase D. Aldolase

C. Pseudocholinesterase C Pseudocholinesterase is found mainly in the liver and functions to hydrolyze acetylcholine. It is depressed by organophosphate insecticides and drugs that function as cholinesterase inhibitors and the serum assay is used to presumptively identify cases of insecticide poisoning. Levels of pseudocholinesterase are decreased in patients with liver disease as a result of depressed synthesis. In cirrhosis and hepatoma, there is a 50%-70% reduction in serum level and a 30%-50%reduction in hepatitis. Elastase-1 is a pancreatic digestive enzyme that breaks down connective tissue protein. Its level in feces is reduced in persons with pancreatic insufficiency. GLD is increased in necrotic jaundice, and aldolase in necrotic jaundice and muscle disease

Select the products formed from the forward reaction of ALT. A. Aspartate and alanine B. Alanine and α-ketoglutarate C. Pyruvate and glutamate D. Glutamine and NAD+

C. Pyruvate and glutamate C Because glutamate is a common product for transaminases, pyruvate (a three-carbon ketoacid) and glutamate would be generated from the transamination reaction between alanine and α-ketoglutarate.

SITUATION: A 6-year-old child being treated with phenytoin was recently placed on valproic acid for better control of seizures. After displaying signs of phenytoin toxicity including ataxia, a stat phenytoin is determined to be 15.0 mg/L (reference range 10 20 mg/L). A peak blood level drawn 5 hours after the last dose is 18.0 mg/L. The valproic acid measured at the same time is within therapeutic limits. Quality control is within acceptable limits for all tests, but the physician questions the accuracy of the results. What is the most appropriate next course of action? A. Repeat the valproic acid level using the last specimen B. Repeat the phenytoin on both trough and peak samples using a different method C. Recommend measurement of free phenytoin using the last specimen D. Recommend a second trough level be measured

C. Recommend measurement of free phenytoin using the last specimen C Phenytoin levels must be monitored closely because toxic drug levels can occur unexpectedly due to changing pharmacokinetics. Phenytoin follows a nonlinear rate of elimination, which means that clearance decreases as blood levels increase. At high blood levels, saturation of the hepatic hydroxylating enzymes can occur, causing an abrupt increase in the blood level from a small increase in dose. The drug half-life estimated from the two drug levels is approximately 15 hours, which is within the range expected for children, so decreased clearance is not likely the problem. Valproic acid competes with phenytoin for binding sites on albumin. Free phenytoin is the physiologically active fraction and is normally very low, so small changes in protein binding can cause a large change in free drug. For example, a 5% fall in protein binding caused by valproic acid can increase the free phenytoin level by 50%. This patient's free phenytoin level should be measured, and the dose of phenytoin reduced to produce a free drug level that is within the therapeutic range

Which statement regarding glycated (glycosylated) Hgb (G-Hgb) is true? A. Has a sugar attached to the C-terminal end of the β chain B. Is a highly reversible aminoglycan C. Reflects the extent of glucose regulation in the 8- to 12-week interval prior to sampling D. Will be abnormal within 4 days following an episode of hyperglycemia

C. Reflects the extent of glucose regulation in the 8- to 12-week interval prior to sampling C G-Hgb results from the nonenzymatic attachment of a sugar such as glucose to the N-terminal valine of the β chain. The reaction is nonreversible and is related to the time-averaged blood glucose concentration over the life span of the RBCs. There are three G-Hgb fractions designated A1a, A1b, and Alc. Hemoglobin A1c makes up about 80% of glycated hemoglobin, and is used to determine the adequacy of insulin therapy. The time-averaged blood glucose is approximated by the formula (G-Hgb × 33.3) - 86 mg/dL, and insulin adjustments can be made to bring this level to within reference limits. Also, glycated protein assay (called fructosamine) provides similar data for the period between 2 and 4 weeks before sampling.

Which condition results in metabolic acidosis with severe hypokalemia and chronic alkaline urine? A. Diabetic ketoacidosis B. Phenformin-induced acidosis C. Renal tubular acidosis D. Acidosis caused by starvation

C. Renal tubular acidosis C Metabolic acidosis can be caused by any condition that lowers bicarbonate. In nonrenal causes, the kidneys will attempt to compensate by increased acid excretion. However, in renal tubular acidosis (RTA), an intrinsic defect in the tubules prevents bicarbonate reabsorption. This causes alkaline instead of acidic urine. Excretion of bicarbonate as potassium bicarbonate (KHCO3) results in severe hypokalemia

Serum protein and immunofixation electrophoresis are ordered on a patient. The former is performed, but there is no evidence of a monoclonal protein. Select the best course of action. A. Perform quantitative Ig G, A, M B. Perform the IFE on the serum C. Report the result; request a urine sample for protein electrophoresis D. Perform IFE on the serum and request a urine sample for IFE

C. Report the result; request a urine sample for protein electrophoresis C An area of restricted mobility should be identified on serum protein electrophoresis before IFE is performed. About one out of four patients with multiple myeloma have monoclonal free λ or κ chains in urine only, and therefore, urine electrophoresis should be included in initial testing.

A patient's blood gas results are as follows: pH = 7.26 dCO2 = 2.0 mmol/L HCO3 - = 29 mmol/L These results would be classified as: A. Metabolic acidosis B. Metabolic alkalosis C. Respiratory acidosis D. Respiratory alkalosis

C. Respiratory acidosis Imbalances are classified as respiratory when the primary disturbance is with PCO2 because PCO2 is regulated by ventilation. PCO2 = dCO2/0.03 or 60 mm Hg (normal 35-45 mm Hg). Increased dCO2 will increase hydrogen ion concentration, causing acidosis. Bicarbonate is moderately increased, but a primary increase in NaHCO3 causes alkalosis. Thus, the cause of this acidosis is CO2 retention (respiratory acidosis), and it is partially compensated by renal retention of bicarbonate.

Quality control results for uric acid are as follows: ---Run- 1 Run- 2 Run- 3- Run 4- Mean- s QC1- 3.5- 3.8- 4.1- 4.2 mg/dL- 3.6 mg/dL- 0.40 QC2- 6.8- 7.2- 7.4- 7.5 mg/dL- 7.0 mg/dL- 0.25 Results should be reported from: A. Run 1 only B. Runs 1 and 2 C. Runs 1, 2, and 3 D. Runs 1, 2, 3, and 4

C. Runs 1, 2, and 3 C Although no single result exceeds the 2s limit, the 41s rule is broken on Run 4. This means that both QC1 and QC2 exceeded +1s on Run 3 and Run 4.

Which of the following statements regarding the naming of transaminases is true? A. Serum glutamic oxaloacetic transaminase (SGOT) is the older abbreviation for ALT B. Serum glutamic pyruvic transaminase (SGPT) is the older abbreviation for AST C. SGPT is the older abbreviation for ALT D. SGOT is the newer abbreviation for AST

C. SGPT is the older abbreviation for ALT C SGOT refers to the products measured in the in vitro reaction, and is more correctly named AST for the four-carbon amino acid substrate aspartate. SGPT is the older name referring to the products of the reaction for ALT. SGPT is more correctly named ALT for the three-carbon amino acid substrate alanine.

Which of the following enzymes can be used to measure plasma or serum salicylate? A. Peroxidase B. Salicylate esterase C. Salicylate hydroxylase D. p-Aminosalicylate oxidase

C. Salicylate hydroxylase C The enzymatic assay of salicylate uses salicylate hydroxylase, which reduces salicylate with NADH and forms catechol and NAD+. Salicylate can also be measured by HPLC and various immunoassays including EMIT. Salicylate toxicity causes an initial respiratory alkalosis because the drug stimulates the respiratory center. However, this is followed by metabolic acidosis as the drug is metabolized. Therefore, it is imperative to identify salicylate as the cause of toxicity before treatment of an acid-base imbalance caused by aspirin overdose.

Which of the following enzymes allows creatinine to be measured by coupling the creatinine amidohydrolase (creatininase) reaction to the peroxidase reaction? A. Glucose-6-phosphate dehydrogenase B. Creatinine iminohydrolase C. Sarcosine oxidase D. Creatine kinase

C. Sarcosine oxidase C The peroxidase-coupled enzymatic assay of creatinine is based upon the conversion of creatinine to creatine by creatinine amidohydrolase (creatininase). The enzyme creatinine amidinohydrolase (creatinase) then hydrolyzes creatine to produce sarcosine and urea. The enzyme sarcosine oxidase converts sarcosine to glycine producing formaldehyde and hydrogen peroxide. Peroxidase then catalyzes the oxidation of a dye (4-aminophenazone and phenol) by the peroxide forming a red-colored product. This method is more specific than the Jaffe reaction, which tends to overestimate creatinine by about 5% in persons with normal renal function.

What is the clinical utility of testing for serum prealbumin? A. Low levels are associated with increased free cortisol B. High levels are an indicator of acute inflammation C. Serial low levels indicate compromised nutritional status D. Levels correlate with glomerular injury in patients with diabetes mellitus

C. Serial low levels indicate compromised nutritional status C Prealbumin (also called transthyretin) is a small protein with a half-life of only 2 days. Serum levels fall rapidly in patients with deficient protein nutrition. As a result, prealbumin is used to detect malnutrition and to measure the patient's response to dietary supplementation. The cutpoint used to identify nutritional deficiency in elderly patients is usually 11 mg/dL. Prealbumin is usually measured by immunonephelometry.

Which test is used to distinguish Cushing's disease (pituitary Cushing's) from Cushing's syndrome caused by adrenal tumors? A. Low-dose overnight dexamethasone suppression B. Petrosal sinus sampling C. Serum ACTH D. Twenty-four-hour urinary free cortisol

C. Serum ACTH C Serum ACTH assays are very helpful in distinguishing the cause of Cushing's syndrome. Patients with adrenal tumors have values approaching zero. Patients with ectopic ACTH tumors have values greater than 200 pg/dL. Fifty percent of patients with Cushing's disease have high 8 a.m. ACTH levels (between 100-200 pg/dL). The high-dose dexamethasone suppression test is also used. Patients with Cushing's disease show more than 50% suppression of cortisol release after receiving an 8-mg dose of dexamethasone, but patients with adrenal tumors or ACTH producing tumors do not. Inferior petrosal sinus sampling (the petrosal sinuses drain the pituitary) is used to determine if a high ACTH is from the pituitary glands, or from an ectopic source.

Which type of filter is best for measuring stray light? A. Wratten B. Didymium C. Sharp cutoff D. Neutral density

C. Sharp cutoff C Sharp cutoff filters transmit almost all incident light until the cutoff wavelength is reached. At that point, they cease to transmit light. Because they give an "all or none effect," only stray light reaches the detector when the selected wavelength is beyond the cutoff.

The phrase "first-pass hepatic metabolism" means that: A. One hundred percent of a drug is excreted by the liver B. All drug is inactivated by hepatic enzymes after one pass through the liver C. Some drug is metabolized from the portal circulation, reducing bioavailability D. The drug must be metabolized in the liver to an active form

C. Some drug is metabolized from the portal circulation, reducing bioavailability C Drugs given orally enter the blood via the portal circulation and are transported directly to the liver. Some drugs are excreted by the liver, and a fraction will be lost by hepatic excretion before the drug reaches the general circulation. An example is propranolol, a β-blocker that reduces heart rate and hypertension. The bioavailable fraction is 0.2-0.4 when given orally because much of the drug is removed by first-pass hepatic metabolism.

A plasma sample is hemolyzed and turbid. What is required to perform a sample blank in order to correct the measurement for the intrinsic absorbance of the sample when performing a spectrophotometric assay? A. Substitute deionized water for the sample B. Dilute the sample 1:2 with a standard of known concentration C. Substitute saline for the reagent D. Use a larger volume of the sample

C. Substitute saline for the reagent C A sample blank is used to subtract the intrinsic absorbance of the sample usually caused by hemolysis, icterus, turbidity, or drug interference. On automated analyzers, this is accomplished by measuring the absorbance after the addition of sample and a first reagent, usually a diluent. For tests using a single reagent, sample blanking can be done prior to the incubation phase before any color develops. Substituting deionized water for sample is done to subtract the absorbance of the reagent (reagent blanking). Diluting the sample with a standard (standard addition) may be done when the absorbance is below the minimum detection limit for the assay. Using a larger volume of sample will make the interference worse.

Which of the following statements regarding amylase methods is true? A. Dilution of serum may result in lower than expected activity B. Methods generating NADH are preferred because they have higher sensitivity C. Synthetic substrates can be conjugated to p-nitrophenol (PNP) for a kinetic assay D. The reference range is consistent from method to method

C. Synthetic substrates can be conjugated to p-nitrophenol (PNP) for a kinetic assay C Many endogenous inhibitors of amylase, such as wheat germ, are found in serum. Diluted samples often show higher than expected activity caused by dilution of the inhibitor. Units of amylase activity vary widely depending upon the method of assay and calibration. Synthetic substrates such as maltotetrose or 4-nitrophenyl maltohepatoside can be used for kinetic assays. Maltotetrose is hydrolyzed to maltose by amylase, and the maltose hydrolyzed by α-glucosidase or maltose phosphorylase, forming glucose or glucose-1 phosphate, respectively. These can be measured by coupling to NADH-generating reactions. Antibodies to the salivary isoenzyme can be added to synthetic substrate assays to inhibit S-type amylase.

The study of pharmacogenomics involves which type of testing? A. Family studies to determine the inheritance of drug resistance B. Testing drugs with cell cultures to determine the minimum toxic dosage C. Testing for single nucleotide polymorphisms known to affect drug metabolism D. Comparison of dose-response curves between family members

C. Testing for single nucleotide polymorphisms known to affect drug metabolism C Pharmacogenomics refers to the study of genes that affect the performance of a drug in an individual. One method is to test for single nucleotide polymorphisms (SNPs) using DNA microarrays in genes such as those that code for the cytochrome P450 enzymes involved in the metabolism of many drugs. Genetic variations of one such enzyme may account for individual pharmacokinetic differences and can be used to predict the efficacy of the drug

Which statement best describes testing recommendations for vitamin D? A. Vitamin D testing should be reserved only for those persons who demonstrate hypercalcemia of an undetermined cause B. Vitamin D testing should be specific for the 1,25(OH)D3 form C. Testing should be for total vitamin D when screening for deficiency D. Vitamin D testing should not be performed if the patient is receiving a vitamin D supplement

C. Testing should be for total vitamin D when screening for deficiency C Vitamin D deficiency is far more common than vitamin D excess, and screening for vitamin D deficiency is advocated especially for dark-skinned persons and people who do not get adequate sunlight. Provitamin D is a steroid, and vitamin D is now considered a hormone rather than a vitamin. The hormone regulates transcription of over 200 genes and has pronounced effects on both dendritic cells and T lymphocytes. Deficiency is associated with many chronic diseases including autoimmune diseases, cancers, hypertension, and heart disease. There are two forms of the vitamin, ergocalciferol (D2) and cholecalciferol (D3). Active D2 and D3 are formed when two hydroxyl groups are added, the first being at the 25 position by the liver and the second at the α-1 position by the kidney. The majority of the circulating vitamin D is in the 25-hydroxylated form of D2 and D3, called 25(OH)D. The plasma 25(OH)D concentration is an expression of both dietary and endogenous vitamin D and is the most appropriate test for detecting nutritional vitamin D deficiency. Since the effect on calcium is derived from the active 1,25 form of the vitamin, plasma 1,25(OH)D concentration is a more specific test for hypervitaminosis D

Which of the following statements regarding iron metabolism is correct? A. Iron absorption is decreased by alcohol ingestion B. Normally, 40%-50% of ingested iron is absorbed C. The daily requirement is higher for pregnant and menstruating women D. Absorption increases with the amount of iron in the body stores

C. The daily requirement is higher for pregnant and menstruating women C For adult men and nonmenstruating women, approximately 1-2 mg/day of iron is needed to replace the small amount lost mainly by exfoliation of cells. Because 5%-10% of dietary iron is absorbed normally, the daily dietary requirement in this group is 10-20 mg/day. Menstruating women have an additional requirement of 1 mg/day and pregnant women 2 mg/day. Absorption efficiency will increase in iron deficiency and decrease in iron overload. Iron absorption is enhanced by low gastric pH and is increased by alcohol ingestion.

Which of the following statements regarding enzymatic reactions is true? A. The enzyme shifts the equilibrium of the reaction to the right B. The enzyme alters the equilibrium constant of the reaction C. The enzyme increases the rate of the reaction D. The enzyme alters the energy difference between reactants and products

C. The enzyme increases the rate of the reaction C An enzyme will accelerate the rate of a reaction, reducing the time required to reach equilibrium. The concentration of reactants and products at equilibrium will be the same with or without the enzyme.

Kjeldahl's procedure for total protein is based upon the premise that: A. Proteins are negatively charged B. The pKa of proteins is the same C. The nitrogen content of proteins is constant D. Proteins have similar tyrosine and tryptophan content

C. The nitrogen content of proteins is constant C Kjeldahl's method measures the nitrogen content of proteins as ammonium ion by back titration following oxidation of proteins by sulfuric acid and heat. It assumes that proteins average 16% nitrogen by weight. Protein in grams per deciliter is calculated by multiplying protein nitrogen by 6.25. The Kjeldahl method is a reference method for total protein that is used to assign a protein assay value to calibrators

SITUATION: An EDTA sample for TnI assay gives a result of 0.04 ng/mL (reference range 0-0.03 ng/mL). The test is repeated 3 hours later on a new specimen and the result is 0.06 ng/mL. A third sample collected 6 hours later gives a result of 0.07 ng/mL. The EKG showed no evidence of ST segment elevation (STEMI). What is the most likely explanation? A. A false-positive result occurred due to matrix interference B. Heparin should have been used instead of EDTA, which causes false positives C. The patient has suffered cardiac injury D. The patient has had an ischemic episode without cardiac injury

C. The patient has suffered cardiac injury C EDTA is the additive of choice for troponin assays because it avoids microclots that can lead to false positive results when serum or heparinized plasma is used. Spurious false positives caused by matrix effects usually revert to normal when the test is repeated on a new sample. An AMI will cause the TnI to increase in subsequent tests. Results between 0.04-0.10 ng/mL are the result of cardiac injury, and indicate either AMI or an increased short-term risk of AMI.

What is the minimum requirement for performing QC for a total protein assay? A. One level assayed every 8 hours B. Two levels assayed within 8 hours C. Two levels assayed within 24 hours D. Three levels assayed within 24 hours

C. Two levels assayed within 24 hours C The minimum requirement for frequency of quality control for a general chemistry analyte (based upon the Clinical Laboratory Improvement Act, 1988) is two levels of control assayed every 24 hours. Some laboratories prefer to assay two control levels every 8 hours to increase the opportunity for error detection. Two controls every 8 hours are required for blood gases, automated hematology, and point-of-care glucose testing to comply with College of American Pathology requirements. Analytes that display different CVs at the low, normal, and high ranges require 3 levels of control in 24 hours. These include blood gases, therapeutic drugs, and hormones

Thyroid hormones are derived from the amino acid: A. Phenylalanine B. Methionine C. Tyrosine D. Histidine

C. Tyrosine C Thyroid hormones are derived from the enzymatic modification of tyrosine residues on thyroglobulin. Tyrosine is halogenated enzymatically with iodine, forming monoiodotyrosine (MIT) and diiodotyrosine (DIT). Enzymatic coupling of these residues form T3 (3,5,3´-triiodothyronine) and T4 (3,5,3´,5´ tetraiodothyronine). These are hydrolyzed from thyroglobulin, forming active hormones

Na K Cl HCO3 BUN Glucose Creatinine Uric Acid 140 mmol/L 5.8 mmol/L 102 mmol/L 18 mmol/L 2.6 mg/dL 20 mg/dL DL DL 132 mmol/L 4.8 mmol/L 98 mmol/L 24 mmol/L DL DL DL DL Two consecutive serum samples give the results shown in the table above (at the top of this page) for a metabolic function profile. The instrument is a random access analyzer that uses two sample probes. The first probe aspirates a variable amount of serum for the spectrophotometric chemistry tests, and the second probe makes a 1:50 dilution of serum for electrolyte measurements. What is the most likely cause of these results? A. Both patients have renal failure B. There is an insufficient amount of sample in both serum tubes C. There is a fibrin strand in the probe used for the spectrophotometric chemistry tests D. The same patient's sample was accidentally run twice

C. There is a fibrin strand in the probe used for the spectrophotometric chemistry tests C Electrolyte results for both patients are within the physiological range but are distinctly different. The first results indicate a high potassium and increased anion gap, and one would expect the BUN, uric acid, and creatinine to be elevated. However, the results for BUN and glucose are unlikely for any patient, and the creatinine and uric acid signals are below the detection limit of the analyzer, indicating that little or no sample was added. This could be caused by a partially obstructed sample probe, or insufficient sample volume. The results for the second sample are below detection limits for all spectrophotometric tests, which may be the result of complete probe obstruction or the inability to generate a detectable signal with the trace quantity of serum that was added. Because all of the low or undetectable signals are for tests sampled by the first probe, the only explanation is that the probe is obstructed or malfunctioning

The reference method for serum lipase is based upon: A. Assay of triglycerides following incubation of serum with olive oil B. Rate turbidimetry C. Titration of fatty acids with dilute NaOH following controlled incubation of serum with olive oil D. Immunochemical assay

C. Titration of fatty acids with dilute NaOH following controlled incubation of serum with olive oil C The reference method of Cherry and Crandall is based upon the titration of fatty acids formed by the hydrolysis of an emulsion of olive oil after incubation for 24 hours at 37°C. Because most of the activity occurs within the first 3 hours, the incubation time may be shortened to as little as 1 hour without loss of clinical utility.

An EDTA blood sample is collected from a nonfasting person for a CBC. The physician collected the sample from the femoral vein because venipuncture from the arm was unsuccessful. He called the lab 15 minutes after the sample arrived and requested a lipid study including triglyceride, total cholesterol, HDL cholesterol, and LDL cholesterol. Which test results should be used to evaluate the patient's risk for coronary artery disease? A. Total cholesterol and LDL cholesterol B. LDL cholesterol and triglyceride C. Total cholesterol and HDL cholesterol D. Total cholesterol and triglyceride

C. Total cholesterol and HDL cholesterol C NCEP recommends a 12-hour fasting sample when screening persons for risk of coronary artery disease. However, if a fasting sample is unavailable, NCEP recommends performing the total cholesterol and HDL cholesterol because these tests are least affected by recent ingestion of food. If the total cholesterol is ≥ 200 mg/dL or the HDL cholesterol is < 40 mg/dL, then testing for LDL cholesterol and triglycerides should be performed when a fasting sample can be obtained. An EDTA plasma sample is acceptable for mostenzymatic cholesterol and triglyceride assays.

Which glucose method is subject to falsely low results caused by ascorbate? A. Hexokinase B. Glucose dehydrogenase C. Trinder glucose oxidase D. Polarography

C. Trinder glucose oxidase C Although glucose oxidase is specific for β-D-glucose, the coupling (indicator) reaction is prone to negative interference from ascorbate, uric acid, acetoacetic acid, and other reducing agents. These compete with the chromogen (e.g., o-dianisidine) for peroxide, resulting in less dye being oxidized to chromophore. The choice of chromogen determines the specificity and linearity. 4-aminophenazone and phenol is more resistant to interference from azo compounds and proteins than is o-dianisidine.

When measuring lead in whole blood using atomic absorption spectrophotometry, what reagent is required to obtain the needed sensitivity and precision? A. Lanthanum B. Lithium C. Triton X-100 D. Chloride

C. Triton X-100 C A graphite furnace is preferred over a flame for measuring lead because it is sufficiently sensitive to detect levels below 5 μg/dL, the cutoff needed for lead screening of children. The matrix modifier consists of Triton X -100, ammonium phosphate and nitric acid. This allows for release of Pb from the RBCs, and solubilization of cell stroma. The matrix modifier also prevents loss of Pb caused by formation of lead halides and promotes interaction between Pb and the tube wall, preventing its loss during the ashing cycle.

Which statement about sample collection for catecholamines and metabolites is true? A. Blood for catecholamines is collected in the usual manner following a 12-hour fast B. Twenty-four-hour urine for vanillylmandelic acid, catecholamines, or metanephrines is collected in 1 mL of boric acid C. Twenty-four-hour urine creatinine should be measured with vanillylmandelic acid, homovanillic acid, or metanephrines D. There is no need to discontinue medications if a 24-hour urine collection is used

C. Twenty-four-hour urine creatinine should be measured with vanillylmandelic acid, homovanillic acid, or metanephrines C Stress, exercise, and an upright position induce catecholamine elevation, and therefore, patients must be resting supine for at least 30 minutes prior to blood collection. The preferred method of collection is catheterization, so that the anxiety of venipuncture is not a factor. A 4-hour fast is also recommended. Many drugs contain epinephrine, which may falsely elevate catecholamine measurements. In addition, many drugs inhibit monoamine oxidase, which is needed to convert metanephrines to VMA. Therefore, medications should be removed prior to testing whenever possible. Twenty-four-hour urine samples for catecholamines are usually preserved with 10 mL of 6N HCl because some degradation occurs during storage when pH is greater than 3. Renal clearance affects excretion of catecholamine metabolites; it is preferable to report VMA, HVA, and metanephrines, in μg/mg creatinine. The urinary creatinine measurement should be at least 0.8 g/day, to validate the completeness of the 24-hour urine sample.

Which of the following statements about fluorometry is accurate? A. Fluorometry is less sensitive than spectrophotometry B. Fluorometry is less specific than spectrophotometry C. Unsaturated cyclic molecules are often fluorescent D. Fluorescence is directly proportional to temperature

C. Unsaturated cyclic molecules are often fluorescent C Increasing temperature results in more random collision between molecules by increasing their motion. This causes energy to be dissipated as heat instead of fluorescence. Temperature is inversely proportional to fluorescence. Fluorescence is more sensitive than spectrophotometry because the detector signal can be amplified when dilute solutions are measured. It is also more specific than spectrophotometry because both the excitation and emission wavelengths are characteristics of the compound being measured.

The ion-selective membrane used to measure potassium is made of: A. High-borosilicate glass membrane B. Polyvinyl chloride dioctylphenyl phosphonate ion exchanger C. Valinomycin gel D. Calomel

C. Valinomycin gel C Valinomycin is an antibiotic with a highly selective reversible-binding affinity for potassium ions. Sodium electrodes are usually composed of a glass membrane with a high content of aluminum silicate. Calcium and lithium ion-selective electrodes are made from organic liquid ion exchangers called neutral carrier ionophores Calomel is made of mercury covered with a paste of mercurous chloride (Hg°/Hg2Cl2) and is used as a reference electrode for pH.

A decreased PAO2-PaO2 difference is found in: A. A/V (arteriovenous) shunting B. V/Q (ventilation/perfusion) inequality C. Ventilation defects D. All of these options

C. Ventilation defects C Patients with A/V shunts, V/Q inequalities, and cardiac failure will have an increased PAO2-PaO2 difference. However, patients with ventilation problems have low alveolar PO2 owing to retention of CO2 in the airway. This reduces the PAO2-PaO2 difference.

What type of measuring circuit is used in a freezing point osmometer? A. Electrometer B. Potentiometer C. Wheatstone bridge D. Thermal conductivity bridge

C. Wheatstone bridge C The resistance of the thermistor is measured using a network of resistors called a Wheatstone bridge. When the sample is frozen, the bridge is balanced using a calibrated variable resistor, so that no current flows to the readout. The resistance required to balance the meter is equal to the resistance of the thermistor.

Which statement is true regarding particle-enhanced turbidimetric inhibition immunoassay methods for therapeutic drugs? A. Drug concentration is proportional to light scatter B. Magnetic separation is needed to remove unbound conjugate C. When particle-bound drug binds to antibody, light scattering is increased D. Two antibodies to the drug are needed

C. When particle-bound drug binds to antibody, light scattering is increased C Particle-enhanced turbidimetric inhibition immunoassays are homogenous immunoassays frequently used to measure proteins and therapeutic drugs in serum or plasma. Polystyrene-modified latex particles conjugated to the drug (particle-bound drug) compete with drug in the sample for a limited number of antibodies. If drug concentration is low, more of the antibody binds to the particle-bound drug, increasing the turbidity of the reaction. Therefore, light scattering is inversely proportional to the drug concentration.

When calibrating a semiautomatic pipet that has a fixed delivery of 10.0 µL using a gravimetric method, what should be the average weight of deionized water transferred? A. 10.0 µg B. 100.0 µg C. 1.0 mg D. 10.0 mg

D. 10.0 mg D Gravimetric and spectrophotometric analysis are the two methods used to verify pipet volume accuracy and precision. Since spectrophotometric analysis involves dilution, gravimetric analysis is associated with greater certainty. At 20°C, the density of pure water is 0.99821 g/mL. Therefore, each microliter weighs almost exactly 1.0 mg.

Two methods for total cholesterol are compared by running 40 paired patient samples in duplicate on each instrument. The following results are obtained: Instrument / Mean / SD Method x (reference method) /235 mg/dL /3.8 Method y (candidate method)/ 246 mg/dL /3.4 Assuming the samples are collected and stored in the same way and the analysis done by a technologist who is familiar with both methods, what is the bias of method y? A. 0.4 B. 7.2 C. 10.6 D. 11.0

D. 11.0 D The bias is defined as the difference between the means of the two methods and is calculated using the formula: bias = y - ×. The bias is an estimate of SE. The student's t test is used to determine if bias is statistically significant. The t statistic is the ratio of bias to the standard error of the mean difference. The greater the bias, the higher the t score.

. A procedure for aspartate aminotransferase (AST) is performed manually because of a repeating error code for nonlinearity obtained on the laboratory's automated chemistry analyzer; 0.05 mL of serum and 1.0 mL of substrate are used. Te reaction rate is measured at 30°C at 340 nm using a 1.0 cM light path, and the delta absorbance (-∆A) per minute is determined to be 0.382. Based upon a molar absorptivity coefficient for NADH at 340 nm of 6.22 X 103 M-1 cM-1 L-1, calculate the enzyme activity in international units (IUs) per liter. A. 26 IU/L B. 326 IU/L C. 1228 IU/L D. 1290 IU/L

D. 1290 IU/L D An IU is defined as 1 μmol of substrate consumed or product produced per minute. The micromoles of NADH consumed in this reaction are determined by dividing the change in absorbance per minute by the absorbance of 1 μmol of NADH. Because 1 mol/L/cm would have an absorbance of 6.22 X 103 absorbance units, then 1 μmol/mL/cm would produce an absorbance of 6.22. Therefore, dividing the δA per minute by 6.22 gives the micromoles of NADH consumed in the reaction. This is multiplied by the dilution of serum to determine the micromoles per milliliter, and multiplied by 1,000 to convert to micromoles per liter. IU/L = ∆A/min x TV(mL) × 1,000 mL/L 6.22(A/μmol/mL/cM) x 1 cm x SV(mL) = ∆A/min × 1.05 x 1,000 6.22 X 0.05 = ∆A/min × 1,050 0.311 = ∆A/min × 3,376 = 0.382 × 3376 = 1,290 IU/L

A new test for prostate cancer is found to have a sensitivity of 80.0% and a specificity of 84.0%. If the prevalence of prostate cancer is 4.0% in men over 42 years old, what is the predictive value of a positive test result (PV+) in this group? A. 96.0% B. 86.0% C. 32.4% D. 17.2%

D. 17.2% D The predictive value of a positive test (PV+) is defined as the percentage of persons with a positive test result who will have the disease or condition. It is dependent upon the sensitivity of the test and the prevalence of the disease in the population tested. PV+ is calculated by multiplying the true positives by 100, then dividing by the sum of true positives and false positives. % PV+ = TP × 100 / (TP + FP) where TP equals (sensitivity × prevalence) and FP equals (1 - specificity) × (1 - prevalence) = 0.80 × 0.04 × 100 / (0.80 × 0.04) + [(1 - 0.84) × (1 - 0.04)] = 0.032 × 100 / 0.032 + (0.96 × 0.16) = 17.2%

Two consecutive controls are both beyond -2s from the mean. How frequently would this occur on the basis of chance alone? A. 1:100 B. 5:100 C. 1:400 D. 1:1,600

D. 1:1,600 D QC results follow a Gaussian or normal distribution. Ninety-five percent of the results fall within ±2s of the mean; therefore, 2.5 out of 100 (1:40) are above +2s and 2.5 out of 100 are below -2s. The probability of two consecutive controls being beyond -2s is the product of their individual probabilities. 1/40 × 1/40 = 1/1,600 trials by chance.

The LD pleural fluid:serum ratio for a transudative fluid is usually: A. 3:1 or higher B. 2:1 C. 1:1 D. 1:2 or less

D. 1:2 or less D The lactate dehydrogenase activity of body fluids is normally less than serum, and a fluid to serum LD ratio greater than 1:2 is highly suggestive of an exudative process. Elevated lactate dehydrogenase in chest fluid is often caused by lung malignancy, metastatic carcinoma, Hodgkin's disease, and leukemia.

How many milliliters of glacial acetic acid are needed to prepare 2.0 L of 10.0% v/v acetic acid? A. 10.0 mL B. 20.0 mL C. 100.0 mL D. 200.0 mL

D. 200.0 mL D The expression percent v/v refers to the volume of one liquid in mL present in 100.0 mL of solution. To calculate, multiply the percentage (as mL) by the volume required (mL), then divide by 100 (mL). (10.0 mL × 2000.0 mL) ÷ 100.0 mL = 200.0 mL To prepare 2.0 L of a 10.0% v/v solution of acetic acid, add approximately 1.0 L of deionized H2O to a 2.0-L volumetric flask. Add 200.0 mL of glacial acetic acid and mix. Then, add sufficient deionized H2O to bring the meniscus to the 2.0-L line and mix again

What is the maximum recommended storage time and temperature for an arterial blood gas sample drawn in a plastic syringe? Storage Time / Temperature A. 10 min 2°C-8°C B. 20 min 2°C-8°C C. 30 min 2°C-8°C D. 30 min 22°C

D. 30 min 22°C D Arterial blood gas samples collected in plastic syringes should be stored at room temperature because cooling the sample allows oxygen to enter the syringe. Storage time should be no more than 30 minutes because longer storage results in a significant drop in pH and PO2 and increased PCO2.

Which monochromator specification is required in order to measure the true absorbance of a compound having a natural absorption bandwidth of 30 nm? A. 50-nm bandpass B. 25-nm bandpass C. 15-nm bandpass D. 5-nm bandpass

D. 5-nm bandpass D Bandpass refers to the range of wavelengths passing through the sample. The narrower the bandpass, the greater the photometric resolution. Bandpass can be made smaller by reducing the width of the exit slit. Accurate absorbance measurements require a bandpass less than one-fifth the natural bandpass of the chromophore

Which type of monochromator produces the purest monochromatic light in the UV range? A. A diffraction grating and a fixed exit slit B. A sharp cutoff filter and a variable exit slit C. Interference filters and a variable exit slit D. A prism and a variable exit slit

D. A prism and a variable exit slit D Diffraction gratings and prisms both produce a continuous range of wavelengths. A diffraction grating produces a uniform separation of wavelengths. A prism produces much better separation of high-frequency light because refraction is greater for higher-energy wavelengths. Instruments using a prism and a variable exit slit can produce UV light of a very narrow bandpass. The adjustable slit is required in order to allow sufficient light to reach the detector to set 100%T.

In absorption spectrophotometry: A. Absorbance is directly proportional to transmittance B. Percent transmittance is directly proportional to concentration C. Percent transmittance is directly proportional to the light path length D. Absorbance is directly proportional to concentration

D. Absorbance is directly proportional to concentration D Beer's law states that A = a × b × c, where a is the absorptivity coefficient (a constant), b is the path length, and c is concentration. Absorbance is directly proportional to both b and c. Doubling the path length results in incident light contacting twice the number of molecules in solution. This causes absorbance to double, the same effect as doubling the concentration of molecules.

Select the chemical that is used in most HPLC procedures to decrease solvent polarity. A. Hexane B. Nonane C. Chloroform D. Acetonitrile

D. Acetonitrile D All of the compounds mentioned have nonpolar properties. Because most HPLC is reverse phase (a polar solvent is used), hexane and nonane are too nonpolar. Acetonitrile is more polar and less toxic than chloroform and along with methanol is a common polarity modifier for HPLC.

In which case might a very low plasma TSH result not correlate with thyroid status? A. Euthyroid sick syndrome B. Congenital hypothyroidism C. When TBG is elevated D. After high-dose corticosteroid treatment

D. After high-dose corticosteroid treatment D In persons with severe chronic diseases or who have hCG-secreting tumors, TSH production may be suppressed. Some drugs, especially high doses of corticosteroids, will suppress TSH production. Low TSH levels not matching thyroid status can also be seen in patients who have recently been treated for hyperthyroidism because there is a delay in the pituitary response. High-sensitivity TSH assays that can measure as little as 0.01 mIU/L and free T4 and T3 can help differentiate these conditions from clinical hyperthyroidism. If the TSH is below .03 mIU/L and the free hormone levels are increased, this points to hyperthyroidism. Lab values in euthyroid sick syndrome may mimic mild hypothyroidism. In euthyroid sick syndrome, thyroid function will be normal, but TSH may be slightly increased owing to lower levels of free T3. In euthyroid sick syndrome, the rT3 will be increased.

Which of the following is more commonly associated with a nonmalignant form of monoclonal gammopathy (MGUS)? A. Bone marrow plasma cells comprise 20% of nucleated cells B. Monoclonal protein (M-protein) concentration is 3.5 g/dL C. M-protein is IgG D. Age greater than 60 at the time of monoclonal protein discovery

D. Age greater than 60 at the time of monoclonal protein discovery D MGUS is the most common cause of monoclonal gammopathy. About 3% of the U.S. population at age 50 and 5% at age 70 have MGUS. The absence of bone lesions and organ damage, plasma cells below 10% of nucleated bone marrow cells, and M-protein below 3.0 g/dL are characteristic of MGUS as opposed to myeloma or other malignant gammopathy. About 50% of persons with MGUS have IgH gene translocations or chromosome 13 deletion associated with multiple myeloma. The risk of transformation of MGUS to malignant disease is about 1% per year.

Which of the following enzymes is considered most tissue specific? A. Creatine kinase (CK) B. Amylase C. Alkaline phosphatase (ALP) D. Alcohol dehydrogenase (ADH)

D. Alcohol dehydrogenase (ADH) D No enzyme is truly tissue specific and diagnostic accuracy depends upon recognizing changes in plasma levels that characterize different diseases. This includes the mass or activity of enzyme released, its rise, peak, and return to normal, the isoenzyme(s) released, and the concomitant changes of other enzymes. Alanine aminotransferase and alcohol dehydrogenase are primarily increased in necrotic liver disease.

Which of the following disorders is associated with lactate acidosis? A. Diarrhea B. Renal tubular acidosis C. Hypoaldosteronism D. Alcoholism

D. Alcoholism D Lactate acidosis often results from hypoxia, which causes a deficit of nicotinamide adenine dinucleotide, the oxidized form (NAD+). This promotes the reduction of pyruvate to lactate, regenerating NAD+ needed for glycolysis. In alcoholic acidosis, oxidation of ethanol to acetaldehyde consumes the NAD+. In diabetes, lactate acidosis can result from depletion of Krebs cycle intermediates. Diarrhea and renal tubular acidosis result in metabolic acidosis via bicarbonate loss. Hypoaldosteronism causes metabolic acidosis via hydrogen and potassium ion retention.

Which of the following will shift the O2 dissociation curve to the left? A. Anemia B. Hyperthermia C. Hypercapnia D. Alkalosis

D. Alkalosis D A left shift in the oxyhemoglobin dissociation curve signifies an increase in the affinity of Hgb for O2. This occurs in alkalosis, hypothermia, and in those hemoglobinopathies such as Hgb Chesapeake that increase the binding of O2 to heme. A right shift in the oxyhemoglobin dissociation curve lowers the affinity of Hgb for O2. This occurs in anemia due to increased 2,3 diphosphoglycerate (2,3-DPG), with increased body temperature, increased hydrogen ion concentration, hypercapnia (increased PCO2), and in some hemoglobinopathies, such as Hgb Kansas

Which of the following conditions is associated with hypokalemia? A. Addison's disease B. Hemolytic anemia C. Digoxin intoxication D. Alkalosis

D. Alkalosis D Addison's disease (adrenocortical insufficiency) results in low levels of adrenal corticosteroid hormones, including aldosterone and cortisol. Because these hormones promote reabsorption of sodium and secretion of potassium by the collecting tubules, patients with Addison's disease display hyperkalemia and hyponatremia. Hemolytic anemia and digoxin intoxication cause release of intracellular potassium. Alkalosis causes potassium to move from the extracellular fluid into the cells as hydrogen ions move from the cells into the extracellular fluid to compensate for alkalosis.

Which formula correctly describes the relationship between absorbance and %T ? A. A = 2 - log %T B. A = log 1/T C. A = -log T D. All of these options

D. All of these options D Absorbance is proportional to the inverse log of transmittance. A = -log T = log 1/T Multiplying the numerator and denominator by 100 gives: A = log (100/100 X T) 100 X T = %T, substituting %T for 100 X T gives: A = log 100/%T A = log 100 - log %T A = 2.0 - log %T For example, if %T = 10.0, then: A = 2.0 - log 10.0 log 10.0 = 1.0 A = 2.0-1.0 = 1.0

Which could account for drug toxicity following a normally prescribed dose? A. Decreased renal clearance caused by kidney disease B. Discontinuance or administration of another drug C. Altered serum protein binding caused by disease D. All of these options

D. All of these options Decreased renal clearance caused by kidney disease, Discontinuance or administration of another drug, and Altered serum protein binding caused by disease D Therapeutic drug monitoring is necessary for drugs that have a narrow therapeutic index. Individual differences alter pharmacokinetics, causing lack of correlation between dose and drug blood level. These include age, diet, ingestion with or without food, genetic factors, exercise, smoking, pregnancy, metabolism of other drugs, protein binding, and disease states.

For which drug group are both peak and trough measurements usually required? A. Antiarrhythmics B. Analgesics C. Tricyclic antidepressants D. Aminoglycoside antibiotics

D. Aminoglycoside antibiotics D Aminoglycoside antibiotics cause damage to the eighth cranial nerve at toxic levels, resulting in hearing loss. When given at subtherapeutic doses, they fail to resolve infection. Most drugs falling in the other classes have a narrow peak-trough difference but are highly toxic when blood levels exceed the therapeutic range. Usually, these can be safely monitored by measuring trough levels.

In addition to polarography, what other electrochemical method can be used to measure glucose in plasma? A. Conductivity B. Potentiometry C. Anodic stripping voltammetry D. Amperometry

D. Amperometry D In some critical care analyzers, amperometric measurement of glucose is used. The glucose oxidaseis impregnated into the membrane covering the electrode. It reacts with glucose in the sample, forming H2O2. This diffuses across the membrane to the anode of the electrode, where it is oxidized to O2. The electrons produced are used to reduce oxygen at the cathode, completing the current path. At the anode (usually platinum), 2H2O2 → 4e- + 2O2 + 4H+. At the cathode (usually silver), O2 + 4H+ + 4e- → 2H2O. The net equation is 2H2O2 → O2 + 2H2O.

All of the following are requirements for a QC material except: A. Long-term stability B. The matrix is similar to the specimens being tested C. The concentration of analytes reflects the clinical range D. Analyte concentration must be independent of the method of assay

D. Analyte concentration must be independent of the method of assay D Quality control materials are stable, made of the same components as the specimen, cover the dynamic linear range of the assay, and can be used for multiple analytes. The target mean for QC samples is determined from replicate assays by the user's method, not the "true" concentration of the analyte. Out-of-control results are linked to analytic performance rather than to the inherent accuracy of the method.

Which of the following is associated with Tangier disease? A. Apoprotein C-II deficiency B. Homozygous apo-B100 deficiency C. Apoprotein C-II activated lipase D. Apoprotein A-I deficiency

D. Apoprotein A-I deficiency D Deficiency of apo A-I is seen in Tangier disease, a familial hypocholesterolemia. Heterozygotes have about half of the normal level of HDL (familial hypoalphalipoproteinemia) and homozygotes have almost no detectable HDL. Tangier disease is caused by a mutation of the ATP-binding cassette gene. The deficient gene prevents apo A-I from binding lipids, and it israpidly catabolized. Abetalipoproteinemia results from defective hepatic transport of apo-B100, and is also inherited as an autosomal recessive condition. LDL is absent, and the condition is associated with hemolytic anemia and central nervous system damage

Which of the statements below about serum urea is true? A. Levels are independent of diet B. Urea is not reabsorbed by the renal tubules C. High BUN levels can result from necrotic liver disease D. BUN is elevated in prerenal as well as renal failure

D. BUN is elevated in prerenal as well as renal failure D Urea is completely filtered by the glomeruli but reabsorbed by the renal tubules at a rate dependent upon filtrate flow and tubular status. Urea levels are a sensitive indicator of renal disease, becoming elevated by glomerular injury, tubular damage, or poor blood flow to the kidneys (prerenal failure). Serum urea (and BUN) levels are influenced by diet and are low in necrotic liver disease.

If the peak level is appropriate but the trough level too low at steady state, then the dose interval should: A. Be lengthened without changing the dose per day B. Be lengthened and dose rate decreased C. Not be changed, but dose per day increased D. Be shortened, but dose per day not changed

D. Be shortened, but dose per day not changed D Increasing the dose rate may result in peak drug levels in the toxic range. Decreasing the dosing interval will raise the trough level so that it is maintained in the therapeutic range. The trough level is affected by the drug clearance rate. If clearance increases, then trough level decreases

Which of the following amylase substrates is recommended by the IFCC? A. Starch B. Maltodextrose C. Maltotetrose D. Blocked maltohepatoside

D. Blocked maltohepatoside D Amylase is commonly measured using synthetic substrates. In the IFCC-recommended method, p-nitrophenyl maltohepatiside is used. One end of the polymer is covalently linked to p-nitrophenol and the other is linked to 4,6 ethylidine to prevent its hydrolysis by α-glucosidase. Amylase hydrolyzes the substrate from both ends producing fragments of 2, 3, and 4 glucose subunits. α-Glucosidase hydrolyzes the subunits containing p-nitrophenyl groups, forming glucose and p-nitrophenol. The increase absorbance at 405 nm is proportional to amylase activity.

SITUATION: A patient breathing room air has the following arterial blood gas and electrolyte results: pH = 7.54 PCO2 = 18.5 mm Hg PO2 = 145 mm Hg HCO3 = 18 mmol/L Na = 135 mmol/L K = 4.6 mmol/L Cl = 98 mmol/L TCO2 = 20 mmol/L The best explanation for these results is: A. Blood for electrolytes was drawn above an IV B. Serum sample was hemolyzed C. Venous blood was sampled for arterial blood gases D. Blood gas sample was exposed to air

D. Blood gas sample was exposed to air D A patient breathing room air cannot have an arterial PO2 greater than 105 mm Hg because alveolar PO2 is 110 mm Hg when breathing 20% O2. Exposure to air caused loss of CO2 gas and increased pH.

A serum ALP level greater than twice the elevation of GGT suggests: A. Misidentification of the specimen B. Focal intrahepatic obstruction C. Acute alcoholic hepatitis D. Bone disease or malignancy

D. Bone disease or malignancy D In obstructive jaundice, GGT is elevated more than ALP. A disproportionate increase in ALP points to a nonhepatic source of ALP, often bone disease. GGT is the most sensitive marker of acute alcoholic hepatitis, rising about fivefold higher than ALP or transaminases

Which of the following statements applies to both measurement of VMA and metanephrines in urine? A. Both can be oxidized to vanillin and measured at 360 nm without interference from dietary compounds B. Both can be measured immunochemically after hydrolysis and derivatization C. Both require acid hydrolysis prior to measurement D. Both can be measured by specific HPLC and MS assays

D. Both can be measured by specific HPLC and MS assays D VMA and metanephrines can both be measured as vanillin after oxidation with periodate. However, these methods are affected by dietary sources of vanillin; coffee, chocolate, bananas, and vanilla must be excluded from the diet. Metanephrines, VMA, and HVA are most often measured by HPLC-EDC.

Which serum protein should be measured in a patient suspected of having Wilson's disease? A. Hemopexin B. Alpha-1 antitrypsin C. Haptoglobin D. Ceruloplasmin

D. Ceruloplasmin D α-1 antitrypsin, haptoglobin, and ceruloplasmin are acute phase proteins and will be increased in inflammatory diseases. Ceruloplasmin is an α-2 globulin that binds the majority of the serum copper. Levels are low in almost all patients with Wilson's disease, an autosomal recessive disorder caused by accumulation of copper in liver, brain, kidney, and other tissues. Low ceruloplasmin may occur in patients with nephrosis, malnutrition, and hepatobiliary disease. Therefore, the diagnosis of Wilson's disease is made by demonstrating decreased plasma ceruloplasmin, increased urinary copper, and the presence of Kayser-Fleischer rings (brown deposits at the edge of the cornea).

SITUATION: A urine sample is received in the laboratory with the appropriate custody control form, and a request for drug of abuse screening. Which test result would be cause for rejecting the sample? A. Temperature after collection 95°F B. pH 5.0 C. Specific gravity 1.005 D. Creatinine 5 mg/dL

D. Creatinine 5 mg/dL D Approximately 5 per 1,000 urine samples received for DAU testing have been adulterated by either dilution, substitution, or addition of substances such as glutaraldehyde that interfere with testing. The majority of these situations can be detected by determining temperature (90°F-100°F) pH (4.5-8.0), specific gravity (1.003-1.019), and creatinine (≥20 mg/dL). All of the values listed are within the limits of an acceptable sample with the exception of creatinine. Dry reagent strips are available that test for pH, specific gravity, creatinine, nitrite, peroxide, pyridinium, and glutaraldehyde.

Given the following QC chart, identify the day in which a violation of the R4s QC rule occurs. A. Day 3 B. Day 8 C. Day 10 D. Day 15

D. Day 15 D An R4s error is defined as the algebraic difference between two controls within the same run. In this Levy-Jennings plot, on day 15, Level 1 is above the +2s limit (approximately +2.5s) and Level 2 is below the -2s limit (approximately -2.5s). These controls are approximately 5s apart (+2.5s minus -2.5s = +5s).

Which of the following conditions is most likely to produce an elevated plasma potassium? A. Hypoparathyroidism B. Cushing's syndrome C. Diarrhea D. Digitalis overdose

D. Digitalis overdose D Digitalis toxicity causes potassium to leave the cells and enter the extracellular fluid, resulting in hyperkalemia. Renal failure, hemolytic anemia and Addison's disease are other frequent causes of hyperkalemia. Hypoparathyroidism indirectly causes hypokalemia by inducing alkalosis via increased renal retention of phosphate and bicarbonate. Cushing's syndrome (adrenal cortical hyperfunction) results in low potassium and elevated sodium. Diarrhea causes loss of sodium and potassium.

SITUATION: Results of an iron profile are: Serum Fe = TIBC = ferritin = 40 µg/dL 400 µg/dL 50 µg/L All of the following tests are useful in establishing a diagnosis of Fe deficiency except: A. Protein electrophoresis B. Erythrocyte zinc protoporphyrin C. Serum transferrin D. Hgb electrophoresis

D. Hgb electrophoresis D Electrophoresis may show an elevated β-globulin (transferrin) characteristic of iron deficiency, or inflammation that would help explain a normal ferritin. Zinc protoporphyrin is elevated in iron deficiency and in lead poisoning. Hemoglobinopathies and thalassemias are not associated with iron deficiency.

Which of the following tests is consistently abnormal in osteoporosis? A. High urinary calcium B. High serum Pi C. Low serum calcium D. High urine or serum N-telopeptide of type 1 collagen

D. High urine or serum N-telopeptide of type 1 collagen D Commonly used markers for other bone diseases such as serum or urinary calcium, inorganic phosphorus, total alkaline phosphatase (ALP), and vitamin D are neither sensitive nor specific for osteoporosis. Calcium and phosphorus are usually within normal limits. Although estrogen deficiency reduces formation of 1,25 hydroxyvitamin D (1,25 hydroxycholecalciferol), promoting postmenopausal osteoporosis, the 1,25 hydroxyvitamin D is low in only 30%-35% of cases, and low levels may be caused by other bone disorders. Serum markers for osteoporosis include both N-telopeptide of type 1 collagen (NTx) and C-telopeptide of type 1 collagen (CTx). These can be used to follow treatment with resorption antagonists (bisphosphonates) because they decrease significantly when therapy is successful.

Na K Cl HCO3 BUN Glucose Creatinine Uric Acid 140 mmol/L 3.6 mmol/L 100 mmol/L 28 mmol/L 130 mg/dL 110 mg/dL 1.2 mg/dL 4.8 mg/dL 148 mmol/L 4.2 mmol/L 110 mmol/L 24 mmol/L 135 mg/dL 86 mg/dL 0.8 mg/dL 3.9 mg/dL 138 mmol/L 4.0 mmol/L 105 mmol/L 22 mmol/L 142 mg/dL 190 mg/dL 1.0 mg/dL 4.6 mg/dL The results shown in the table above are obtained from three consecutive serum samples using an automated random access analyzer that samples directly from a bar-coded tube. Calibration and QC performed at the start of the shift are within the acceptable range, and no error codes are reported by the analyzer for any tests on the three samples. Upon results verification, what is the most appropriate course of action? A. Report the results and proceed with other tests since no analytical problems are noted B. Repeat the controls before continuing with further testing, but report the results C. Check sample identification prior to reporting D. Do not report BUN results for these patients or continue BUN testing

D. Do not report BUN results for these patients or continue BUN testing D BUN is elevated 5- to 10-fold for three consecutive patients in the absence of any other laboratory evidence of renal disease. The glucose results show conclusively that the samples are not from the same patient. Therefore, the BUN results must be caused by a systematic error, and should not be reported. Further testing for BUN should cease until the analytical components of the BUN assay are completely evaluated and the cause of these results identified and corrected. This is demonstrated by successful recalibration and performance of controls within acceptable limits. Following this, the BUN assay should be repeated on the three samples along with all other specimens with a spurious BUN result that have occurred since the start of the shift.

In addition to measuring blood glucose, Hgb A1c, and microalbumin, which test should be done on diabetic persons once per year? A. Urine glucose B. Urine ketones C. Plasma fructosamines D. Estimated glomerular filtration rate

D. Estimated glomerular filtration rate D While urinary glucose can identify persons who may have diabetes, it is not sensitive enough to manage glucose control on a daily basis, and has been replaced by whole-blood glucose monitoring or continuous glucose monitoring. While the urinary ketone test is a useful screening test for diabetic and other forms of ketosis, the plasma β hydroxybutyrate test should be used to identify and monitor ketosis in diabetic persons. Fructosamine is a useful adjunct to Hgb A1c to identify poor control of blood glucose in the past 2-4 weeks, but has not been recommended for routine use in all diabetic patients.

SITUATION: Results of an iron profile are: serum Fe = 40 µg/dL TIBC = 400 µg/dL ferritin = 40µg/L transferrin = 300 mg/dL (reference range 15-200) These results indicate: A. Error in calculation of TIBC B. Serum iron falls before ferritin in iron deficiency C. A defect in iron transport and not Fe deficiency D. Excess release of ferritin caused by injury

D. Excess release of ferritin caused by injury D Serum ferritin levels fall before iron or TIBC in iron deficiency, and a low level of serum ferritin is diagnostic. However, low tissue levels of ferritin may be masked by increased release into the blood in liver disease, infection, and acute inflammation. Although this patient's serum ferritin is within reference limits, serum iron is low and percent saturation is only 10%. Note that the TIBC and transferrin results are both elevated and agree. TIBC can be estimated by multiplying the serum transferrin by 1.4. These results point to iron deficiency

Which condition can result in acidosis? A. Cystic fibrosis B. Vomiting C. Hyperaldosteronism D. Excessive O2 therapy

D. Excessive O2 therapy D When O2 saturation of venous blood is greatly elevated, Hgb cannot release O2. Oxyhemoglobin cannot bind CO2 or hydrogen ions and acidosis results. Pure O2 may cause neurological damage, leading to convulsion and blindness, especially in infants. It can induce respiratory failure by causing pulmonary hemorrhage, edema, and hyalinization. The other three conditions cause alkalosis. Vomiting and cystic fibrosis cause loss of chloride, resulting in hypovolemia and intestinal bicarbonate absorption. Hyperaldosteronism causes hypokalemia; this results in increased renal H+ excretion and a shift of H+ into cells in exchange for K+

In addition to the number of true negatives (TN), which of the following measurements is needed to calculate specificity? A. True positives B. Prevalence C. False negatives D. False positives

D. False positives D The clinical specificity of a laboratory test is defined as the true negatives divided by the sum of true negatives and false positives (FP). % Specificity = (TN × 100)/(TN + FP) Specificity is defined as the percentage of disease-free people who have a negative test result. The probability of false positives is calculated from the specificity as: 1-(% specificity/100)

Which statement regarding the measurement of bilirubin by the Jendrassik-Grof method is correct? A. The same diluent is used for both total and direct assays to minimize differences in reactivity B. Positive interference by Hgb is prevented by the addition of HCl after the diazo reaction C. The color of the azobilirubin product is intensified by the addition of ascorbic acid D. Fehling's reagent is added after the diazo reaction to reduce optical interference by hemoglobin

D. Fehling's reagent is added after the diazo reaction to reduce optical interference by hemoglobin D The Jendrassik-Grof method uses HCl as the diluent for the measurement of direct bilirubin because unconjugated bilirubin is poorly soluble at low pH. Total bilirubin is measured using an acetate buffer with caffeine added to increase the solubility of the unconjugated bilirubin. After addition of diazotized sulfanilic acid and incubatiion, the diazo group is reduced by ascorbic acid, and Fehling's reagent is added to alkalinize the diluent. At an alkaline pH the product changes from pink to blue, shifting the absorbance maximum to 600 nm where Hgb does not contribute significantly to absorbance.

Select the enzyme that is most specific for β-D-glucose. A. Hexokinase B. G-6-PD C. Phosphohexisomerase D. Glucose oxidase

D. Glucose oxidase D Glucose oxidase is the most specific enzyme reacting with only β-D-glucose. However, the peroxida coupling reaction used in the glucose oxidase method is subject to positive and negative interference. Therefore, hexokinase is used in the reference method.

In the liver, bilirubin is conjugated by addition of: A. Vinyl groups B. Methyl groups C. Hydroxyl groups D. Glucuronyl groups

D. Glucuronyl groups D The esterification of glucuronic acid to the propionyl side chains of the inner pyrrole rings (I and II) makes bilirubin water soluble. Conjugation is required before bilirubin can be excreted via the bile.

Which test becomes abnormal in the earliest stage of the acute coronary syndrome? A. Myosin light chain 1 B. CK-MB isoforms C. Myoglobin D. High-sensitivity C-reactive protein

D. High-sensitivity C-reactive protein D The acute coronary syndrome (ACS) refers to the evolution of coronary artery events that lead up to AMI. Coronary artery disease (CAD) begins with formation of a plaque comprised of lipid from dead endothelium that proliferates into the artery lumen. The plaque becomes disrupted and the vessel wall inflamed in the asymptomatic stage of CAD. If platelet activation occurs and results in thrombosis, blood flow becomes significantly reduced, resulting in angina. This signals the transition to more advanced disease in which ischemia to heart muscle occurs and eventually to AMI. Myoglobin and CK-MB isoforms are not increased until the end stage of ACS. High-sensitivity C-reactive protein (hs-CRP) is an ultrasensitive CRP assay that accurately measures CRP below 1 mg/L. CRP is an acute-phase protein increased in inflammation. Levels of CRP between 3.2-10 mg/L signal low-grade inflammation, which occurs in the asymptomatic phase of ACS. Such inflammation occurs when coronary artery plaques become disrupted, and therefore, persons with CAD who have a mildly increased CRP are at high risk of disease progression. Glycogen phosphorylase-BB and albumin cobalt binding are increased by ischemia. Troponins are increased when there is unstable angina and ischemic injury, and indicate an increased risk for AMI.

Which of the following materials is best suited for verifying the wavelength calibration of a spectrophotometer? A. Neutral density filters B. Potassium dichromate solutions traceable to the National Bureau of Standards reference C. Wratten filters D. Holmium oxide glass

D. Holmium oxide glass D Wavelength accuracy is verified by determining the wavelength reading that gives the highest absorbance (or transmittance) when a substance with a narrow natural bandpass (sharp absorbance or transmittance peak) is scanned. For example, didymium glass has a sharp absorbance peak at 585 nm. Therefore, an instrument should give its highest absorbance reading when the wavelength dial is set at 585 nm. Holmium oxide produces a very narrow absorbance peak at 361 nm; likewise, the hydrogen lamp of a UV spectrophotometer produces a 656-nm emission line that can be used to verify wavelength. Neutral density filters and dichromate solutions are used to verify absorbance accuracy or linearity. A Wratten filter is a widebandpass filter made by placing a thin layer of colored gelatin between two glass plates and is unsuitable for spectrophotometric calibration

Which of the following is classified as a mucopolysaccharide storage disease? A. Pompe's disease B. von Gierke disease C. Hers' disease D. Hurler's syndrome

D. Hurler's syndrome D Hurler's syndrome is an autosomal recessive disease resulting from a deficiency of iduronidase. Glycosaminoglycans (mucopolysaccharides) accumulate in the lysosomes. Multiple organ failure and mental retardation occur, resulting in early mortality. Excess dermatan and heparin sulfate are excreted in urine. Other mucopolysaccharidoses (MPS storage diseases) are Hunter's, Scheie's, Sanfilippo's, and Morquio's syndromes.

Which of the following abnormal laboratory results is found in von Gierke's disease? A. Hyperglycemia B. Increased glucose response to epinephrine administration C. Metabolic alkalosis D. Hyperlipidemia

D. Hyperlipidemia D Von Gierke's disease (type 1 glycogen storage disease) results from a deficiency of glucose-6-phosphatase. This blocks the hydrolysis of glucose-6-PO4 to glucose and Pi, preventing degradation of glycogen to glucose. The disease is associated with increased triglyceride levels because fats are mobilized for energy and lactate acidosis caused by increased glycolysis. A presumptive diagnosis is made when intravenous galactose administration fails to increase serum glucose, and can be confirmed by demonstrating glucose-6-phosphatase deficiency or decreased glucose production in response to epinephrine.

Which of the following conditions is associated with both metabolic and respiratory alkalosis? A. Hyperchloremia B. Hypernatremia C. Hyperphosphatemia D. Hypokalemia

D. Hypokalemia D Hypokalemia is both a cause and result of alkalosis. In alkalosis, hydrogen ions may move from the cells into the extracellular fluid and potassium into the cells. In hypokalemia caused by overproduction of aldosterone, hydrogen ions are secreted by the renal tubules. This increase in net acid excretion results in metabolic alkalosis.

Which of the following proteins has the highest pI? A. Albumin B. Transferrin C. Ceruloplasmin D. IgG

D. IgG D Albumin is the fastest migrating protein toward the anode at pH 8.6 followed by α1-, α2-, β-, and γ-globulins. Because albumin is fastest, it has the greatest net negative charge and lowest pI (about 4.6). γ-Globulins are predominantly immunoglobulins and have the highest pI (about 7.2).

When should blood samples for trough drug levels be collected? A. 30 minutes after peak levels B. 45 minutes before the next dose C. 1-2 hours after the last dose D. Immediately before the next dose is given

D. Immediately before the next dose is given D The trough concentration of a drug is the lowest concentration obtained in the dosing interval. This occurs immediately before the absorption of the next dose given. Trough levels are usually collected just before the next dose is given

The increase in the level of serum enzymes used to detect cholestatic liver disease is caused mainly by: A. Enzyme release from dead cells B. Leakage from cells with altered membrane permeability C. Decreased perfusion of the tissue D. Increased production and secretion by cells

D. Increased production and secretion by cells D The amount of enzyme in the serum can be increased by necrosis, altered permeability, secretion, or synthesis. It is also dependent upon tissue perfusion, enzyme half-life, molecular size, and location of the enzyme within the cell. Most enzymes are liberated by necrosis, but a few, such as ALP and γ glutamyltransferase, are produced and secreted at a greater rate in obstructive liver disease

Which route of administration is associated with 100% bioavailability? A. Sublingual B. Intramuscular C. Oral D. Intravenous

D. Intravenous D When a drug is administered intravenously, all the drug enters the bloodstream, and therefore, the bioavailable fraction is 1.0. All other routes of administration require absorption through cells, and this process reduces the bioavailable fraction. The bioavailable fraction for a drug given orally can be calculated by dividing the peak blood concentration after oral administration by the peak drug concentration after IV administration. A value of 0.7 or higher is desired for drugs given orally

Which statement regarding gestational diabetes mellitus (GDM) is correct? A. Is diagnosed using the same oral glucose tolerance criteria as in nonpregnancy B. Converts to diabetes mellitus after pregnancy in 60%-75% of cases C. Presents no increased health risk to the fetus D. Is defined as glucose intolerance originating during pregnancy

D. Is defined as glucose intolerance originating during pregnancy D Control of GDM reduces perinatal complications such as respiratory distress syndrome, high birth weight, and neonatal jaundice. Women at risk are usually screened between 24 and 28 weeks' gestation. The screening test can be performed nonfasting and consists of an oral 50-g glucose challenge followed by serum or plasma glucose measurement at 1 hour. A result ≥ 140 mg/dL is followed by a 2-hour or 3-hour oral glucose tolerance test to confirm gestational diabetes. For the 3-hour test, a 100-g dose of glucose is used and at least two of the following cutoffs must be exceeded: fasting, ≥ 95 mg/dL or higher; 1 hour, ≥ 180 mg/dL or higher; 2 hour ≥ 155 mg/dL or higher; 3 hour, ≥ 140 mg/dL or higher. The same cutpoints are used for the 2-hour test except that a 75-g dose is used. GDM converts to diabetes mellitus within 10 years in 30%-40% of cases. ADA recommends testing persons with GDM for diabetes 6-12 weeks after delivery

Which of the following statements regarding the metabolism of bilirubin is true? A. It is formed by hydrolysis of the α methene bridge of urobilinogen B. It is reduced to biliverdin prior to excretion C. It is a by-product of porphyrin production D. It is produced from the destruction of RBCs

D. It is produced from the destruction of RBCs D Synthesis of porphyrins results in production of heme and metabolism of porphyrins other than protoporphyrin IX yields uroporphyrins and coproporphyrins, not bilirubin. Reticuloendothelial cells in the spleen digest Hgb and release the iron from heme. The tetrapyrrole ring is opened at the α methene bridge by heme oxygenase, forming biliverdin. Bilirubin is formed by reduction of biliverdin at the γ methene bridge. It is complexed to albumin and transported to the liver.

Which enzyme is least useful in differentiating necrotic from obstructive jaundice? A. GGT B. ALT C. 5' Nucleotidase D. LD

D. LD D GGT and 5´ nucleotidase are markedly elevated in both intra- and posthepatic obstruction. ALT is slightly elevated in obstructive jaundice but is markedly elevated in necrotic jaundice. Although LD is usually greater in necrotic jaundice than in obstructive jaundice, elevations in these conditions overlap frequently and result from many other causes.

When measuring calcium by atomic absorption spectrophotometry, which is required? A. An organic extraction reagent to deconjugate calcium from protein B. An internal standard C. A magnesium chelator D. Lanthanum oxide to chelate phosphates

D. Lanthanum oxide to chelate phosphates D An acidic diluent such as hydrochloric acid (HCl) will displace calcium bound to albumin. However, calcium forms a thermostable bond with phosphate that causes chemical interference in atomic absorption. Lanthanum displaces calcium, forming lanthanum phosphate, and eliminates interference from phosphates. Unlike in some colorimetric methods for calcium (e.g., o-cresolphthalein complexone), magnesium does not interfere because it does not absorb the 422.7 nm emission line from the calcium-hollow cathode lamp.

Which of the following tests is least essential to the operation of an emergency department at a general hospital? A. Carboxyhemoglobin B. Osmolality C. Salicylate D. Lead

D. Lead D The vast majority of acute toxicology situations seen in the emergency department (ED) involve poisoning with alcohol, acetaminophen, salicylate, abuse substances, or carbon monoxide. Emergency departments should offer a minimum of these tests. In the absence of specific tests for abuse substances or a comprehensive drug screen, the serum osmolality measured by freezing point depression is a sensitive surrogate test for drug and alcohol overdose. In the ED environment, a difference between measured and calculated osmolality greater than 10 mOsm/Kg almost always indicates drug or alcohol poisoning. Toxicity from lead poisoning and most other trace metals is usually a chronic condition that does not often require immediate access to laboratory testing.

What effect does hematocrit have on POCT tests for whole-blood glucose monitoring? A. Low hematocrit decreases glucose readings on all devices B. High hematocrit raises glucose readings on all devices C. The effect is variable and dependent on the enzyme/coenzyme system D. Low hematocrit raises readings and high hematocrit lowers readings unless corrected

D. Low hematocrit raises readings and high hematocrit lowers readings unless corrected D Hematocrit affects POCT glucose measurements. High hematocrit lowers the glucose because RBC glucose concentration is lower than plasma concentration. Other factors include binding of oxygen to hemoglobin and the slower diffusion of glucose onto the solid phase—both of which occur when the hematocrit is high. Bias due to an abnormal hematocrit can be avoided by simultaneously measuring the conductivity of the sample. The hematocrit is calculated and used to mathematically correct the glucose measurement.

The syndrome of inappropriate antidiuretic hormone secretion (SIADH) causes: A. Low serum vasopressin B. Hypernatremia C. Urine osmolality to be lower than plasma D. Low serum electrolytes

D. Low serum electrolytes D SIADH results in excessive secretion of vasopressin (ADH) from the posterior pituitary, causing fluid retention and low plasma osmolality, sodium, potassium, and other electrolytes by hemodilution. It is suspected when urine osmolality is higher than plasma, but urine sodium concentration is normal or increased. Patients with sodium depletion have a urine osmolality higher than plasma, but low urine sodium.

Which of the following conditions is associated with an increase in ionized calcium (Cai) in the blood? A. Alkalosis B. Hypoparathyroidism C. Hyperalbuminemia D. Malignancy

D. Malignancy D Increased Cai occurs in hyperparathyroidism, malignancy, and acidosis. Cai is elevated in primary hyperparathyroidism due to resorption of calcium from bone. Many nonparathyroid malignancies create products called parathyroid hormone-related proteins that stimulate the parathyroid receptors of cells. Acidosis alters the equilibrium between bound and free calcium, favoring ionization. Hyperalbuminemia increases the total calcium by increasing the proteinbound fraction, but does not affect the Ca

Which of the following conditions is most commonly associated with an elevated level of total protein? A. Glomerular disease B. Starvation C. Liver failure D. Malignancy

D. Malignancy D Malignant disease is usually associated with increased immunoglobulin and acute-phase protein production. However, nutrients required for protein synthesis are consumed, causing reduced hepatic albumin production. Glomerular damage causes albumin and other low molecular weight proteins to be lost through the kidneys. Liver failure and starvation result in decreased protein synthesis.

Which aminoaciduria results in the overflow of branched chain amino acids? A. Hartnup's disease B. Alkaptonuria C. Homocystinuria D. Maple syrup urine disease

D. Maple syrup urine disease D Valine, leucine, and isoleucine accumulate due to branched-chain decarboxylase deficiency in maple syrup urine disease. These are transaminated to ketoacids that are excreted, giving urine a maple sugar odor. Alkaptonuria is caused by homogentisic acid oxidase deficiency, causing homogentisic aciduria. Homocystinuria is a no-threshold-type aminoaciduria that usually results from cystathionine synthase deficiency

Which substance has the longest detection time? A. Amphetamines B. Cocaine C. Benzodiazepines D. Marijuana

D. Marijuana D Some drugs have a long half-life, and can be detected for longer periods after use, but the detection window also depends on other variables such as dosage, frequency of use, and method sensitivity. Marijuana is stored in fatty tissue and is metabolized slowly. In persons who use marijuana several times per week, cannabinoids can be detected several weeks after last use. For chronic daily users, this extends to months after discontinuation. Other drugs with detection windows of a week or more include long-acting barbiturates, LSD, anabolic steroids, and phencyclidine (PCP)

Which test is the most useful way to evaluate the response to treatment for multiple myeloma? A. Measure of total immunoglobulin B. Measurement of 24-hour urinary light chain concentration (Bence-Jones protein) C. Capillary electrophoresis of M-protein recurrence D. Measurement of serum-free light chains

D. Measurement of serum-free light chains D Unlike electrophoresis methods, serum free light chain assays are quantitative and an increase in free light chain production with an abnormal kappa:lambda ratio occurs earliest in recurrence of myeloma. Light chains have a shorter plasma half life than intact Ig and therefore, the reduction in free light chain concentration is an earlier indicator of treatment effect than measurement of intact Ig. It is not subject to the variation in 24-hour urinary light chain measurement caused by sample collection error and abnormal renal function. A 50% or more reduction in serum free light chain concentration is considered a partial response to treatment. A full response is indicated by reduction to within normal limits and a return of the kappa:lambda ratio to normal. An abnormal free light chain ratio has a 3.5-fold higher risk of progression to myeloma in persons with monoclonal gammopathy of undetermined significance (MGUS).

Which assay using 24-hour urine is considered the best single screening test for pheochromocytoma? A. Total urinary catecholamines B. VMA C. Homovanillic acid (HVA) D. Metanephrines

D. Metanephrines D Catecholamines are metabolized to metanephrines and VMA. Urinary catecholamines are increased by exercise and dietary ingestion. Measurement of 24-hour urinary metanephrine is about 95% sensitive for pheochromocytoma, and is the best single test. Specificity and sensitivity for detecting pheochromocytoma approach 100% when both VMA and metanephrines are measured.

What is the primary clinical utility of measuring CEA? A. Diagnosis of liver cancer B. Diagnosis of colorectal cancer C. Screening for cancers of endodermal origin D. Monitoring for recurrence of cancer

D. Monitoring for recurrence of cancer D CEA is a glycoprotein that is secreted into plasma by various cancers of endodermal origin, including breast, lung, colorectal, and stomach cancer. However, it is present in only 40%-60% of such cancers, is present at low levels (<3.0 ng/mL) in normal adults, and is increased by causes other than cancer (e.g., smoking). Its clinical use is to detect recurrence and the need for second-look surgery in persons who have been treated and to evaluate the response to treatment.

Which of the following cardiac markers derived from neutrophils predicts an increased risk for myocardial infarction? A. Phospholipase A2 (PLA2) B. Glycogen phosphorylase BB (GPBB) C. Soluble CD40 ligand (sCD40l) D. Myeloperoxidase (MPO)

D. Myeloperoxidase (MPO) D All of the answer choices are markers for acute coronary syndrome and increased risk of AMI. MPO is released from neutrophils and is thought to destabilize the arterial plaque by oxidizing both LDL and HDL and reducing nitric oxide levels in the coronary arteries. Levels in the upper third quartile predict an increased risk of a coronary event even when troponin is normal. GPBB is released from myocytes early in an ischemic episode and becomes abnormal about 2 hours after an AMI. sCD40l is released from activated platelets and indicates an unstable plaque. PLA2 is produced by the arterial wall. It removes a fatty acid from phospholipids and increases the amount of oxidized LDL, leading to foam cell formation. Like hs-CRP, it is a marker for an inflamed plaque.

Which product is measured in the coupling step of the urease-UV method for BUN? A. CO2 B. Dinitrophenylhydrazine C. Diphenylcarbazone D. NAD+

D. NAD+ D In the urease-UV method, urease is used to hydrolyze urea, forming CO2 and ammonia. Glutamate dehydrogenase catalyzes the oxidation of NADH, forming glutamate from 2-oxoglutarate and ammonia. The glutamate dehydrogenase reaction is used for measuring both BUN and ammonia.

A patient has a plasma myoglobin of 10 µg/L at admission. Three hours later, the myoglobin is 14 µg/L and the troponin I is 0.02 µg/L (reference range 0-0.03 µg/L). These results are consistent with which condition? A. Skeletal muscle injury B. Acute myocardial infarction C. Unstable angina D. No evidence of myocardial or skeletal muscle injury

D. No evidence of myocardial or skeletal muscle injury D This person displays very low plasma myoglobin (reference range for females is approximately 17-75 μg/L). The TnI result is also within normal limits. These results are consistent with baseline levels and no evidence of cardiac or skeletal muscle injury. TnI cutpoints for diagnosis of AMI are dependent upon the method, and may be higher than the upper limit of normal. Troponin results above the upper reference limit but below the cutpoint for AMI indicate myocardial injury and increased risk for AMI.

Which condition is most often associated with a high serum iron level? A. Nephrosis B. Chronic infection or inflammation C. Polycythemia vera D. Noniron deficiency anemias

D. Noniron deficiency anemias D Anemia associated with chronic infection causes a low serum iron, but unlike iron deficiency, causes a low (or normal) TIBC and does not cause low ferritin. Noniron deficiency anemias such as pernicious anemia and sideroblastic anemia produce high serum iron and low TIBC. Nephrosis causes iron loss by the kidneys. Polycythemia is associated with increased iron within the RBCs and depletion of iron stores.

Which statement about steady-state drug levels is true? A. The absorbed drug must be greater than the amount excreted B. Steady state can be measured after two elimination half-lives C. Constant intravenous infusion will give the same minima and maxima as an oral dose D. Oral dosing intervals give peaks and troughs in the dose-response curve

D. Oral dosing intervals give peaks and troughs in the dose-response curve D When drugs are infused intravenously, both the distribution and elimination rates are constant. This eliminates the peaks and troughs seen in the dose-response curve. Peak and trough levels are characteristics of intermittent dosing regimens. The steady state is reached when drug in the next dose is sufficient only to replace the drug eliminated since the last dose. Steady state can be measured after five drug half-lives because blood levels will have reached 97% of steady state

Which condition is least likely to be associated with increased serum ALP? A. Osteomalacia B. Biliary obstruction C. Hyperparathyroidism and hyperthyroidism D. Osteoporosis

D. Osteoporosis D ALP is elevated in osteomalacia (rickets), bone cancer, and bone disease secondary to hyperthyroidism and hyperparathyroidism, but total ALP it is high in less than 30% of osteoporosis patients. Pancreatic disease associated with biliary obstruction, such as cancer at the head of the pancreas, is associated with elevated ALP.

Which of the following conditions is associated with a low serum magnesium? A. Addison's disease B. Hemolytic anemia C. Hyperparathyroidism D. Pancreatitis

D. Pancreatitis D Low magnesium can be caused by gastrointestinal loss, as occurs in diarrhea and pancreatitis (loss of Mg and Ca as soaps). Hyperparathyroidism causes increased release of both calcium and magnesium from bone. Addison's disease (adrenocorticosteroid deficiency) may be associated with increased magnesium accompanying hyperkalemia. Hemolytic anemia causes increased release of magnesium as well as potassium from damaged red blood cells (RBCs).

A quantitative sandwich enzyme immunoassay for intact serum hCG was performed on week 4 and the result was 40,000 mIU/mL (reference range 10,000-80,000 mIU/mL). The physician suspected a molar pregnancy and requested that the laboratory repeat the test checking for the hook effect. Which process would identify this problem? A. Obtain a new plasma specimen and heat inactivate before testing B. Obtain a urine specimen and perform the assay C. Perform a qualitative pregnancy test D. Perform a serial dilution of the sample and repeat the test

D. Perform a serial dilution of the sample and repeat the test D The hook effect is the result of excessive antigen concentration and results in a dose response (calibration) curve that reverses direction at very high antigen concentrations. It occurs in two-site double antibody sandwich assays when both the capture antibody and the enzyme-conjugated antibody are incubated with the antigen at the same time. The excess antigen saturates both antibodies preventing formation of a double antibody sandwich. The hook effect can cause results to be sufficiently low to cause misdiagnosis. It can be detected by diluting the sample (antigen) in which case the assay result will be greater than in the undiluted sample. An alternative solution is to perform the test using a competitive binding assay or a sandwich assay in which the enzyme-labeled antibody is not added until after separation of free and bo und antigen.

The most commonly employed method of assay for plasma or serum lipase is based on: A. Hydrolysis of olive oil B. Rate turbidimetry C. Immunoassay D. Peroxidase coupling

D. Peroxidase coupling D Although all of the methods cited are available, the most commonly used method for lipase assay is based upon the hydrolysis of a synthetic diglyceride substrate yielding 2-monoglyceride. This is hydrolyzed and forms glycerol, which is phosphorylated and forms glycerol-3-phosphate. This is oxidized by glycerophosphate oxidase, yielding hydrogen peroxide. 1,2 diglyceride + H2O Lipase 2-monoglyceride + fatty acid 2- monoglyceride + H2O Monoglyceride esterase glycerol + fatty acid glycerol + ATP Glycerol kinase glycerol-3-phosphate + ADP glycerol-3-PO4 + O2 Glycerophosphate oxidase dihydroxyacetone phosphate + H2O2 H2O2 + 4-aminoantipyrene + TOOS Peroxidase quinoneimine dye + H2O

Which test is the most specific for establishing a diagnosis of Cushing's disease (pituitary Cushing's)? A. Low-dose dexamethasone suppression B. High-dose dexamethasone suppression C. Twenty-four-hour urinary free cortisol D. Petrosal sinus sampling following corticotropin-releasing hormone stimulation

D. Petrosal sinus sampling following corticotropin-releasing hormone stimulation D Although dexamethasone suppression tests have a high sensitivity, some patients without Cushing's syndrome have indeterminate results (e.g., values between 5 and 10 μg/dL) or abnormal results owing to medications or other conditions. When corticotropin-releasing hormone is given intravenously, patients with Cushing's disease have an exaggerated ACTH response. Samples are drawn from the sinuses draining the pituitary gland and from the peripheral blood. In patients with pituitary tumors, the ACTH will be several times higher in the sinus samples than in the peripheral blood samples

Which photodetector is most sensitive to low levels of light? A. Barrier layer cell B. Photodiode C. Diode array D. Photomultiplier tube

D. Photomultiplier tube D The photomultiplier tube uses dynodes of increasing voltage to amplify the current produced by the photosensitive cathode. It is 10,000 times as sensitive as a barrier layer cell, which has no amplification. A photomultiplier tube requires a DC-regulated lamp because it responds to light fluctuations caused by the AC cycle.

Which isoenzyme of ALP is most heat stable? A. Bone B. Liver C. Intestinal D. Placenta

D. Placenta D Placental ALP and tumor-associated isoenzymes such as the Regan isoenzyme associated with lung cancer are the only isoenzymes that retain activity when serum is heated to 65° C for 10 minutes. Heat inactivation is used primarily to distinguish liver ALP from bone ALP. If less than 20% activity remains after heating serum to 56°C for 10 minutes, then bone ALP is most likely present.

In which of the following conditions is PSA least likely to be increased? A. Precancerous lesions of the prostate B. Postprostate biopsy C. Benign prostatic hypertrophy D. Post-digital rectal examination

D. Post-digital rectal examination D PSA is a serine protease responsible for liquefaction of the seminal fluid. PSA has been used successfully to monitor for recurrence and follow the response of patients to androgen-suppression therapy. Currently, it is one of the few FDA-approved tumor markers for cancer screening. Although digital rectal examination raises the prostatic acid phosphatase level, it does not increase the concentration of PSA in the plasma. In addition to prostate cancer, PSA may be increased in acute or chronic prostate inflammation, benign prostate hypertrophy, and after transurethral prostate resection or prostate biopsy. As a result, the specificity of PSA is approximately 60% and the predictive value of a positive result approximately 30%

Which of the following conditions will increase total T4 by increasing TBG? A. Acute illness B. Anabolic steroid use C. Nephrotic syndrome D. Pregnancy or estrogens

D. Pregnancy or estrogens D Pregnancy and estrogens are the most common cause of increased TBG. Other causes include hepatitis, morphine, and clofibrate therapy. Acute illness, anabolic steroids, and nephrotic syndrome decrease the level of TBG. Normal pregnancy causes an elevated serum total T4. Suitable assays are available that estimate free T4 and T3 and these should be used instead of total hormone assays

Which of the following statements regarding ALP is true? A. In normal adults, the primary tissue source is fast-twitch skeletal muscle B. Geriatric patients have a lower serum ALP than other adults C. Serum ALP levels are lower in children than in adults D. Pregnant women have a higher level of serum ALP than other adults

D. Pregnant women have a higher level of serum ALP than other adults D ALP is higher in children than in adults due to bone growth. Children and geriatric patients have higher serum ALP due to increased bone isoenzyme. Serum ALP levels are often two- or threefold higher than the URL in the third term of pregnancy. In nonpregnant normal adults, serum ALP is derived from liver and bone. Liver, bone, placental, renal, and intestinal isoenzymes of ALP can be separated by electrophoresis, and many other ALP isoenzymes have been identified by isofocusing.

Select the hormone which when elevated is associated with galactorrhea, pituitary adenoma, and amenorrhea. A. E2 B. Progesterone C. Follicle-stimulating hormone (FSH) D. Prolactin

D. Prolactin D Serum prolactin may be increased from hypothalamic dysfunction or pituitary adenoma. When levels are greater than five times the URL, a pituitary tumor is suspected. Prolactin is measured by enzyme immunoassay (EIA).

Serial TnI assays are ordered on a patient at admission, 3 hours, and 6 hours afterwards. The samples were collected in heparinized plasma separator tubes. Following are the results (reference range 0 0.03 µg/L Admission = 3 hours = 6 hours = 0.03 µg/L 0.07 µg/L 0.02 µg/ These results indicate: A. A positive test for acute myocardial infarction B. Unstable angina C. Cardiac injury of severity less than myocardial infarction D. Random error with the 3-hour sample

D. Random error with the 3-hour sample D Troponin assays produce very little fluorescence or chemiluminescence when plasma levels are within the reference range and near the minimum detection limit of the assay. Fibrin, tube additives, and heterophile antibodies have been known to cause spurious elevations, and this result should be treated as a random error because the result before and after are both normal.

Which of the following results falls within the diagnostic criteria for diabetes mellitus? A. Fasting plasma glucose of 120 mg/dL B. Two-hour postprandial plasma glucose of 160 mg/dL C. Two-hour plasma glucose of 180 mg/dL following a 75 g oral glucose challenge D. Random plasma glucose of 250 mg/dL and presence of symptoms

D. Random plasma glucose of 250 mg/dL and presence of symptoms D The American Diabetes Association recommends the following criteria for diagnosing diabetes mellitus: fasting glucose ≥ 126 mg/dL, casual (random) glucose ≥ 200 mg/dL in the presence of symptoms (polyuria, increased thirst, weight loss), glucose ≥ 200 mg/dL at 2 hours after an oral dose of 75 g of glucose, and hemoglobin A1c ≥ 6.5%. A diagnosis of diabetes mellitus is indicated if any one or combination of these four criteria is met on more than a single testing event. The fasting plasma glucose test requires at least 8 hours with no food or drink except water. The 2-hour postloading test should be conducted according to the oral glucose tolerance guidelines currently recommended by the World Health Organization

After installing a new analyzer and reviewing the results of patients for 1 month, the lead technologist notices a greater frequency of patients with abnormally high triglyceride results. Analysis of all chemistry profiles run the next day indicated that triglyceride results are abnormal whenever the test is run immediately after any sample that is measured for lipase. These observations point to which type of error? A. Specificity of the triglyceride reagents B. Precision in pipetting of lipemic samples C. Bias caused by sequence of analysis D. Reagent carryover

D. Reagent carryover D Carryover errors are usually attributed to interference caused by a sample with a very high concentration of analyte preceding a normal sample. However, reagent carryover may also occur on automated systems that use common reagent delivery lines or reusable cuvettes. In the case of lipase methods, triglycerides used in the reagent may coat the reagent lines or cuvettes interfering with the triglyceride measurements that directly follow.

SITUATION: A physician calls to request a CK on a sample already sent to the laboratory for coagulation studies. The sample is 2-hour-old citrated blood and has been stored at 4°C. The plasma shows very slight hemolysis. What is the best course of action and the reason for it? A. Perform the CK assay on the sample because no interferent is present B. Reject the sample because it is slightly hemolyzed C. Reject the sample because it has been stored too long D. Reject the sample because the citrate will interfere

D. Reject the sample because the citrate will interfere D CK activity is lost with excessive storage, the most labile isoenzyme being CK-1. However, CK in serum is stable at room temperature for about 4 hours and up to 1 week at 4°C provided that an optimized method is used. Slight hemolysis does not interfere because CK is absent from RBCs. More significant hemolysis may cause positive interference by contributing ATP, glucose-6-PO4, and adenylate kinase to the serum. Calcium chelators remove magnesium as well as calcium and should not be used.

Which of the following conditions will cause an increased anion gap? A. Diarrhea B. Hypoaldosteronism C. Hyperkalemia D. Renal failure

D. Renal failure D An increased anion gap occurs when there is production or retention of anions other than bicarbonate or chloride (measured anions). For example, in renal failure, retention of phosphates and sulfates (as sodium salts) increases the anion gap. Other common causes of metabolic acidosis with an increased anion gap are diabetic ketoacidosis and lactate acidosis. The anion gap may also be increased in the absence of an acid-base disorder. Common causes include hypocalcemia, drug overdose, and laboratory error when measuring electrolytes.

A lipemic sample gives a sodium of 130 mmol/L on an analyzer that uses a 1:50 dilution of serum or plasma before introducing it to the ion selective electrodes. The same sample gives a sodium of 142 mmol/L using a direct (undiluted) ion selective electrode. Assuming acceptable quality control, which of the following is the most appropriate course of action? A. Report a sodium result of 136 mmol/L B. Ultracentrifuge the sample and repeat by ISE C. Dilute the sample 1:4 and repeat by ISE D. Report the undiluted ion selective electrode result

D. Report the undiluted ion selective electrode result D Lipemic samples give lower results for sodium (pseudohyponatremia) when diluted prior to measurement because the H2O phase is mostly diluent and a significant component of the sample volume is displaced by lipid. Direct ISEs measure sodium in the plasma water, more accurately reflecting patient status.

A technologist is asked to use the serum from a clot tube left over from a chemistry profile run at 8 a.m. for a stat ionized calcium (Cai) at 11 a.m. The technologist should: A. Perform the assay on the 8 a.m. sample B. Perform the test only if the serum container was tightly capped C. Perform the assay on the 8 a.m. sample only if it was refrigerated D. Request a new sample

D. Request a new sample D Cai is pH dependent. Heparinized blood is preferred because it can be assayed immediately. Serum may be used, but the specimen must remain tightly capped while clotting and centrifuging, and analyzed as soon as possible

Which of the following is characteristic of type 1 diabetes mellitus? A. Requires an oral glucose tolerance test for diagnosis B. Is the most common form of diabetes mellitus C. Usually occurs after age 40 D. Requires insulin replacement to prevent ketosis

D. Requires insulin replacement to prevent ketosis D Type 1, or juvenile, diabetes is also called insulindependent diabetes because patients must be given insulin to prevent ketosis. Type 1 accounts for only about 10%-20% of cases of diabetes mellitus, and is usually diagnosed by a fasting plasma glucose. Two consecutive results ≥126 mg/dL is diagnostic. Approximately 95% of patients produce autoantibodies against the beta cells of the pancreatic islets. Other autoantibodies may be produced against insulin, glutamate decarboxylase, and tyrosine phosphorylase IA2. There is genetic association between type 1 diabetes and human leukocyte antigens (HLA) DR3 and DR4.

SITUATION: A patient previously diagnosed with primary hypothyroidism and started on thyroxine replacement therapy is seen for follow-up testing after 2 weeks. Te serum-free T4 is normal but the TSH is still elevated. What is the most likely explanation for these results? A. Laboratory error in measurement of free T4 B. Laboratory error in measurement of TSH C. In vitro drug interference with the free T4 assay D. Results are consistent with a euthyroid patient in the early phase of therapy

D. Results are consistent with a euthyroid patient in the early phase of therapy D Results of thyroid tests (especially in hospitalized patients) may sometimes appear discrepant because medications and nonthyroid illnesses can affect test results. The pituitary is slow to respond to thyroxine replacement, and 6-8 weeks are usually required before TSH levels fall back to normal. In the early stage of therapy, the patient should be monitored by the free T4 result. This patient's free T4 is normal, indicating that replacement therapy is adequate. The high TSH sometimes seen in treated patients is called pituitary lag.

Which substance is used to generate the light signal in electrochemiluminescence? A. Acridinium B. Luminol C. Dioxetane phosphate D. Ruthenium

D. Ruthenium D All of these substances are chemiluminescent. Dioxetane phosphate is excited by alkaline phosphatase. Acridinium and luminol are excited by hydrogen peroxide. In electrochemiluminesence, ruthenium is used to label antibody or antigen. Antigen-antibody complexes containing the ruthenium label are bound to paramagnetic particles via a strepavidin-biotin reaction. The paramagnetic particles are attracted to an electrode surface. The flowcell is washed with a solution containing tripropylamine (TPA) to remove unbound ruthenium label. At the electrode surface, the TPA is oxidized and the electrons excite the ruthenium, causing production of 620-nm light

Which of the following trace elements is considered an essential micronutrient? A. Thallium B. Aluminum C. Mercury D. Selenium

D. Selenium D Trace elements can be divided into two categories, those that have no known biological purpose and those that do. The former include thallium, mercury, lead, cadmium, and aluminum. All others can be considered essential, including arsenic that has been shown necessary for normal methionine metabolism. Most trace elements are of medical importance because excessive levels lead to toxicity. However, a deficiency of trace elements such as selenium, zinc, and copper are commonly caused by total parenteral nutrition and are medically important.

Which test is the most sensitive in detecting early monoclonal gammopathies? A. High-resolution serum protein electrophoresis B. Urinary electrophoresis for monoclonal light chains C. Capillary electrophoresis of serum and urine D. Serum-free light chain immunoassay

D. Serum-free light chain immunoassay D Immunonephelometric free light chain assays can detect monoclonal protein production before the mass is sufficient to cause a monoclonal spike on protein electrophoresis or capillary electrophoresis, but will be positive only in cases where monoclonal light chain production occurs. Therefore, measurement of free light chains is recommended along with protein electrophoresis when testing for myeloma. Free light chains are normally present in serum because L chains are made at a faster rate than H chains. However, in cases where free L chains are the result of monoclonal plasma cell proliferation, the kappa:lambda ratio will be abnormal in addition to one of the L chain types being elevated

Which electrolyte is least likely to be elevated in renal failure? A. Potassium B. Magnesium C. Inorganic phosphorus D. Sodium

D. Sodium D Reduced glomerular filtration coupled with decreased tubular secretion causes accumulation of potassium, magnesium, and inorganic phosphorus. Poor tubular reabsorption of sodium offsets reduced glomerular filtration. Unfiltered sodium draws both chloride and water, causing osmotic equilibration between filtrate, serum, and the tissues. In renal disease, serum sodium is often normal, although total body sodium is increased owing to fluid and salt retention.

SITUATION: A blood sample in a red-stoppered tube is delivered to the laboratory for electrolytes, calcium, and phosphorus. The tube is approximately half full and is accompanied by a purple stoppered tube for a complete blood count that is approximately three-quarters full. The chemistry results are as follows: Na K Cl HCO3 Ca InP 135 mmol/L 11.2 mmol/L 103 mmol/L 14 mmol/L 2.6 mg/dL 3.8 mg/dL What is the most likely explanation of these serum calcium results? A. Severe hemolysis during sample collection B. Laboratory error in the calcium measurement C. The wrong order of draw was used for vacuum tube collection D. Some anticoagulated blood was added to the red-stoppered tube

D. Some anticoagulated blood was added to the red-stoppered tube D The potassium and the calcium results are above and below physiological limit values, respectively. Although hemolysis could explain the high potassium, hemolysis does not cause a significant change in serum calcium. The wrong order of draw could result in the falsely low calcium value but would not be sufficient to cause a result that is incompatible with life (and does not explain a grossly elevated potassium). The results and the condition of the tubes indicate that blood from a full tube collected in K3 EDTA was added to the clot tube, chelating the calcium and increasing the potassium.

When comparing the laboratory's monthly mean to its peer group to determine if bias is present, what statistic is most appropriate? A. F test B. Linear regression analysis C. Correlation coefficient D. Standard deviation index

D. Standard deviation index D The standard deviation index (SDI) compares the lab's mean to the peer group's mean in terms of standard deviations instead of concentration. This normalizes the value so that it is independent of mean, and allows performance comparisons for any analyte. The SDI equals the lab's mean minus the peer group's mean divided by the peer group's standard deviation. It has a similar probability distribution to a t test and a value greater than 2.0 is considered significant.

SITUATION: A sample for ammonia assay is taken from an IV line that had been capped and injected with lithium heparin (called a heparin lock). The sample is drawn in a syringe containing lithium heparin, and immediately capped and iced. The plasma is separated and analyzed within 20 minutes of collection, and the result is 50 µg/dL higher than one measured 4 hours before. What is the most likely explanation of these results? A. Significantly greater physiological variation is seen with patients having systemic, hepatic, and gastrointestinal diseases B. The syringe was contaminated with ammonia C. One of the two samples was collected from the wrong patient D. Stasis of blood in the line caused increased ammonia

D. Stasis of blood in the line caused increased ammonia D Falsely elevated blood ammonia levels are commonly caused by improper specimen collection. Venous stasis and prolonged storage cause peripheral deamination of amino acids, causing a falsely high ammonia level. Plasma is the sample of choice since ammonia levels increase with storage. Lithium heparin and EDTA are acceptable anticoagulants; the anticoagulant used should be tested to make sure it is free of ammonia. A vacuum tube can be used if filled completely. Serum may be used provided the tube is iced immediately, and the serum is separated as soon as the sample clots. The patient should be fasting and must not have smoked for 8 hours because tobacco smoke can double the plasma ammonia level.

A linearity study is performed on a visible spectrophotometer at 650 nm and the following absorbance readings are obtained: Concentration of Standard 10.0 mg/dL 20.0 mg/dL 30.0 mg/dL 40.0 mg/dL 50.0 mg/dL Absorbance 0.20 0.41 0.62 0.79 0.92 The study was repeated using freshly prepared standards and reagents, but results were identical to those shown. What is the most likely cause of these results? A. Wrong wavelength used B. Insufficient chromophore concentration C. Matrix interference D. Stray light

D. Stray light D Stray light is the most common cause of loss of linearity at high-analyte concentrations. Light transmitted through the cuvette is lowest when absorption is highest. Therefore, stray light is a greater percentage of the detector response when sample concentration is high. Stray light is usually most significant when measurements are made at the extremes of the visible spectrum because lamp output and detector response are low.

SITUATION: The following lab results are reported. Which result is most likely to be erroneous? Arterial blood gases: pH = 7.42 pO2 = 90 mm Hg pCO2 = 38.0 mm Hg bicarbonate = 24 mmol/L. Plasma electrolytes: Na = 135 mmol/L Cl = 98 mmol/L K = 4.6 mmol/L TCO2 = 33 mmol/L A. pH B. Na C. K D. TCO2

D. TCO2 D The pH, pCO2, and bicarbonate are normal, and therefore, agree. The electrolytes are normal also, but the TCO2 is increased significantly. The reference range for venous TCO2 is 22-28 mmol/L. Although TCO2 is the sum of bicarbonate and dissolved CO2, the venous TCO2 is determined almost entirely by the bicarbonate, since dCO2 is lost as CO2 gas when the venous blood is exposed to air during processing. A TCO2 value of 32 mmol/L would be expected in a person with metabolic alkalosis.

Select the most appropriate single screening test for thyroid disease. A. Free thyroxine index B. Total T3 assay C. Total T4 D. TSH assay

D. TSH assay D TSH is produced by the anterior pituitary in response to low levels of free T4 or T3. A normal TSH rules out thyroid disease. TSH is low in primary hyperthyroidism and high in primary hypothyroidism.

A female with severe excessive pubic and facial hair growth (hirsutism) should be tested for which of the following hormones? A. Estrogen and progesterone B. Chorionic gonadotropin C. Growth hormone D. Testosterone and dehydroepiandrosterone sulfate

D. Testosterone and dehydroepiandrosterone sulfate D Excessive hair grown in females results from excessive androgen production, and is most commonly seen in polycystic ovarian syndrome, which produces high levels of ovarian-derived testosterone. It will also occur as a consequence of Cushing's syndrome and mild congenital adrenal hyperplasia. Therefore, cortisol and 17 α-hydroxyprogesterone can help identify those causes. Rapid onset of hirsutism can result from an ovarian or adrenal tumor. Dehydroepiandrosterone sulfate is produced only by the adrenals and would be useful in identifying those rare cases where the cause is an androgen-secreting adrenal tumor

Which statement regarding nephelometry is true? A. Nephelometry is less sensitive than absorption spectrophotometry B. Nephelometry follows Beer's law C. The optical design is identical to a turbidimeter except that a HeNe laser light source is used D. The detector response is directly proportional to concentration

D. The detector response is directly proportional to concentration D In nephelometry, the detector output is proportional to concentration (as opposed to turbidimetry where the detector is behind the cuvette). The detector(s) is (are) usually placed at an angle between 25° and 90° to the incident light, depending upon the application. Nephelometers, like fluorometers, are calibrated to read zero with the light path blocked, and sensitivity can be increased up to 1,000 times by amplification of the detector output or increasing the photomultiplier tube dynode voltage

A sample of amniotic fluid collected for fetal lung maturity studies from a woman with a pregnancy compromised by hemolytic disease of the newborn (HDN) has a creatinine of 88 mg/dL. What is the most likely cause of this result? A. The specimen is contaminated with blood B. Bilirubin has interfered with the measurement of creatinine C. A random error occurred when the absorbance signal was being processed by the analyzer D. The fluid is urine from accidental puncture of the urinary bladder

D. The fluid is urine from accidental puncture of the urinary bladder D Creatinine levels in this range are found only in urine specimens. Adults usually excrete between 1.2 and 1.5 g of creatinine per day. For this reason, creatinine is routinely measured in 24-hour urine samples to determine the completeness of collection. A 24-hour urine with less than 0.8 g/day indicates that some of the urine was probably discarded. Creatinine is also used to evaluate fetal maturity. As gestation progresses, more creatinine is excreted into the amniotic fluid by the fetus. Although a level above 2 mg/dL is not a specific indicator of maturity, a level below 2 mg/dL indicates immaturity

Which explanation is the best interpretation of the following BUN bias plot? A. The new method consistently overestimates the BUN by a constant concentration B. The new method is greater than the reference method but not by a statistically significant margin C. The new method is lower than the reference method by 5 mg/dL D. The new method is lower than the reference and the magnitude is concentration dependent.

D. The new method is lower than the reference and the magnitude is concentration dependent. D A bias plot compares the bias (candidate method minus reference method) to the result of the reference method. Ideally, points should be scattered equally on both sides of the zero line. When the majority of points is below the zero line, the candidate method is negatively biased (lower than the reference). In this case, the difference between the methods increases in proportion to the BUN concentration. This type of plot occurs when the slope of the linear regression line is low

Which of the following conditions is most likely to cause a falsely high creatinine clearance result? A. The patient uses the midstream void procedure when collecting his or her urine B. The patient adds tap water to the urine container because he or she forgets to save one of the urine samples C. The patient does not empty his or her bladder at the conclusion of the test D. The patient empties his or her bladder at the start of the test and adds the urine to the collection

D. The patient empties his or her bladder at the start of the test and adds the urine to the collection D Urine in the bladder should be eliminated and not saved at the start of the test because it represents urine formed prior to the test period. The other conditions (choices A-C ) will result in falsely low urine creatinine or volume and, therefore, falsely lower clearance results. Error is introduced by incomplete emptying of the bladder when short times are used to measure clearance. A 24-hour timed urine is the specimen of choice. When filtrate flow falls below 2 mL/min, error is introduced because tubular secretion of creatinine occurs. The patient must be kept well hydrated during the test to prevent this.

Evaluate the following chromatogram of a whole-blood hemolysate, and identify the cause and best course of action. A. Result is not reportable because hemoglobin F is present and interferes B. The result is not reportable because hemoglobin C is present and interferes C. The result is not reportable because labile hemoglobin A1c is present D. The result is reportable; neither hemoglobin F or C interfere

D. The result is reportable; neither hemoglobin F or C interfere D The chromatogram is from a person with hemoglobin AC; however, hemoglobin C is completely separated from Hgb A1c and does not interfere. Hgb F is also present, but does not interfere unless its concentration is > 30%. Labile hemoglobin is formed initially when the aldehyde of glucose reacts with the N-terminal valine of the β globin chain. This Shiff base is reversible but is converted to Hgb A1c by rearrangement to a ketoamine. It is called labile A1c and produces a peak (LA1c) after HgF and before Hgb A1c. Therefore, it does not interfere.

Which of the following statements is correct? A. Both HDL and LDL are homogenous B. There are several subfractions of LDL but not HDL C. There are several subfractions of HDL but not LDL D. There are several subfractions of both HDL and LDL

D. There are several subfractions of both HDL and LDL D There are 7 subfractions of LDL and 10 subfractions of HDL. These are grouped into subclasses defined by their molecular sizes. In general, the small, dense LDL subclasses contain more oxidized LDL and are more atherogenic than the larger LDL molecules. The larger HDL subfractions comprising the HDL-3 subclass are associated with a lower risk of coronary artery disease.

In which of the following cases is qualitative analysis of the drug usually adequate? A. To determine whether the dose of a drug with a low therapeutic index is likely to be toxic B. To determine whether a patient is complying with the physician's instructions C. To adjust dose if individual differences or disease alter expected response D. To determine whether the patient has been taking amphetamines

D. To determine whether the patient has been taking amphetamines D The purpose of therapeutic drug monitoring is to achieve a therapeutic blood drug level rapidly and minimize the risk of drug toxicity caused by overdose. Therapeutic drug monitoring is a quantitative procedure performed for drugs with a narrow therapeutic index (ratio of the concentration producing the desired effect to the concentration producing toxicity). Drug groups that require monitoring because of high risk of toxicity include aminoglycoside antibiotics, anticonvulsants, antiarrhythmics, antiasthmatics, immunosuppressive agents used for transplant rejection, and psychoactive drugs. When testing for abuse substances, the goal is usually to determine whether the drug is present or absent. The most common approach is to compare the result to a cutoff determined by measuring a standard containing the lowest level of drug that is considered significant

Which of the following proteins migrates in the β region at pH 8.6? A. Haptoglobin B. Orosomucoprotein C. Antichymotrypsin D. Transferrin

D. Transferrin D Transferrin, β lipoprotein, C3, and C4 are the dominant proteins in the β-globulin region. Haptoglobin and α2- macroglobulin are the principal proteins in the α2-fraction. α1-Antitrypsin, α1-lipoprotein, and α1-acid glycoprotein (orosomucoprotein) make up most of the α1-fraction. Immunoglobulins dominate the γ region. Plasma is not used for protein electrophoresis because fibrinogen will produce a band resembling a small monoclonal protein in the beta region.

A patient treated for a germ cell tumor has a total and free β-hCG assay performed prior to surgery. The result is 40,000 mIU/mL. One week following surgery, the hCG is 5,000 mIU/mL. Chemotherapy is started, and the hCG is measured 1 week later and found to be 10,000 mIU/mL. What does this indicate? A. Recurrence of the tumor B. Falsely increased hCG owing to drug interference with the assay C. Analytical error with the test reported as 5,000 mIU/mL D. Transient hCG increase caused by chemotherapy

D. Transient hCG increase caused by chemotherapy D Treatment of tumors with chemotherapy often causes a transient increase in the production of tumor markers as the drugs destroy tumor cells. The half-life of hCG is 24-36 hours; therefore, the expected decline 1 week postsurgery was consistent with the result of 5,000 mIU/mL. Initiation of chemotherapy probably caused the hCG to double in the following week. The hCG assay should be monitored at regular intervals for several months, since a failure for it to decline or an increased level would suggest recurrence

Which of the following methods for HDL cholesterol is the reference method? A. Manganese-heparin B. Magnesium-phosphotungstate C. Magnesium-dextran D. Ultracentrifugation

D. Ultracentrifugation D Ultracentrifugation of plasma in a potassium bromide solution with a density of 1.063 is used to separate HDL from LDL and VLDL. The HDL fraction is transferred from the bottom of the tube and assayed for cholesterol content by the Abell-Kendall method. The remaining three methods rely upon selective precipitation of lipoproteins containing apoprotein B using a polyanionic solution. All of these methods are subject to interference by very high triglycerides and vary somewhat in specificity depending on the efficiency of precipitation.

Which of the following biochemical processes is promoted by insulin? A. Glycogenolysis B. Gluconeogenesis C. Lipolysis D. Uptake of glucose by cells

D. Uptake of glucose by cells D Insulin reduces blood glucose levels by increasing glucose uptake by cells. It promotes lipid and glycogen production, induces synthesis of glycolytic enzymes, and inhibits formation of glucose from pyruvate and Krebs cycle intermediates.

What role does vitamin D measurement play in the management of osteoporosis? A. Vitamin D deficiency must be demonstrated to establish the diagnosis B. Vitamin D is consistently elevated in osteoporosis C. A normal vitamin D level rules out osteoporosis D. Vitamin D deficiency is a risk factor for developing osteoporosis

D. Vitamin D deficiency is a risk factor for developing osteoporosis D Vitamin D assay is not used to diagnose osteoporosis. Vitamin D deficiency is a cause of secondary osteoporosis, and together with low PTH, calcium, and estrogen are important risk factors. If one or more of these is abnormal, then bone resorption or remodeling may be abnormal, predisposing one to osteoporosis. Deficiency of vitamin D also causes rickets (called osteomalacia in adults), a condition in which bones become soft owing to reduced deposition of hydroxyapatite.

Which of the following assays is used to determine the risk of developing cancer? A. Epidermal growth factor receptor (EGF-R) B. Squamous cell carcinoma antigen (SCC) C. c-erb B-2 gene expression D. p53 gene mutation

D. p53 gene mutation D The p53 gene (tumor suppressor gene) is located on chromosome 17 and produces a protein that down-regulates the cell cycle. A mutation of p53 is associated with an increased incidence of many cancers. The c-erb B-2 gene is the same as HER-2/neu; it codes for a growth factor receptor with tyrosine kinase activity on the cell membrane. EGF-R is a receptor for epidermal growth factor and its overexpression in breast tissue is associated with a poorer prognosis. SCC is a glycoprotein antigen found in the cytoplasm of tumors of squamous origin and is secreted in the plasma of persons with uterine cancer.

Which would be consistent with partially compensated respiratory acidosis? A. pH PCO2 Bicarbonate increased increased increased B. pH PCO2 Bicarbonate increased decreased decreased C. pH PCO2 Bicarbonate decreased decreased decreased D. pH PCO2 Bicarbonate decreased increased increased

D. pH PCO2 Bicarbonate decreased increased increased D Acidosis = low pH; respiratory = disturbance of PCO2; a low pH is caused by increased PCO2. In partially compensated respiratory acidosis, the metabolic component of the buffer system, bicarbonate, is retained. This helps to compensate for retention of PCO2 by titrating hydrogen ions. The compensatory component always moves in the same direction as the cause of the acid-base disturbance.

Select the most sensitive marker for alcoholic liver disease. A. GLD B. ALT C. AST D. γ-Glutamyltransferase (GGT)

D. γ-Glutamyltransferase (GGT) D Although AST and ALT are elevated in alcoholic hepatitis, GGT is a more sensitive indicator of alcoholic liver disease. Levels of GGT can reach in excess of 25 times the URL in alcoholic hepatitis. It is also markedly elevated in obstructive jaundice; a high GGT supports the inference that liver is the tissue source of an elevated ALP.


Kaugnay na mga set ng pag-aaral

Microeconomics Chapter 23- Perfect competition

View Set

ATI fundamentals practice test B

View Set

Bandura: Social Cognitive Theory

View Set

Excel Chapter 1: Creating a worksheet and charting data

View Set

Medical Terminology Ch. 3-Gastroenterology

View Set